Allowed Publications
Slot System
Featured Buckets
Featured Buckets Admin
Reverse Chronological Sort
Allow Teaser Image

Diversity, equity, and inclusion in reproductive health care

Article Type
Changed
Fri, 10/06/2023 - 16:18

A disproportionate and unique number of obstacles exist in reproductive health care for patients based on race, ethnicity, geography, socioeconomic, LGBTQ+, and disability status. These barriers represent inequality in access to reproductive medical services.

These challenges are also seen in other reproductive disorders such as polycystic ovary syndrome (PCOS), fibroids, and endometriosis. It is estimated that < 25% of individuals with infertility in the United States access the resources required to have their treatment needs met (Fertil Steril. 2015 Nov;104(5):1104-10. doi: 10.1016/j.fertnstert.2015.07.113)

Trolice_Mark _P_FLA_2022_web.jpg
Dr. Mark P. Trolice

In 2020, the American Society for Reproductive Medicine (ASRM) created a task force on Diversity, Equity, and Inclusion (DEI) chaired by Board Member Michael A. Thomas, MD. Two years later, the ASRM elevated this task force to a committee that is now chaired by Gloria Richard-Davis, MD. As health care systems and societies increasingly recognize these obstacles to care, I invited Dr. Thomas, the current president of the ASRM, to address this vital concern. Dr. Thomas is professor and chair, department of obstetrics and gynecology, at the University of Cincinnati.
 

While not limited to reproductive health care, how prevalent is the lack of DEI and what factors contribute to this problem?

When we established the initial ASRM DEI task force, we wanted to look at DEI issues within our profession and as an access-to-care initiative. We found that ASRM and ABOG (American Board of Obstetrics and Gynecology) were not asking questions about the makeup of our REI (Reproductive Endocrinology & Infertility) providers, nursing staff, and lab personnel. We had some older data from 2018 about the REI fellowships. Since that time, there appears to be an upward trend of people of color in REI fellowships.

Thomas_Michael_OH_web.jpg
Dr. Michael A. Thomas

We still need more data about academic, hybrid, and private REI practices when it comes to all employees. The goal would be to increase the number of people of color in all aspects of our field.

As far as access to care, we know that people of color do not have the ability to undergo ART (assisted reproductive technology) procedures at the same rate. This could be due to affordability, slower and/or later referral patterns, and personal stigma issues. Even in mandated states, people of color are seen by IVF providers in lower numbers. There is a need for a better understanding of the access-to-care issues, especially when affordability is not a problem, and the barriers to our LGBTQ+ patients.
 

Can you provide information about actions by the ASRM DEI task force and any plans for the future?

The DEI task force is now an ASRM committee. This committee is chaired by Dr. Gloria Richard-Davis and continues to work on increasing people of color in the REI workforce and understanding and decreasing access to care issues faced by people of color and members of the LGBTQ+ community.

 

 

What can physicians do at the local, state, and national level to support DEI?

All REI and ob.gyn. physicians can work with insurance companies to work on the current barriers that stand in the way of patients who want to have a family. For example, physicians can work with insurance companies to remove their definition of infertility as exposure to sperm for 1 year before fertility coverage can take effect. Also, mandated insurance coverage in all 50 states would allow even smaller companies to require this benefit to patients.

Many people of color work in smaller companies that, unfortunately, are not required to offer IVF coverage in states where mandated insurance coverage is available. As potential encouraging news, ASRM, RESOLVE (The National Infertility Association) and other patient advocacy groups are working with each state to help enact fertility mandates.
 

Which group, if any, has been most negatively affected by a lack of DEI?

People of color, LGBTQ+ communities, people with disabilities, single individuals, and those with income challenges are the most likely to be affected by adverse DEI policies.

While it is long overdue, why do you believe DEI has become such a touchstone and pervasive movement at this time?

This is the million-dollar question. After the George Floyd death, there was a global re-examination of how people of color were treated in every aspect of society. ASRM was the first to start this DEI initiative in women’s health.

ASRM and its patient advocacy partners are working with every nonmandated state toward the goal of passing infertility legislation to dramatically reduce the financial burden on all patients. We are starting to see more states either coming on board with mandates or at least discussing the possibilities. ASRM and RESOLVE have seen some recent positive outcomes with improved insurance for military families and government workers.
 

We can all agree that access to infertility treatment, particularly IVF, is not equivalent among different racial/ethnic populations. Part of the ASRM DEI task force is to evaluate research on IVF outcomes and race/ethnicity. Can you share why pregnancy outcomes would be included to potentially improve DEI?

More research needs to be done on pregnancy outcomes in women of color. We know that women of color have a decreased pregnancy rate in ART cycles even when controlling for age and other factors. We also know that birth outcomes are worse in these women. More understanding of this problem for women of color, especially African American women needs to be done.

Estimates are that more than one in eight LGBTQ+ patients live in states where physicians can refuse to treat them. Consequently, how can we improve DEI in these regions?

As someone with a number of family members in the LGBTQ+ community, this is a problem that is close to my heart. There appear to be many barriers that are being built to disenfranchise our LGBTQ+ community members. It is up to ASRM and patient advocacy groups to work with legislators to pass more inclusive laws and for insurance companies to update their definitions of infertility to be more inclusive for all.

Any final comments?

Everyone should have the right to become a parent whether they want to now or in the future!

Dr. Trolice is director of The IVF Center in Winter Park, Fla., and professor of obstetrics and gynecology at the University of Central Florida, Orlando.

Publications
Topics
Sections

A disproportionate and unique number of obstacles exist in reproductive health care for patients based on race, ethnicity, geography, socioeconomic, LGBTQ+, and disability status. These barriers represent inequality in access to reproductive medical services.

These challenges are also seen in other reproductive disorders such as polycystic ovary syndrome (PCOS), fibroids, and endometriosis. It is estimated that < 25% of individuals with infertility in the United States access the resources required to have their treatment needs met (Fertil Steril. 2015 Nov;104(5):1104-10. doi: 10.1016/j.fertnstert.2015.07.113)

Trolice_Mark _P_FLA_2022_web.jpg
Dr. Mark P. Trolice

In 2020, the American Society for Reproductive Medicine (ASRM) created a task force on Diversity, Equity, and Inclusion (DEI) chaired by Board Member Michael A. Thomas, MD. Two years later, the ASRM elevated this task force to a committee that is now chaired by Gloria Richard-Davis, MD. As health care systems and societies increasingly recognize these obstacles to care, I invited Dr. Thomas, the current president of the ASRM, to address this vital concern. Dr. Thomas is professor and chair, department of obstetrics and gynecology, at the University of Cincinnati.
 

While not limited to reproductive health care, how prevalent is the lack of DEI and what factors contribute to this problem?

When we established the initial ASRM DEI task force, we wanted to look at DEI issues within our profession and as an access-to-care initiative. We found that ASRM and ABOG (American Board of Obstetrics and Gynecology) were not asking questions about the makeup of our REI (Reproductive Endocrinology & Infertility) providers, nursing staff, and lab personnel. We had some older data from 2018 about the REI fellowships. Since that time, there appears to be an upward trend of people of color in REI fellowships.

Thomas_Michael_OH_web.jpg
Dr. Michael A. Thomas

We still need more data about academic, hybrid, and private REI practices when it comes to all employees. The goal would be to increase the number of people of color in all aspects of our field.

As far as access to care, we know that people of color do not have the ability to undergo ART (assisted reproductive technology) procedures at the same rate. This could be due to affordability, slower and/or later referral patterns, and personal stigma issues. Even in mandated states, people of color are seen by IVF providers in lower numbers. There is a need for a better understanding of the access-to-care issues, especially when affordability is not a problem, and the barriers to our LGBTQ+ patients.
 

Can you provide information about actions by the ASRM DEI task force and any plans for the future?

The DEI task force is now an ASRM committee. This committee is chaired by Dr. Gloria Richard-Davis and continues to work on increasing people of color in the REI workforce and understanding and decreasing access to care issues faced by people of color and members of the LGBTQ+ community.

 

 

What can physicians do at the local, state, and national level to support DEI?

All REI and ob.gyn. physicians can work with insurance companies to work on the current barriers that stand in the way of patients who want to have a family. For example, physicians can work with insurance companies to remove their definition of infertility as exposure to sperm for 1 year before fertility coverage can take effect. Also, mandated insurance coverage in all 50 states would allow even smaller companies to require this benefit to patients.

Many people of color work in smaller companies that, unfortunately, are not required to offer IVF coverage in states where mandated insurance coverage is available. As potential encouraging news, ASRM, RESOLVE (The National Infertility Association) and other patient advocacy groups are working with each state to help enact fertility mandates.
 

Which group, if any, has been most negatively affected by a lack of DEI?

People of color, LGBTQ+ communities, people with disabilities, single individuals, and those with income challenges are the most likely to be affected by adverse DEI policies.

While it is long overdue, why do you believe DEI has become such a touchstone and pervasive movement at this time?

This is the million-dollar question. After the George Floyd death, there was a global re-examination of how people of color were treated in every aspect of society. ASRM was the first to start this DEI initiative in women’s health.

ASRM and its patient advocacy partners are working with every nonmandated state toward the goal of passing infertility legislation to dramatically reduce the financial burden on all patients. We are starting to see more states either coming on board with mandates or at least discussing the possibilities. ASRM and RESOLVE have seen some recent positive outcomes with improved insurance for military families and government workers.
 

We can all agree that access to infertility treatment, particularly IVF, is not equivalent among different racial/ethnic populations. Part of the ASRM DEI task force is to evaluate research on IVF outcomes and race/ethnicity. Can you share why pregnancy outcomes would be included to potentially improve DEI?

More research needs to be done on pregnancy outcomes in women of color. We know that women of color have a decreased pregnancy rate in ART cycles even when controlling for age and other factors. We also know that birth outcomes are worse in these women. More understanding of this problem for women of color, especially African American women needs to be done.

Estimates are that more than one in eight LGBTQ+ patients live in states where physicians can refuse to treat them. Consequently, how can we improve DEI in these regions?

As someone with a number of family members in the LGBTQ+ community, this is a problem that is close to my heart. There appear to be many barriers that are being built to disenfranchise our LGBTQ+ community members. It is up to ASRM and patient advocacy groups to work with legislators to pass more inclusive laws and for insurance companies to update their definitions of infertility to be more inclusive for all.

Any final comments?

Everyone should have the right to become a parent whether they want to now or in the future!

Dr. Trolice is director of The IVF Center in Winter Park, Fla., and professor of obstetrics and gynecology at the University of Central Florida, Orlando.

A disproportionate and unique number of obstacles exist in reproductive health care for patients based on race, ethnicity, geography, socioeconomic, LGBTQ+, and disability status. These barriers represent inequality in access to reproductive medical services.

These challenges are also seen in other reproductive disorders such as polycystic ovary syndrome (PCOS), fibroids, and endometriosis. It is estimated that < 25% of individuals with infertility in the United States access the resources required to have their treatment needs met (Fertil Steril. 2015 Nov;104(5):1104-10. doi: 10.1016/j.fertnstert.2015.07.113)

Trolice_Mark _P_FLA_2022_web.jpg
Dr. Mark P. Trolice

In 2020, the American Society for Reproductive Medicine (ASRM) created a task force on Diversity, Equity, and Inclusion (DEI) chaired by Board Member Michael A. Thomas, MD. Two years later, the ASRM elevated this task force to a committee that is now chaired by Gloria Richard-Davis, MD. As health care systems and societies increasingly recognize these obstacles to care, I invited Dr. Thomas, the current president of the ASRM, to address this vital concern. Dr. Thomas is professor and chair, department of obstetrics and gynecology, at the University of Cincinnati.
 

While not limited to reproductive health care, how prevalent is the lack of DEI and what factors contribute to this problem?

When we established the initial ASRM DEI task force, we wanted to look at DEI issues within our profession and as an access-to-care initiative. We found that ASRM and ABOG (American Board of Obstetrics and Gynecology) were not asking questions about the makeup of our REI (Reproductive Endocrinology & Infertility) providers, nursing staff, and lab personnel. We had some older data from 2018 about the REI fellowships. Since that time, there appears to be an upward trend of people of color in REI fellowships.

Thomas_Michael_OH_web.jpg
Dr. Michael A. Thomas

We still need more data about academic, hybrid, and private REI practices when it comes to all employees. The goal would be to increase the number of people of color in all aspects of our field.

As far as access to care, we know that people of color do not have the ability to undergo ART (assisted reproductive technology) procedures at the same rate. This could be due to affordability, slower and/or later referral patterns, and personal stigma issues. Even in mandated states, people of color are seen by IVF providers in lower numbers. There is a need for a better understanding of the access-to-care issues, especially when affordability is not a problem, and the barriers to our LGBTQ+ patients.
 

Can you provide information about actions by the ASRM DEI task force and any plans for the future?

The DEI task force is now an ASRM committee. This committee is chaired by Dr. Gloria Richard-Davis and continues to work on increasing people of color in the REI workforce and understanding and decreasing access to care issues faced by people of color and members of the LGBTQ+ community.

 

 

What can physicians do at the local, state, and national level to support DEI?

All REI and ob.gyn. physicians can work with insurance companies to work on the current barriers that stand in the way of patients who want to have a family. For example, physicians can work with insurance companies to remove their definition of infertility as exposure to sperm for 1 year before fertility coverage can take effect. Also, mandated insurance coverage in all 50 states would allow even smaller companies to require this benefit to patients.

Many people of color work in smaller companies that, unfortunately, are not required to offer IVF coverage in states where mandated insurance coverage is available. As potential encouraging news, ASRM, RESOLVE (The National Infertility Association) and other patient advocacy groups are working with each state to help enact fertility mandates.
 

Which group, if any, has been most negatively affected by a lack of DEI?

People of color, LGBTQ+ communities, people with disabilities, single individuals, and those with income challenges are the most likely to be affected by adverse DEI policies.

While it is long overdue, why do you believe DEI has become such a touchstone and pervasive movement at this time?

This is the million-dollar question. After the George Floyd death, there was a global re-examination of how people of color were treated in every aspect of society. ASRM was the first to start this DEI initiative in women’s health.

ASRM and its patient advocacy partners are working with every nonmandated state toward the goal of passing infertility legislation to dramatically reduce the financial burden on all patients. We are starting to see more states either coming on board with mandates or at least discussing the possibilities. ASRM and RESOLVE have seen some recent positive outcomes with improved insurance for military families and government workers.
 

We can all agree that access to infertility treatment, particularly IVF, is not equivalent among different racial/ethnic populations. Part of the ASRM DEI task force is to evaluate research on IVF outcomes and race/ethnicity. Can you share why pregnancy outcomes would be included to potentially improve DEI?

More research needs to be done on pregnancy outcomes in women of color. We know that women of color have a decreased pregnancy rate in ART cycles even when controlling for age and other factors. We also know that birth outcomes are worse in these women. More understanding of this problem for women of color, especially African American women needs to be done.

Estimates are that more than one in eight LGBTQ+ patients live in states where physicians can refuse to treat them. Consequently, how can we improve DEI in these regions?

As someone with a number of family members in the LGBTQ+ community, this is a problem that is close to my heart. There appear to be many barriers that are being built to disenfranchise our LGBTQ+ community members. It is up to ASRM and patient advocacy groups to work with legislators to pass more inclusive laws and for insurance companies to update their definitions of infertility to be more inclusive for all.

Any final comments?

Everyone should have the right to become a parent whether they want to now or in the future!

Dr. Trolice is director of The IVF Center in Winter Park, Fla., and professor of obstetrics and gynecology at the University of Central Florida, Orlando.

Publications
Publications
Topics
Article Type
Sections
Teambase XML
<?xml version="1.0" encoding="UTF-8"?>
<!--$RCSfile: InCopy_agile.xsl,v $ $Revision: 1.35 $-->
<!--$RCSfile: drupal.xsl,v $ $Revision: 1.7 $-->
<root generator="drupal.xsl" gversion="1.7"> <header> <fileName>165375</fileName> <TBEID>0C04C871.SIG</TBEID> <TBUniqueIdentifier>MD_0C04C871</TBUniqueIdentifier> <newsOrJournal>News</newsOrJournal> <publisherName>Frontline Medical Communications</publisherName> <storyname/> <articleType>353</articleType> <TBLocation>QC Done-All Pubs</TBLocation> <QCDate>20231006T141048</QCDate> <firstPublished>20231006T160808</firstPublished> <LastPublished>20231006T160808</LastPublished> <pubStatus qcode="stat:"/> <embargoDate/> <killDate/> <CMSDate>20231006T160808</CMSDate> <articleSource/> <facebookInfo/> <meetingNumber/> <byline>Thomas and Trolice</byline> <bylineText>MICHAEL A. THOMAS, MD, AND MARK P. TROLICE MD, MBA</bylineText> <bylineFull>MICHAEL A. THOMAS, MD, AND MARK P. TROLICE MD, MBA</bylineFull> <bylineTitleText/> <USOrGlobal/> <wireDocType/> <newsDocType>Column</newsDocType> <journalDocType/> <linkLabel/> <pageRange/> <citation/> <quizID/> <indexIssueDate/> <itemClass qcode="ninat:text"/> <provider qcode="provider:imng"> <name>IMNG Medical Media</name> <rightsInfo> <copyrightHolder> <name>Frontline Medical News</name> </copyrightHolder> <copyrightNotice>Copyright (c) 2015 Frontline Medical News, a Frontline Medical Communications Inc. company. All rights reserved. This material may not be published, broadcast, copied, or otherwise reproduced or distributed without the prior written permission of Frontline Medical Communications Inc.</copyrightNotice> </rightsInfo> </provider> <abstract/> <metaDescription>A disproportionate and unique number of obstacles exist in reproductive health care for patients based on race, ethnicity, geography, socioeconomic, LGBTQ+, and</metaDescription> <articlePDF/> <teaserImage>284599</teaserImage> <teaser>People of color do not have the ability to undergo ART (assisted reproductive technology) procedures at the same rate as White people, says Dr. Michael Thomas.</teaser> <title>Diversity, equity, and inclusion in reproductive health care</title> <deck/> <disclaimer/> <AuthorList/> <articleURL/> <doi/> <pubMedID/> <publishXMLStatus/> <publishXMLVersion>1</publishXMLVersion> <useEISSN>0</useEISSN> <urgency/> <pubPubdateYear/> <pubPubdateMonth/> <pubPubdateDay/> <pubVolume/> <pubNumber/> <wireChannels/> <primaryCMSID/> <CMSIDs/> <keywords/> <seeAlsos/> <publications_g> <publicationData> <publicationCode>ob</publicationCode> <pubIssueName/> <pubArticleType/> <pubTopics/> <pubCategories/> <pubSections/> </publicationData> </publications_g> <publications> <term canonical="true">23</term> </publications> <sections> <term>52</term> <term canonical="true">68807</term> </sections> <topics> <term canonical="true">287</term> <term>66772</term> </topics> <links> <link> <itemClass qcode="ninat:picture"/> <altRep contenttype="image/jpeg">images/24010881.jpg</altRep> <description role="drol:caption">Dr. Mark P. Trolice</description> <description role="drol:credit">Fertility CARE</description> </link> <link> <itemClass qcode="ninat:picture"/> <altRep contenttype="image/jpeg">images/240122ac.jpg</altRep> <description role="drol:caption">Dr. Michael A. Thomas</description> <description role="drol:credit">University of Cincinnati</description> </link> </links> </header> <itemSet> <newsItem> <itemMeta> <itemRole>Main</itemRole> <itemClass>text</itemClass> <title>Diversity, equity, and inclusion in reproductive health care</title> <deck/> </itemMeta> <itemContent> <p><span class="tag metaDescription">A disproportionate and unique number of obstacles exist in reproductive health care for patients based on race, ethnicity, geography, socioeconomic, LGBTQ+, and disability status.</span> These barriers represent inequality in access to reproductive medical services.</p> <p>These challenges are also seen in other reproductive disorders such as polycystic ovary syndrome (PCOS), fibroids, and endometriosis. It is estimated that &lt; 25% of individuals with infertility in the United States access the resources required to have their treatment needs met (Fertil Steril. 2015 Nov;104(5):1104-10. doi: <span class="Hyperlink"><a href="https://www.fertstert.org/article/S0015-0282(15)01650-7/fulltext">10.1016/j.fertnstert.2015.07.113</a></span>)<br/><br/>[[{"fid":"284599","view_mode":"medstat_image_flush_left","fields":{"format":"medstat_image_flush_left","field_file_image_alt_text[und][0][value]":"Dr. Trolice is director of The IVF Center in Winter Park, Fla., and professor of obstetrics and gynecology at the University of Central Florida, Orlando.","field_file_image_credit[und][0][value]":"Fertility CARE","field_file_image_caption[und][0][value]":"Dr. Mark P. Trolice"},"type":"media","attributes":{"class":"media-element file-medstat_image_flush_left"}}]]In 2020, the American Society for Reproductive Medicine (ASRM) created a task force on Diversity, Equity, and Inclusion (DEI) chaired by Board Member Michael A. Thomas, MD. Two years later, the ASRM elevated this task force to a committee that is now chaired by Gloria Richard-Davis, MD. As health care systems and societies increasingly recognize these obstacles to care, I invited Dr. Thomas, the current president of the ASRM, to address this vital concern. Dr. Thomas is professor and chair, department of obstetrics and gynecology, at the University of Cincinnati.<br/><br/></p> <h2>While not limited to reproductive health care, how prevalent is the lack of DEI and what factors contribute to this problem?</h2> <p>When we established the initial ASRM DEI task force, we wanted to look at DEI issues within our profession and as an access-to-care initiative. We found that ASRM and ABOG (American Board of Obstetrics and Gynecology) were not asking questions about the makeup of our REI (Reproductive Endocrinology &amp; Infertility) providers, nursing staff, and lab personnel. We had some older data from 2018 about the REI fellowships. Since that time, there appears to be an upward trend of people of color in REI fellowships.</p> <p>[[{"fid":"298396","view_mode":"medstat_image_flush_right","fields":{"format":"medstat_image_flush_right","field_file_image_alt_text[und][0][value]":"Dr. Michael A. Thomas, Department of Obstetrics and Gynecology at the University of Cincinnati College of Medicine","field_file_image_credit[und][0][value]":"University of Cincinnati","field_file_image_caption[und][0][value]":"Dr. Michael A. Thomas"},"type":"media","attributes":{"class":"media-element file-medstat_image_flush_right"}}]]We still need more data about academic, hybrid, and private REI practices when it comes to all employees. The goal would be to increase the number of people of color in all aspects of our field.<br/><br/>As far as access to care, we know that people of color do not have the ability to undergo ART (assisted reproductive technology) procedures at the same rate. This could be due to affordability, slower and/or later referral patterns, and personal stigma issues. Even in mandated states, people of color are seen by IVF providers in lower numbers. There is a need for a better understanding of the access-to-care issues, especially when affordability is not a problem, and the barriers to our LGBTQ+ patients.<br/><br/></p> <h2>Can you provide information about actions by the ASRM DEI task force and any plans for the future?</h2> <p>The DEI task force is now an ASRM committee. This committee is chaired by Dr. Gloria Richard-Davis and continues to work on increasing people of color in the REI workforce and understanding and decreasing access to care issues faced by people of color and members of the LGBTQ+ community.</p> <h2>What can physicians do at the local, state, and national level to support DEI?</h2> <p>All REI and ob.gyn. physicians can work with insurance companies to work on the current barriers that stand in the way of patients who want to have a family. For example, physicians can work with insurance companies to remove their definition of infertility as exposure to sperm for 1 year before fertility coverage can take effect. Also, mandated insurance coverage in all 50 states would allow even smaller companies to require this benefit to patients.</p> <p>Many people of color work in smaller companies that, unfortunately, are not required to offer IVF coverage in states where mandated insurance coverage is available. As potential encouraging news, ASRM, RESOLVE (The National Infertility Association) and other patient advocacy groups are working with each state to help enact fertility mandates.<br/><br/></p> <h2>Which group, if any, has been most negatively affected by a lack of DEI?</h2> <p>People of color, LGBTQ+ communities, people with disabilities, single individuals, and those with income challenges are the most likely to be affected by adverse DEI policies.</p> <h2>While it is long overdue, why do you believe DEI has become such a touchstone and pervasive movement at this time?</h2> <p>This is the million-dollar question. After the George Floyd death, there was a global re-examination of how people of color were treated in every aspect of society. ASRM was the first to start this DEI initiative in women’s health.</p> <p>ASRM and its patient advocacy partners are working with every nonmandated state toward the goal of passing infertility legislation to dramatically reduce the financial burden on all patients. We are starting to see more states either coming on board with mandates or at least discussing the possibilities. ASRM and RESOLVE have seen some recent positive outcomes with improved insurance for military families and government workers.<br/><br/></p> <h2>We can all agree that access to infertility treatment, particularly IVF, is not equivalent among different racial/ethnic populations. Part of the ASRM DEI task force is to evaluate research on IVF outcomes and race/ethnicity. Can you share why pregnancy outcomes would be included to potentially improve DEI?</h2> <p>More research needs to be done on pregnancy outcomes in women of color. We know that women of color have a decreased pregnancy rate in ART cycles even when controlling for age and other factors. We also know that birth outcomes are worse in these women. More understanding of this problem for women of color, especially African American women needs to be done.</p> <h2>Estimates are that more than one in eight LGBTQ+ patients live in states where <span class="Hyperlink"><a href="https://www.nbcnews.com/nbc-out/out-health-and-wellness/1-8-lgbtq-people-live-states-doctors-can-refuse-treat-rcna39161">physicians can refuse to treat them</a></span>. Consequently, how can we improve DEI in these regions?</h2> <p>As someone with a number of family members in the LGBTQ+ community, this is a problem that is close to my heart. There appear to be many barriers that are being built to disenfranchise our LGBTQ+ community members. It is up to ASRM and patient advocacy groups to work with legislators to pass more inclusive laws and for insurance companies to update their definitions of infertility to be more inclusive for all.</p> <h2>Any final comments?</h2> <p>Everyone should have the right to become a parent whether they want to now or in the future!<span class="end"/></p> <p> <em>Dr. Trolice is director of The IVF Center in Winter Park, Fla., and professor of obstetrics and gynecology at the University of Central Florida, Orlando.</em> </p> </itemContent> </newsItem> <newsItem> <itemMeta> <itemRole>teaser</itemRole> <itemClass>text</itemClass> <title/> <deck/> </itemMeta> <itemContent> </itemContent> </newsItem> </itemSet></root>
Disallow All Ads
Content Gating
No Gating (article Unlocked/Free)
Alternative CME
Disqus Comments
Default
Use ProPublica
Hide sidebar & use full width
render the right sidebar.
Conference Recap Checkbox
Not Conference Recap
Clinical Edge
Display the Slideshow in this Article
Medscape Article
Display survey writer
Reuters content
Disable Inline Native ads
WebMD Article

Affordable IVF – Are we there yet?

Article Type
Changed
Fri, 07/14/2023 - 17:22

 

The price for an in vitro fertilization (IVF) cycle continues to increase annually by many clinics, particularly because of “add-ons” of dubious value.

Trolice_Mark _P_FLA_2022_web.jpg
Dr. Mark P. Trolice

The initial application of IVF was for tubal factor infertility. Over the decades since 1981, the year of the first successful live birth in the United States, indications for IVF have dramatically expanded – ovulation dysfunction, unexplained infertility, male factor, advanced stage endometriosis, unexplained infertility, embryo testing to avoid an inherited genetic disease from the intended parents carrying the same mutation, and family balancing for gender, along with fertility preservation, including before potentially gonadotoxic treatment and “elective” planned oocyte cryopreservation.

The cost of IVF remains a significant, and possibly leading, stumbling block for women, couples, and men who lack insurance coverage. From RESOLVE.org, the National Infertility Association: “As of June 2022, 20 states have passed fertility insurance coverage laws, 14 of those laws include IVF coverage, and 12 states have fertility preservation laws for iatrogenic (medically induced) infertility.” Consequently, “affordable IVF” is paramount to providing equal access for patients.

Doody_Kevin_TX_web.jpg
Dr. Kevin Doody

I spoke with the past president of The Society for Assisted Reproductive Technology (SART.org), Kevin Doody, MD, HCLD, to discuss current IVF treatment options for couples that may decrease their financial burden, particularly by applying a novel approach – called INVOcell – that involves using the woman’s vagina as the embryo “incubator.” Dr. Doody is director of CARE Fertility in Bedford, Tex., and clinical professor at UT Southwestern Medical Center, Dallas.
 

How does limiting the dosage of gonadotropins in IVF cycles, known as “minimal stimulation,” affect pregnancy outcomes?

IVF medications are often costly, so it is logical to try and minimize expenses by using them judiciously. “Minimal stimulation” generally is not the best approach, as having more eggs usually leads to better pregnancy rates. High egg yield increases short-term success and provides additional embryos for future attempts.

However, extremely high gonadotropin doses do not necessarily yield more eggs or successful pregnancies. The dose response to gonadotropins follows a sigmoid curve, and typically doses beyond 225-300 IU per day do not offer additional benefits, except for women with an elevated body weight. Yet, some physicians continue to use higher doses in women with low ovarian reserve, which is often not beneficial and can add unnecessary costs.
 

Is “natural cycle” IVF cost-effective with acceptable pregnancy success rates?

Although the first-ever IVF baby was conceived through a natural cycle, this approach has very low success rates. Even with advancements in IVF laboratory technologies, the outcomes of natural cycle IVF have remained disappointingly low and are generally considered unacceptable.

Are there other cost-saving alternatives for IVF that still maintain reasonable success rates?

 

 

Some patients can undergo a more simplified ovarian stimulation protocol that reduces the number of monitoring visits, thus reducing costs. In couples without a severe male factor, the application and additional expense of intracytoplasmic sperm injection (ICSI) is unnecessary. Pre-implantation genetic testing for embryo aneuploidy, another “add-on” procedure, has specific indications and medical evidence does not support its use in all patient cycles.

How can the cost of a standard IVF cycle be reduced, especially in areas without mandated infertility insurance coverage?

Addressing this issue involves considering principles of justice in medical ethics, which emphasize equal health care access for all individuals. Infertility is a medical condition and IVF is expensive, so lack of insurance coverage often restricts access. Our clinic offers a more affordable option called “effortless IVF” using an intravaginal culture system (INVOcell), which minimizes the monitoring process while maintaining satisfactory success rates and reducing the risks associated with ovarian hyperstimulation syndrome.

What is INVOcell, and how successful is it in terms of live birth rates?

INVOcell is an innovative approach to IVF, where an intravaginal culture system is used as an “embryo incubator whereby freshly harvested eggs along with sperm are immediately added to a small chamber device that is placed in the woman’s vagina for up to 5 days to allow for fertilization and embryo development.” The woman, typically, has no discomfort from the device. For appropriately selected patients, the literature has shown live birth rates are comparable to those achieved using conventional laboratory incubation systems.

As an early participant in INVOcell research, can you share insights on the ideal candidates for this procedure and any contraindications?

The INVOcell system is best suited for straightforward cases. It is not recommended for severe male factor infertility requiring ICSI, since this will delay application of the chamber device and increase cost. Further, cases involving preimplantation genetic testing are not recommended because the embryos may not develop synchronously within the device to the embryo stage needed for a biopsy.

What training is required for embryologists and physicians to use INVOcell?

Embryologists require training for a few hours to learn the basics of INVOcell. They must master loading eggs into and retrieving embryos from the device. Practicing on discarded eggs and embryos, embryologists can accelerate the acquisition of the proper technique needed for INVOcell. Physicians find the training easier; they mainly need to learn the correct placement and removal of the device in the vagina.
 

Is INVOcell gaining acceptance among patients and IVF centers?

Acceptance varies. In our practice, INVOcell has largely replaced superovulation and intrauterine insemination treatments. However, some clinics still need to determine how this tool fits within their practice.

Have IVF success rates plateaued as affordable options increase?

IVF success rates grew substantially in the 1980s and 1990s, fostered by improved embryo culture systems and higher numbers of embryos transferred, the latter at the expense of a multiple gestation. While the rate of improvement has slowed, coinciding with the increasing use of single embryo transfer, advancements in IVF continue toward the goal of improving the singleton live birth rate per IVF cycle. There is still room for enhancement in success rates alongside cost reduction. Continued innovation is needed, especially for patients with challenging underlying biological issues.

 

 

Can you provide insight into the next potential breakthrough in IVF that may reduce costs, be less invasive, and maintain optimal pregnancy rates?

I am very excited about recent breakthroughs in in vitro maturation (IVM) of oocytes. The bottleneck in IVF clinics (and significant expense) primarily relates to the need to stimulate the ovaries to get mature and competent eggs. The technology of IVM has existed for decades but has yet to be fully embraced by clinics because of the poor competency of oocytes matured in the laboratory.

Immature eggs resume meiosis immediately upon removal from the ovary. Nuclear maturation of eggs in the lab is easy. In fact, it happens too quickly, thereby not allowing for the maturation of the egg cytoplasm. This has previously led to poor development of embryos following fertilization and low success rates.

Recently, a new laboratory strategy has resulted in a significant improvement in success. This improved culture system uses a peptide that prevents the resumption of meiosis for the initial culture time frame. Substances, including follicle stimulating hormone, can be added to the media to promote oocyte cytoplasmic maturation. Following this, the eggs are placed in a media without the meiosis inhibitor to allow for nuclear maturation. This results in a significantly higher proportion of competent mature eggs.



Dr. Trolice is director of The IVF Center in Winter Park, Fla., and professor of obstetrics and gynecology at the University of Central Florida, Orlando.

Publications
Topics
Sections

 

The price for an in vitro fertilization (IVF) cycle continues to increase annually by many clinics, particularly because of “add-ons” of dubious value.

Trolice_Mark _P_FLA_2022_web.jpg
Dr. Mark P. Trolice

The initial application of IVF was for tubal factor infertility. Over the decades since 1981, the year of the first successful live birth in the United States, indications for IVF have dramatically expanded – ovulation dysfunction, unexplained infertility, male factor, advanced stage endometriosis, unexplained infertility, embryo testing to avoid an inherited genetic disease from the intended parents carrying the same mutation, and family balancing for gender, along with fertility preservation, including before potentially gonadotoxic treatment and “elective” planned oocyte cryopreservation.

The cost of IVF remains a significant, and possibly leading, stumbling block for women, couples, and men who lack insurance coverage. From RESOLVE.org, the National Infertility Association: “As of June 2022, 20 states have passed fertility insurance coverage laws, 14 of those laws include IVF coverage, and 12 states have fertility preservation laws for iatrogenic (medically induced) infertility.” Consequently, “affordable IVF” is paramount to providing equal access for patients.

Doody_Kevin_TX_web.jpg
Dr. Kevin Doody

I spoke with the past president of The Society for Assisted Reproductive Technology (SART.org), Kevin Doody, MD, HCLD, to discuss current IVF treatment options for couples that may decrease their financial burden, particularly by applying a novel approach – called INVOcell – that involves using the woman’s vagina as the embryo “incubator.” Dr. Doody is director of CARE Fertility in Bedford, Tex., and clinical professor at UT Southwestern Medical Center, Dallas.
 

How does limiting the dosage of gonadotropins in IVF cycles, known as “minimal stimulation,” affect pregnancy outcomes?

IVF medications are often costly, so it is logical to try and minimize expenses by using them judiciously. “Minimal stimulation” generally is not the best approach, as having more eggs usually leads to better pregnancy rates. High egg yield increases short-term success and provides additional embryos for future attempts.

However, extremely high gonadotropin doses do not necessarily yield more eggs or successful pregnancies. The dose response to gonadotropins follows a sigmoid curve, and typically doses beyond 225-300 IU per day do not offer additional benefits, except for women with an elevated body weight. Yet, some physicians continue to use higher doses in women with low ovarian reserve, which is often not beneficial and can add unnecessary costs.
 

Is “natural cycle” IVF cost-effective with acceptable pregnancy success rates?

Although the first-ever IVF baby was conceived through a natural cycle, this approach has very low success rates. Even with advancements in IVF laboratory technologies, the outcomes of natural cycle IVF have remained disappointingly low and are generally considered unacceptable.

Are there other cost-saving alternatives for IVF that still maintain reasonable success rates?

 

 

Some patients can undergo a more simplified ovarian stimulation protocol that reduces the number of monitoring visits, thus reducing costs. In couples without a severe male factor, the application and additional expense of intracytoplasmic sperm injection (ICSI) is unnecessary. Pre-implantation genetic testing for embryo aneuploidy, another “add-on” procedure, has specific indications and medical evidence does not support its use in all patient cycles.

How can the cost of a standard IVF cycle be reduced, especially in areas without mandated infertility insurance coverage?

Addressing this issue involves considering principles of justice in medical ethics, which emphasize equal health care access for all individuals. Infertility is a medical condition and IVF is expensive, so lack of insurance coverage often restricts access. Our clinic offers a more affordable option called “effortless IVF” using an intravaginal culture system (INVOcell), which minimizes the monitoring process while maintaining satisfactory success rates and reducing the risks associated with ovarian hyperstimulation syndrome.

What is INVOcell, and how successful is it in terms of live birth rates?

INVOcell is an innovative approach to IVF, where an intravaginal culture system is used as an “embryo incubator whereby freshly harvested eggs along with sperm are immediately added to a small chamber device that is placed in the woman’s vagina for up to 5 days to allow for fertilization and embryo development.” The woman, typically, has no discomfort from the device. For appropriately selected patients, the literature has shown live birth rates are comparable to those achieved using conventional laboratory incubation systems.

As an early participant in INVOcell research, can you share insights on the ideal candidates for this procedure and any contraindications?

The INVOcell system is best suited for straightforward cases. It is not recommended for severe male factor infertility requiring ICSI, since this will delay application of the chamber device and increase cost. Further, cases involving preimplantation genetic testing are not recommended because the embryos may not develop synchronously within the device to the embryo stage needed for a biopsy.

What training is required for embryologists and physicians to use INVOcell?

Embryologists require training for a few hours to learn the basics of INVOcell. They must master loading eggs into and retrieving embryos from the device. Practicing on discarded eggs and embryos, embryologists can accelerate the acquisition of the proper technique needed for INVOcell. Physicians find the training easier; they mainly need to learn the correct placement and removal of the device in the vagina.
 

Is INVOcell gaining acceptance among patients and IVF centers?

Acceptance varies. In our practice, INVOcell has largely replaced superovulation and intrauterine insemination treatments. However, some clinics still need to determine how this tool fits within their practice.

Have IVF success rates plateaued as affordable options increase?

IVF success rates grew substantially in the 1980s and 1990s, fostered by improved embryo culture systems and higher numbers of embryos transferred, the latter at the expense of a multiple gestation. While the rate of improvement has slowed, coinciding with the increasing use of single embryo transfer, advancements in IVF continue toward the goal of improving the singleton live birth rate per IVF cycle. There is still room for enhancement in success rates alongside cost reduction. Continued innovation is needed, especially for patients with challenging underlying biological issues.

 

 

Can you provide insight into the next potential breakthrough in IVF that may reduce costs, be less invasive, and maintain optimal pregnancy rates?

I am very excited about recent breakthroughs in in vitro maturation (IVM) of oocytes. The bottleneck in IVF clinics (and significant expense) primarily relates to the need to stimulate the ovaries to get mature and competent eggs. The technology of IVM has existed for decades but has yet to be fully embraced by clinics because of the poor competency of oocytes matured in the laboratory.

Immature eggs resume meiosis immediately upon removal from the ovary. Nuclear maturation of eggs in the lab is easy. In fact, it happens too quickly, thereby not allowing for the maturation of the egg cytoplasm. This has previously led to poor development of embryos following fertilization and low success rates.

Recently, a new laboratory strategy has resulted in a significant improvement in success. This improved culture system uses a peptide that prevents the resumption of meiosis for the initial culture time frame. Substances, including follicle stimulating hormone, can be added to the media to promote oocyte cytoplasmic maturation. Following this, the eggs are placed in a media without the meiosis inhibitor to allow for nuclear maturation. This results in a significantly higher proportion of competent mature eggs.



Dr. Trolice is director of The IVF Center in Winter Park, Fla., and professor of obstetrics and gynecology at the University of Central Florida, Orlando.

 

The price for an in vitro fertilization (IVF) cycle continues to increase annually by many clinics, particularly because of “add-ons” of dubious value.

Trolice_Mark _P_FLA_2022_web.jpg
Dr. Mark P. Trolice

The initial application of IVF was for tubal factor infertility. Over the decades since 1981, the year of the first successful live birth in the United States, indications for IVF have dramatically expanded – ovulation dysfunction, unexplained infertility, male factor, advanced stage endometriosis, unexplained infertility, embryo testing to avoid an inherited genetic disease from the intended parents carrying the same mutation, and family balancing for gender, along with fertility preservation, including before potentially gonadotoxic treatment and “elective” planned oocyte cryopreservation.

The cost of IVF remains a significant, and possibly leading, stumbling block for women, couples, and men who lack insurance coverage. From RESOLVE.org, the National Infertility Association: “As of June 2022, 20 states have passed fertility insurance coverage laws, 14 of those laws include IVF coverage, and 12 states have fertility preservation laws for iatrogenic (medically induced) infertility.” Consequently, “affordable IVF” is paramount to providing equal access for patients.

Doody_Kevin_TX_web.jpg
Dr. Kevin Doody

I spoke with the past president of The Society for Assisted Reproductive Technology (SART.org), Kevin Doody, MD, HCLD, to discuss current IVF treatment options for couples that may decrease their financial burden, particularly by applying a novel approach – called INVOcell – that involves using the woman’s vagina as the embryo “incubator.” Dr. Doody is director of CARE Fertility in Bedford, Tex., and clinical professor at UT Southwestern Medical Center, Dallas.
 

How does limiting the dosage of gonadotropins in IVF cycles, known as “minimal stimulation,” affect pregnancy outcomes?

IVF medications are often costly, so it is logical to try and minimize expenses by using them judiciously. “Minimal stimulation” generally is not the best approach, as having more eggs usually leads to better pregnancy rates. High egg yield increases short-term success and provides additional embryos for future attempts.

However, extremely high gonadotropin doses do not necessarily yield more eggs or successful pregnancies. The dose response to gonadotropins follows a sigmoid curve, and typically doses beyond 225-300 IU per day do not offer additional benefits, except for women with an elevated body weight. Yet, some physicians continue to use higher doses in women with low ovarian reserve, which is often not beneficial and can add unnecessary costs.
 

Is “natural cycle” IVF cost-effective with acceptable pregnancy success rates?

Although the first-ever IVF baby was conceived through a natural cycle, this approach has very low success rates. Even with advancements in IVF laboratory technologies, the outcomes of natural cycle IVF have remained disappointingly low and are generally considered unacceptable.

Are there other cost-saving alternatives for IVF that still maintain reasonable success rates?

 

 

Some patients can undergo a more simplified ovarian stimulation protocol that reduces the number of monitoring visits, thus reducing costs. In couples without a severe male factor, the application and additional expense of intracytoplasmic sperm injection (ICSI) is unnecessary. Pre-implantation genetic testing for embryo aneuploidy, another “add-on” procedure, has specific indications and medical evidence does not support its use in all patient cycles.

How can the cost of a standard IVF cycle be reduced, especially in areas without mandated infertility insurance coverage?

Addressing this issue involves considering principles of justice in medical ethics, which emphasize equal health care access for all individuals. Infertility is a medical condition and IVF is expensive, so lack of insurance coverage often restricts access. Our clinic offers a more affordable option called “effortless IVF” using an intravaginal culture system (INVOcell), which minimizes the monitoring process while maintaining satisfactory success rates and reducing the risks associated with ovarian hyperstimulation syndrome.

What is INVOcell, and how successful is it in terms of live birth rates?

INVOcell is an innovative approach to IVF, where an intravaginal culture system is used as an “embryo incubator whereby freshly harvested eggs along with sperm are immediately added to a small chamber device that is placed in the woman’s vagina for up to 5 days to allow for fertilization and embryo development.” The woman, typically, has no discomfort from the device. For appropriately selected patients, the literature has shown live birth rates are comparable to those achieved using conventional laboratory incubation systems.

As an early participant in INVOcell research, can you share insights on the ideal candidates for this procedure and any contraindications?

The INVOcell system is best suited for straightforward cases. It is not recommended for severe male factor infertility requiring ICSI, since this will delay application of the chamber device and increase cost. Further, cases involving preimplantation genetic testing are not recommended because the embryos may not develop synchronously within the device to the embryo stage needed for a biopsy.

What training is required for embryologists and physicians to use INVOcell?

Embryologists require training for a few hours to learn the basics of INVOcell. They must master loading eggs into and retrieving embryos from the device. Practicing on discarded eggs and embryos, embryologists can accelerate the acquisition of the proper technique needed for INVOcell. Physicians find the training easier; they mainly need to learn the correct placement and removal of the device in the vagina.
 

Is INVOcell gaining acceptance among patients and IVF centers?

Acceptance varies. In our practice, INVOcell has largely replaced superovulation and intrauterine insemination treatments. However, some clinics still need to determine how this tool fits within their practice.

Have IVF success rates plateaued as affordable options increase?

IVF success rates grew substantially in the 1980s and 1990s, fostered by improved embryo culture systems and higher numbers of embryos transferred, the latter at the expense of a multiple gestation. While the rate of improvement has slowed, coinciding with the increasing use of single embryo transfer, advancements in IVF continue toward the goal of improving the singleton live birth rate per IVF cycle. There is still room for enhancement in success rates alongside cost reduction. Continued innovation is needed, especially for patients with challenging underlying biological issues.

 

 

Can you provide insight into the next potential breakthrough in IVF that may reduce costs, be less invasive, and maintain optimal pregnancy rates?

I am very excited about recent breakthroughs in in vitro maturation (IVM) of oocytes. The bottleneck in IVF clinics (and significant expense) primarily relates to the need to stimulate the ovaries to get mature and competent eggs. The technology of IVM has existed for decades but has yet to be fully embraced by clinics because of the poor competency of oocytes matured in the laboratory.

Immature eggs resume meiosis immediately upon removal from the ovary. Nuclear maturation of eggs in the lab is easy. In fact, it happens too quickly, thereby not allowing for the maturation of the egg cytoplasm. This has previously led to poor development of embryos following fertilization and low success rates.

Recently, a new laboratory strategy has resulted in a significant improvement in success. This improved culture system uses a peptide that prevents the resumption of meiosis for the initial culture time frame. Substances, including follicle stimulating hormone, can be added to the media to promote oocyte cytoplasmic maturation. Following this, the eggs are placed in a media without the meiosis inhibitor to allow for nuclear maturation. This results in a significantly higher proportion of competent mature eggs.



Dr. Trolice is director of The IVF Center in Winter Park, Fla., and professor of obstetrics and gynecology at the University of Central Florida, Orlando.

Publications
Publications
Topics
Article Type
Sections
Teambase XML
<?xml version="1.0" encoding="UTF-8"?>
<!--$RCSfile: InCopy_agile.xsl,v $ $Revision: 1.35 $-->
<!--$RCSfile: drupal.xsl,v $ $Revision: 1.7 $-->
<root generator="drupal.xsl" gversion="1.7"> <header> <fileName>164207</fileName> <TBEID>0C04B0F0.SIG</TBEID> <TBUniqueIdentifier>MD_0C04B0F0</TBUniqueIdentifier> <newsOrJournal>News</newsOrJournal> <publisherName>Frontline Medical Communications</publisherName> <storyname>Reproductive Rounds column</storyname> <articleType>353</articleType> <TBLocation>QC Done-All Pubs</TBLocation> <QCDate>20230714T165218</QCDate> <firstPublished>20230714T165329</firstPublished> <LastPublished>20230714T165329</LastPublished> <pubStatus qcode="stat:"/> <embargoDate/> <killDate/> <CMSDate>20230714T165328</CMSDate> <articleSource/> <facebookInfo/> <meetingNumber/> <byline>Other Writer</byline> <bylineText>KEVIN DOODY, MD; MARK TROLICE MD, MBA</bylineText> <bylineFull>KEVIN DOODY, MD; MARK TROLICE MD, MBA</bylineFull> <bylineTitleText/> <USOrGlobal/> <wireDocType/> <newsDocType>Column</newsDocType> <journalDocType/> <linkLabel/> <pageRange/> <citation/> <quizID/> <indexIssueDate/> <itemClass qcode="ninat:text"/> <provider qcode="provider:imng"> <name>IMNG Medical Media</name> <rightsInfo> <copyrightHolder> <name>Frontline Medical News</name> </copyrightHolder> <copyrightNotice>Copyright (c) 2015 Frontline Medical News, a Frontline Medical Communications Inc. company. All rights reserved. This material may not be published, broadcast, copied, or otherwise reproduced or distributed without the prior written permission of Frontline Medical Communications Inc.</copyrightNotice> </rightsInfo> </provider> <abstract/> <metaDescription>The cost of IVF remains a significant, and possibly leading, stumbling block for women, couples, and men who lack insurance coverage.</metaDescription> <articlePDF/> <teaserImage>284599</teaserImage> <teaser>The price for an in vitro fertilization cycle continues to increase annually, particularly because of “add-ons” of dubious value, says Dr. Kevin Doody.</teaser> <title>Affordable IVF – Are we there yet?</title> <deck/> <disclaimer/> <AuthorList/> <articleURL/> <doi/> <pubMedID/> <publishXMLStatus/> <publishXMLVersion>1</publishXMLVersion> <useEISSN>0</useEISSN> <urgency/> <pubPubdateYear/> <pubPubdateMonth/> <pubPubdateDay/> <pubVolume/> <pubNumber/> <wireChannels/> <primaryCMSID/> <CMSIDs/> <keywords/> <seeAlsos/> <publications_g> <publicationData> <publicationCode>ob</publicationCode> <pubIssueName/> <pubArticleType/> <pubTopics/> <pubCategories/> <pubSections/> </publicationData> </publications_g> <publications> <term canonical="true">23</term> </publications> <sections> <term canonical="true">68807</term> <term>52</term> </sections> <topics> <term canonical="true">287</term> </topics> <links> <link> <itemClass qcode="ninat:picture"/> <altRep contenttype="image/jpeg">images/24010881.jpg</altRep> <description role="drol:caption">Dr. Mark P. Trolice</description> <description role="drol:credit">Fertility CARE</description> </link> <link> <itemClass qcode="ninat:picture"/> <altRep contenttype="image/jpeg">images/24011fd2.jpg</altRep> <description role="drol:caption">Dr. Kevin Doody</description> <description role="drol:credit">CARE Fertility</description> </link> </links> </header> <itemSet> <newsItem> <itemMeta> <itemRole>Main</itemRole> <itemClass>text</itemClass> <title>Affordable IVF – Are we there yet?</title> <deck/> </itemMeta> <itemContent> <p>The price for an in vitro fertilization (IVF) cycle continues to increase annually by many clinics, particularly because of “add-ons” of dubious value.</p> <p>[[{"fid":"284599","view_mode":"medstat_image_flush_left","fields":{"format":"medstat_image_flush_left","field_file_image_alt_text[und][0][value]":"Dr. Trolice is director of The IVF Center in Winter Park, Fla., and professor of obstetrics and gynecology at the University of Central Florida, Orlando.","field_file_image_credit[und][0][value]":"Fertility CARE","field_file_image_caption[und][0][value]":"Dr. Mark P. Trolice"},"type":"media","attributes":{"class":"media-element file-medstat_image_flush_left"}}]]The initial application of IVF was for tubal factor infertility. Over the decades since 1981, the year of the first successful live birth in the United States, indications for IVF have dramatically expanded – ovulation dysfunction, unexplained infertility, male factor, advanced stage endometriosis, unexplained infertility, embryo testing to avoid an inherited genetic disease from the intended parents carrying the same mutation, and family balancing for gender, along with fertility preservation, including before potentially gonadotoxic treatment and “elective” planned oocyte cryopreservation.<br/><br/><span class="tag metaDescription">The cost of IVF remains a significant, and possibly leading, stumbling block for women, couples, and men who lack insurance coverage.</span> From RESOLVE.org, the National Infertility Association: “As of June 2022, 20 states have passed fertility insurance coverage laws, 14 of those laws include IVF coverage, and 12 states have fertility preservation laws for iatrogenic (medically induced) infertility.” Consequently, “affordable IVF” is paramount to providing equal access for patients.<br/><br/>[[{"fid":"296432","view_mode":"medstat_image_flush_right","fields":{"format":"medstat_image_flush_right","field_file_image_alt_text[und][0][value]":"Dr. Kevin Doody, director of CARE Fertility in Bedford, Tex., and clinical professor at UT Southwestern Medical Center, Dallas","field_file_image_credit[und][0][value]":"CARE Fertility","field_file_image_caption[und][0][value]":"Dr. Kevin Doody"},"type":"media","attributes":{"class":"media-element file-medstat_image_flush_right"}}]]I spoke with the past president of The Society for Assisted Reproductive Technology (SART.org), Kevin Doody, MD, HCLD, to discuss current IVF treatment options for couples that may decrease their financial burden, particularly by applying a novel approach – called INVOcell – that involves using the woman’s vagina as the embryo “incubator.” Dr. Doody is director of CARE Fertility in Bedford, Tex., and clinical professor at UT Southwestern Medical Center, Dallas.<br/><br/></p> <h2>How does limiting the dosage of gonadotropins in IVF cycles, known as “minimal stimulation,” affect pregnancy outcomes?</h2> <p>IVF medications are often costly, so it is logical to try and minimize expenses by using them judiciously. “Minimal stimulation” generally is not the best approach, as having more eggs usually leads to better pregnancy rates. High egg yield increases short-term success and provides additional embryos for future attempts.</p> <p>However, extremely high gonadotropin doses do not necessarily yield more eggs or successful pregnancies. The dose response to gonadotropins follows a sigmoid curve, and typically doses beyond 225-300 IU per day do not offer additional benefits, except for women with an elevated body weight. Yet, some physicians continue to use higher doses in women with low ovarian reserve, which is often not beneficial and can add unnecessary costs.<br/><br/></p> <h2>Is “natural cycle” IVF cost-effective with acceptable pregnancy success rates?</h2> <p>Although the first-ever IVF baby was conceived through a natural cycle, this approach has very low success rates. Even with advancements in IVF laboratory technologies, the outcomes of natural cycle IVF have remained disappointingly low and are generally considered unacceptable.</p> <h2>Are there other cost-saving alternatives for IVF that still maintain reasonable success rates?</h2> <p>Some patients can undergo a more simplified ovarian stimulation protocol that reduces the number of monitoring visits, thus reducing costs. In couples without a severe male factor, the application and additional expense of intracytoplasmic sperm injection (ICSI) is unnecessary. Pre-implantation genetic testing for embryo aneuploidy, another “add-on” procedure, has specific indications and medical evidence does not support its use in all patient cycles. </p> <h2>How can the cost of a standard IVF cycle be reduced, especially in areas without mandated infertility insurance coverage?</h2> <p>Addressing this issue involves considering principles of justice in medical ethics, which emphasize equal health care access for all individuals. Infertility is a medical condition and IVF is expensive, so lack of insurance coverage often restricts access. Our clinic offers a more affordable option called “effortless IVF” using an intravaginal culture system (INVOcell), which minimizes the monitoring process while maintaining satisfactory success rates and reducing the risks associated with ovarian hyperstimulation syndrome.</p> <h2>What is INVOcell, and how successful is it in terms of live birth rates?</h2> <p>INVOcell is an innovative approach to IVF, where an intravaginal culture system is used as an “embryo incubator whereby freshly harvested eggs along with sperm are immediately added to a small chamber device that is placed in the woman’s vagina for up to 5 days to allow for fertilization and embryo development.” The woman, typically, has no discomfort from the device. For appropriately selected patients, the literature has shown live birth rates are comparable to those achieved using conventional laboratory incubation systems. </p> <h2>As an early participant in INVOcell research, can you share insights on the ideal candidates for this procedure and any contraindications?</h2> <p>The INVOcell system is best suited for straightforward cases. It is not recommended for severe male factor infertility requiring ICSI, since this will delay application of the chamber device and increase cost. Further, cases involving preimplantation genetic testing are not recommended because the embryos may not develop synchronously within the device to the embryo stage needed for a biopsy.</p> <h2>What training is required for embryologists and physicians to use INVOcell?</h2> <p>Embryologists require training for a few hours to learn the basics of INVOcell. They must master loading eggs into and retrieving embryos from the device. Practicing on discarded eggs and embryos, embryologists can accelerate the acquisition of the proper technique needed for INVOcell. Physicians find the training easier; they mainly need to learn the correct placement and removal of the device in the vagina.<br/><br/></p> <h2>Is INVOcell gaining acceptance among patients and IVF centers?</h2> <p>Acceptance varies. In our practice, INVOcell has largely replaced superovulation and intrauterine insemination treatments. However, some clinics still need to determine how this tool fits within their practice.</p> <h2>Have IVF success rates plateaued as affordable options increase?</h2> <p>IVF success rates grew substantially in the 1980s and 1990s, fostered by improved embryo culture systems and higher numbers of embryos transferred, the latter at the expense of a multiple gestation. While the rate of improvement has slowed, coinciding with the increasing use of single embryo transfer, advancements in IVF continue toward the goal of improving the singleton live birth rate per IVF cycle. There is still room for enhancement in success rates alongside cost reduction. Continued innovation is needed, especially for patients with challenging underlying biological issues.</p> <h2>Can you provide insight into the next potential breakthrough in IVF that may reduce costs, be less invasive, and maintain optimal pregnancy rates?</h2> <p>I am very excited about recent breakthroughs in in vitro maturation (IVM) of oocytes. The bottleneck in IVF clinics (and significant expense) primarily relates to the need to stimulate the ovaries to get mature and competent eggs. The technology of IVM has existed for decades but has yet to be fully embraced by clinics because of the poor competency of oocytes matured in the laboratory. </p> <p>Immature eggs resume meiosis immediately upon removal from the ovary. Nuclear maturation of eggs in the lab is easy. In fact, it happens too quickly, thereby not allowing for the maturation of the egg cytoplasm. This has previously led to poor development of embryos following fertilization and low success rates.<br/><br/>Recently, a new laboratory strategy has resulted in a significant improvement in success. This improved culture system uses a peptide that prevents the resumption of meiosis for the initial culture time frame. Substances, including follicle stimulating hormone, can be added to the media to promote oocyte cytoplasmic maturation. Following this, the eggs are placed in a media without the meiosis inhibitor to allow for nuclear maturation. This results in a significantly higher proportion of competent mature eggs.<span class="end"/></p> <p> <em><br/><br/>Dr. Trolice is director of The IVF Center in Winter Park, Fla., and professor of obstetrics and gynecology at the University of Central Florida, Orlando.</em> </p> </itemContent> </newsItem> <newsItem> <itemMeta> <itemRole>teaser</itemRole> <itemClass>text</itemClass> <title/> <deck/> </itemMeta> <itemContent> </itemContent> </newsItem> </itemSet></root>
Disallow All Ads
Content Gating
No Gating (article Unlocked/Free)
Alternative CME
Disqus Comments
Default
Use ProPublica
Hide sidebar & use full width
render the right sidebar.
Conference Recap Checkbox
Not Conference Recap
Clinical Edge
Display the Slideshow in this Article
Medscape Article
Display survey writer
Reuters content
Disable Inline Native ads
WebMD Article

Applications of office hysteroscopy for the infertility patient

Article Type
Changed
Fri, 06/02/2023 - 16:02

 

What role does diagnostic office hysteroscopy play in an infertility evaluation?

Performed properly, office hysteroscopy can transform your practice by accurately, gently, and safely assessing the uterine cavity as well as assessing tubal patency.1

More specifically, hysteroscopy is the gold standard for assessing the uterine cavity. The sensitivity, specificity, and positive predictive and negative predictive values of hysterosalpingography (HSG) in evaluating uterine cavity abnormalities were 44.83%; 86.67%; 56.52%; and 80.25%, respectively.2 Given the poor sensitivity of HSG, a diagnosis of endometrial polyps and/or chronic endometritis is more likely to be missed.

Parry_Preston_J_MS_web.jpg
Dr. J. Preston Parry

Our crossover trial comparing HSG to office hysteroscopy for tubal patency showed that women were 110 times more likely to have the maximum level of pain with HSG than diagnostic hysteroscopy when using a 2.8-mm flexible hysteroscope.3 Further, infection rates and vasovagal events were far lower with hysteroscopy.1

Finally, compared with HSG, we showed 98%-100% sensitivity and 84% specificity for tubal occlusion with hysteroscopy by air-infused saline. Conversely, HSG typically is associated with 76%-96% sensitivity and 67%-100% specificity.4 Additionally, we can often perform diagnostic hysteroscopies for approximately $35 per procedure for total fixed and disposable equipment costs.
 

How should physicians perform office hysteroscopy to minimize patient discomfort?

The classic paradigm has been to focus on paracervical blocks, anxiolytics, and a supportive environment (such as mood music). However, those are far more important when your hysteroscope is larger than the natural cervical lumen. If you can use small hysteroscopes (< 3 mm for the nulliparous cervix, < 4 mm for the parous cervix), most women will not require cervical dilation, which further enhances the patient experience.

Trolice_Mark _P_FLA_2022_web.jpg
Dr. Mark P. Trolice

Using a flexible hysteroscope for suspected pathology, making sure not to overdistend the uterus (particularly in high-risk patients such as those with tubal occlusion and cervical stenosis), and vaginoscopy can all minimize patient discomfort. We have published data showing that by using a 2.8-mm flexible diagnostic hysteroscope in a group of mostly nulliparous women, greater than 50% have no discomfort, and more than 90% will have mild to no discomfort.3

What operative hysteroscopy procedures can be performed safely in a physician’s office, and what equipment is required?

Though highly dependent on experience and resources, reproductive endocrinology and infertility specialists (REIs) arguably have the easiest transition to operative office hysteroscopy by utilizing the analgesia and procedure room that is standard for oocyte retrieval and simply adding hysteroscopic procedures. The accompanying table stratifies general hysteroscopic procedures by difficulty.

163585_Office Hysteroscopy_web.jpg
Relative Stages of Difficulty for Operative Office Hysteroscopy

If one can use propofol or a similar level of sedation (which is routinely utilized for oocyte aspiration), there are few hysteroscopies that cannot be accomplished in the office. However, the less sedation and analgesia, the more judicious one must be in patient selection. Moreover, there are trade-offs between visualization, comfort, and instrumentation.

The greater the uterine distention and diameter of the hysteroscope, the more patients experience pain. One-third of patients (especially nulliparous) will discontinue a procedure with a 5-mm hysteroscope because of discomfort.5 However, as one drops to 4.5 mm and smaller operative hysteroscopes, instruments often occupy the inflow channel, limiting distention and visualization, which also can affect completion rates and safety.
 

 

 

When is operative hysteroscopy best suited for the OR?

In addition to physician experience and clinical resources, the critical factors guiding our choices for selecting the OR rather than the office, include:

  • Loss of landmarks. Though Dr. Parry now does most severe intrauterine adhesion cases in the office with ultrasound guidance, when neither ostia can be visualized there is meaningful risk for perforation. Preoperative estrogen, development of planes with the diagnostic hysteroscope prior, and preparing the patient for a possible multistage procedure are all important.
  • Use of energy. There are many excellent hysteroscopic surgeons who use the resectoscope well in the office. However, with possible patient movement and potential perforation with energy leading to a bowel injury, there can be greater risk when using energy relative to other methods (such as forceps, scissors, and mechanical morcellation).
  • Deeper fibroids. Fibroids displace rather than invade the myometrium, and one can sonographically visualize the myometrium reapproximate over a fibroid as it herniates more into the uterine cavity. Nevertheless, the closer a fibroid comes to the serosa, the more mindful one should be of risks and balances for hysteroscopic removal.

In a patient with a severely stenotic cervix or tortuous endocervical canal, what preprocedure methods do you find helpful, and do you utilize abdominal ultrasound guidance?

If using a 2.8-mm flexible diagnostic hysteroscope, we find 99.8%-99.9% of cervices can be successfully cannulated in the office, with rare exception, that is, following cryotherapy or chlamydia cervicitis. This is the equivalent of your dilator having a camera on the tip and fully articulating to adjust to the cervical path.

Transvaginal sonography prior to hysteroscopy where one maps the cervical lumen helps anticipate problems (along with being familiar with the patient’s history). For the rare dilation under anesthesia, concurrent sonography with a 2.8-mm flexible hysteroscope and intermittent dilator use has been sufficient for our exceptions without the need for lacrimal dilators, vasopressin, misoprostol, and other adjuncts. Of note, we use a 1080p flexible endoscope, as lower resolution would make this more challenging.
 

In patients with recurrent implantation failure following IVF, is hysteroscopy superior to 3D saline infusion sonogram?

At an American Society of Reproductive Medicine 2021 session, Ilan Tur-Kaspa, MD, and Dr. Parry debated the topic of 2D ultrasound combined with hysteroscopy vs. 3D saline infusion sonography. Core areas of agreement were that expert hands for any approach are better than nonexpert, and high-resolution technology is better than lower resolution. There was also agreement that extrauterine and myometrial disease, such as intramural fibroids and adenomyosis, are contributory factors.

So, sonography will always have a role. However, existing and forthcoming data show hysteroscopy to improve live birth rates for patients with recurrent implantation failure after IVF. Dr. Parry finds diagnostic hysteroscopy easier for identifying endometritis, sessile and cornual polyps, retained products of conception (which are often isoechogenic with the endometrium) and lateral adhesions.

The reality is that there is variability among physicians and midlevel providers in both sonographic and diagnostic hysteroscopic skill. If one wants to verify findings with another team member, acknowledging that there can be nuances to identifying these pathologies by sonography, it is easier to share and discuss findings through hysteroscopic video than sonographic records.
 

 

 

When is endometrial biopsy indicated during office hysteroscopy?

The patients of an REI are very unlikely to have endometrial cancer (or even hyperplasia) outside of polyps (or arguably hypervascular areas of overgrowth), so the focus is on resecting visualized pathology relative to random biopsy.

However, the threshold for biopsy should be adjusted to the patient population, as well as to individual findings and risk. RVUs are greatly increased (11.1 > 41.57) with biopsy, helping sustainability. Additionally, if one places the hysteroscope on endometrium and applies suction through the inflow channel, one can obtain a sample with small-caliber diagnostic hysteroscopes and without having to use forceps.
 

What is your threshold for fluid deficit in hysteroscopy?

We follow AAGL guidelines, which for operative hysteroscopy are 2,500 mL of isotonic fluids or 1,000 mL of hypotonic fluids in low-risk patients. This should be further reduced to 500 mL of isotonic fluids in the elderly and even 300 mL in those with cardiovascular compromise.6

For patients who request sedation for office hysteroscopy, which option do you recommend – paracervical block alone, nitrous oxide, or the combination?

For diagnostic, greater than 95% of our patients do not require even over-the-counter analgesic medications. For operative, we consider all permissible resources that allow for a safe combination that is appropriate to the pathology and clinical setting, such as paracervical blocks, nitrous oxide, NSAIDs such as ketorolac, anxiolytics, and more.

The goal is to optimize the patient experience. However, the top three criteria that influence successful operative office hysteroscopy for a conscious patient are a parous cervix, judicious patient selection, and pre- and intraoperative verbal analgesia. Informed consent and engagement improve the experience of both the patient and physician.

Dr. Parry is the founder of Positive Steps Fertility in Madison, Miss. Dr. Trolice is director of The IVF Center in Winter Park, Fla., and professor of obstetrics and gynecology at the University of Central Florida, Orlando.

References

1. Parry JP et al. J Minim Invasive Gynecol. 2017 May-Jun. doi: 10.1016/j.jmig.2017.02.010.

2. Wadhwa L et al. 2017 Apr-Jun. doi: 10.4103/jhrs.JHRS_123_16.

3. Parry JP et al. Fertil Steril. 2017 Oct. doi: 10.1016/j.fertnstert.2017.07.1159.

4. Penzias A et al. Fertil Steril. 2021 Nov. doi: 10.1016/j.fertnstert.2021.08.038.

5. Campo R et al. Hum Reprod. 2005 Jan;20(1):258-63. doi: 10.1093/humrep/deh559.

6. AAGL AAGL practice report: Practice guidelines for the management of hysteroscopic distending media. J Minim Invasive Gynecol. 2013 Mar-Apr. doi: 10.1016/j.jmig.2012.12.002.

Publications
Topics
Sections

 

What role does diagnostic office hysteroscopy play in an infertility evaluation?

Performed properly, office hysteroscopy can transform your practice by accurately, gently, and safely assessing the uterine cavity as well as assessing tubal patency.1

More specifically, hysteroscopy is the gold standard for assessing the uterine cavity. The sensitivity, specificity, and positive predictive and negative predictive values of hysterosalpingography (HSG) in evaluating uterine cavity abnormalities were 44.83%; 86.67%; 56.52%; and 80.25%, respectively.2 Given the poor sensitivity of HSG, a diagnosis of endometrial polyps and/or chronic endometritis is more likely to be missed.

Parry_Preston_J_MS_web.jpg
Dr. J. Preston Parry

Our crossover trial comparing HSG to office hysteroscopy for tubal patency showed that women were 110 times more likely to have the maximum level of pain with HSG than diagnostic hysteroscopy when using a 2.8-mm flexible hysteroscope.3 Further, infection rates and vasovagal events were far lower with hysteroscopy.1

Finally, compared with HSG, we showed 98%-100% sensitivity and 84% specificity for tubal occlusion with hysteroscopy by air-infused saline. Conversely, HSG typically is associated with 76%-96% sensitivity and 67%-100% specificity.4 Additionally, we can often perform diagnostic hysteroscopies for approximately $35 per procedure for total fixed and disposable equipment costs.
 

How should physicians perform office hysteroscopy to minimize patient discomfort?

The classic paradigm has been to focus on paracervical blocks, anxiolytics, and a supportive environment (such as mood music). However, those are far more important when your hysteroscope is larger than the natural cervical lumen. If you can use small hysteroscopes (< 3 mm for the nulliparous cervix, < 4 mm for the parous cervix), most women will not require cervical dilation, which further enhances the patient experience.

Trolice_Mark _P_FLA_2022_web.jpg
Dr. Mark P. Trolice

Using a flexible hysteroscope for suspected pathology, making sure not to overdistend the uterus (particularly in high-risk patients such as those with tubal occlusion and cervical stenosis), and vaginoscopy can all minimize patient discomfort. We have published data showing that by using a 2.8-mm flexible diagnostic hysteroscope in a group of mostly nulliparous women, greater than 50% have no discomfort, and more than 90% will have mild to no discomfort.3

What operative hysteroscopy procedures can be performed safely in a physician’s office, and what equipment is required?

Though highly dependent on experience and resources, reproductive endocrinology and infertility specialists (REIs) arguably have the easiest transition to operative office hysteroscopy by utilizing the analgesia and procedure room that is standard for oocyte retrieval and simply adding hysteroscopic procedures. The accompanying table stratifies general hysteroscopic procedures by difficulty.

163585_Office Hysteroscopy_web.jpg
Relative Stages of Difficulty for Operative Office Hysteroscopy

If one can use propofol or a similar level of sedation (which is routinely utilized for oocyte aspiration), there are few hysteroscopies that cannot be accomplished in the office. However, the less sedation and analgesia, the more judicious one must be in patient selection. Moreover, there are trade-offs between visualization, comfort, and instrumentation.

The greater the uterine distention and diameter of the hysteroscope, the more patients experience pain. One-third of patients (especially nulliparous) will discontinue a procedure with a 5-mm hysteroscope because of discomfort.5 However, as one drops to 4.5 mm and smaller operative hysteroscopes, instruments often occupy the inflow channel, limiting distention and visualization, which also can affect completion rates and safety.
 

 

 

When is operative hysteroscopy best suited for the OR?

In addition to physician experience and clinical resources, the critical factors guiding our choices for selecting the OR rather than the office, include:

  • Loss of landmarks. Though Dr. Parry now does most severe intrauterine adhesion cases in the office with ultrasound guidance, when neither ostia can be visualized there is meaningful risk for perforation. Preoperative estrogen, development of planes with the diagnostic hysteroscope prior, and preparing the patient for a possible multistage procedure are all important.
  • Use of energy. There are many excellent hysteroscopic surgeons who use the resectoscope well in the office. However, with possible patient movement and potential perforation with energy leading to a bowel injury, there can be greater risk when using energy relative to other methods (such as forceps, scissors, and mechanical morcellation).
  • Deeper fibroids. Fibroids displace rather than invade the myometrium, and one can sonographically visualize the myometrium reapproximate over a fibroid as it herniates more into the uterine cavity. Nevertheless, the closer a fibroid comes to the serosa, the more mindful one should be of risks and balances for hysteroscopic removal.

In a patient with a severely stenotic cervix or tortuous endocervical canal, what preprocedure methods do you find helpful, and do you utilize abdominal ultrasound guidance?

If using a 2.8-mm flexible diagnostic hysteroscope, we find 99.8%-99.9% of cervices can be successfully cannulated in the office, with rare exception, that is, following cryotherapy or chlamydia cervicitis. This is the equivalent of your dilator having a camera on the tip and fully articulating to adjust to the cervical path.

Transvaginal sonography prior to hysteroscopy where one maps the cervical lumen helps anticipate problems (along with being familiar with the patient’s history). For the rare dilation under anesthesia, concurrent sonography with a 2.8-mm flexible hysteroscope and intermittent dilator use has been sufficient for our exceptions without the need for lacrimal dilators, vasopressin, misoprostol, and other adjuncts. Of note, we use a 1080p flexible endoscope, as lower resolution would make this more challenging.
 

In patients with recurrent implantation failure following IVF, is hysteroscopy superior to 3D saline infusion sonogram?

At an American Society of Reproductive Medicine 2021 session, Ilan Tur-Kaspa, MD, and Dr. Parry debated the topic of 2D ultrasound combined with hysteroscopy vs. 3D saline infusion sonography. Core areas of agreement were that expert hands for any approach are better than nonexpert, and high-resolution technology is better than lower resolution. There was also agreement that extrauterine and myometrial disease, such as intramural fibroids and adenomyosis, are contributory factors.

So, sonography will always have a role. However, existing and forthcoming data show hysteroscopy to improve live birth rates for patients with recurrent implantation failure after IVF. Dr. Parry finds diagnostic hysteroscopy easier for identifying endometritis, sessile and cornual polyps, retained products of conception (which are often isoechogenic with the endometrium) and lateral adhesions.

The reality is that there is variability among physicians and midlevel providers in both sonographic and diagnostic hysteroscopic skill. If one wants to verify findings with another team member, acknowledging that there can be nuances to identifying these pathologies by sonography, it is easier to share and discuss findings through hysteroscopic video than sonographic records.
 

 

 

When is endometrial biopsy indicated during office hysteroscopy?

The patients of an REI are very unlikely to have endometrial cancer (or even hyperplasia) outside of polyps (or arguably hypervascular areas of overgrowth), so the focus is on resecting visualized pathology relative to random biopsy.

However, the threshold for biopsy should be adjusted to the patient population, as well as to individual findings and risk. RVUs are greatly increased (11.1 > 41.57) with biopsy, helping sustainability. Additionally, if one places the hysteroscope on endometrium and applies suction through the inflow channel, one can obtain a sample with small-caliber diagnostic hysteroscopes and without having to use forceps.
 

What is your threshold for fluid deficit in hysteroscopy?

We follow AAGL guidelines, which for operative hysteroscopy are 2,500 mL of isotonic fluids or 1,000 mL of hypotonic fluids in low-risk patients. This should be further reduced to 500 mL of isotonic fluids in the elderly and even 300 mL in those with cardiovascular compromise.6

For patients who request sedation for office hysteroscopy, which option do you recommend – paracervical block alone, nitrous oxide, or the combination?

For diagnostic, greater than 95% of our patients do not require even over-the-counter analgesic medications. For operative, we consider all permissible resources that allow for a safe combination that is appropriate to the pathology and clinical setting, such as paracervical blocks, nitrous oxide, NSAIDs such as ketorolac, anxiolytics, and more.

The goal is to optimize the patient experience. However, the top three criteria that influence successful operative office hysteroscopy for a conscious patient are a parous cervix, judicious patient selection, and pre- and intraoperative verbal analgesia. Informed consent and engagement improve the experience of both the patient and physician.

Dr. Parry is the founder of Positive Steps Fertility in Madison, Miss. Dr. Trolice is director of The IVF Center in Winter Park, Fla., and professor of obstetrics and gynecology at the University of Central Florida, Orlando.

References

1. Parry JP et al. J Minim Invasive Gynecol. 2017 May-Jun. doi: 10.1016/j.jmig.2017.02.010.

2. Wadhwa L et al. 2017 Apr-Jun. doi: 10.4103/jhrs.JHRS_123_16.

3. Parry JP et al. Fertil Steril. 2017 Oct. doi: 10.1016/j.fertnstert.2017.07.1159.

4. Penzias A et al. Fertil Steril. 2021 Nov. doi: 10.1016/j.fertnstert.2021.08.038.

5. Campo R et al. Hum Reprod. 2005 Jan;20(1):258-63. doi: 10.1093/humrep/deh559.

6. AAGL AAGL practice report: Practice guidelines for the management of hysteroscopic distending media. J Minim Invasive Gynecol. 2013 Mar-Apr. doi: 10.1016/j.jmig.2012.12.002.

 

What role does diagnostic office hysteroscopy play in an infertility evaluation?

Performed properly, office hysteroscopy can transform your practice by accurately, gently, and safely assessing the uterine cavity as well as assessing tubal patency.1

More specifically, hysteroscopy is the gold standard for assessing the uterine cavity. The sensitivity, specificity, and positive predictive and negative predictive values of hysterosalpingography (HSG) in evaluating uterine cavity abnormalities were 44.83%; 86.67%; 56.52%; and 80.25%, respectively.2 Given the poor sensitivity of HSG, a diagnosis of endometrial polyps and/or chronic endometritis is more likely to be missed.

Parry_Preston_J_MS_web.jpg
Dr. J. Preston Parry

Our crossover trial comparing HSG to office hysteroscopy for tubal patency showed that women were 110 times more likely to have the maximum level of pain with HSG than diagnostic hysteroscopy when using a 2.8-mm flexible hysteroscope.3 Further, infection rates and vasovagal events were far lower with hysteroscopy.1

Finally, compared with HSG, we showed 98%-100% sensitivity and 84% specificity for tubal occlusion with hysteroscopy by air-infused saline. Conversely, HSG typically is associated with 76%-96% sensitivity and 67%-100% specificity.4 Additionally, we can often perform diagnostic hysteroscopies for approximately $35 per procedure for total fixed and disposable equipment costs.
 

How should physicians perform office hysteroscopy to minimize patient discomfort?

The classic paradigm has been to focus on paracervical blocks, anxiolytics, and a supportive environment (such as mood music). However, those are far more important when your hysteroscope is larger than the natural cervical lumen. If you can use small hysteroscopes (< 3 mm for the nulliparous cervix, < 4 mm for the parous cervix), most women will not require cervical dilation, which further enhances the patient experience.

Trolice_Mark _P_FLA_2022_web.jpg
Dr. Mark P. Trolice

Using a flexible hysteroscope for suspected pathology, making sure not to overdistend the uterus (particularly in high-risk patients such as those with tubal occlusion and cervical stenosis), and vaginoscopy can all minimize patient discomfort. We have published data showing that by using a 2.8-mm flexible diagnostic hysteroscope in a group of mostly nulliparous women, greater than 50% have no discomfort, and more than 90% will have mild to no discomfort.3

What operative hysteroscopy procedures can be performed safely in a physician’s office, and what equipment is required?

Though highly dependent on experience and resources, reproductive endocrinology and infertility specialists (REIs) arguably have the easiest transition to operative office hysteroscopy by utilizing the analgesia and procedure room that is standard for oocyte retrieval and simply adding hysteroscopic procedures. The accompanying table stratifies general hysteroscopic procedures by difficulty.

163585_Office Hysteroscopy_web.jpg
Relative Stages of Difficulty for Operative Office Hysteroscopy

If one can use propofol or a similar level of sedation (which is routinely utilized for oocyte aspiration), there are few hysteroscopies that cannot be accomplished in the office. However, the less sedation and analgesia, the more judicious one must be in patient selection. Moreover, there are trade-offs between visualization, comfort, and instrumentation.

The greater the uterine distention and diameter of the hysteroscope, the more patients experience pain. One-third of patients (especially nulliparous) will discontinue a procedure with a 5-mm hysteroscope because of discomfort.5 However, as one drops to 4.5 mm and smaller operative hysteroscopes, instruments often occupy the inflow channel, limiting distention and visualization, which also can affect completion rates and safety.
 

 

 

When is operative hysteroscopy best suited for the OR?

In addition to physician experience and clinical resources, the critical factors guiding our choices for selecting the OR rather than the office, include:

  • Loss of landmarks. Though Dr. Parry now does most severe intrauterine adhesion cases in the office with ultrasound guidance, when neither ostia can be visualized there is meaningful risk for perforation. Preoperative estrogen, development of planes with the diagnostic hysteroscope prior, and preparing the patient for a possible multistage procedure are all important.
  • Use of energy. There are many excellent hysteroscopic surgeons who use the resectoscope well in the office. However, with possible patient movement and potential perforation with energy leading to a bowel injury, there can be greater risk when using energy relative to other methods (such as forceps, scissors, and mechanical morcellation).
  • Deeper fibroids. Fibroids displace rather than invade the myometrium, and one can sonographically visualize the myometrium reapproximate over a fibroid as it herniates more into the uterine cavity. Nevertheless, the closer a fibroid comes to the serosa, the more mindful one should be of risks and balances for hysteroscopic removal.

In a patient with a severely stenotic cervix or tortuous endocervical canal, what preprocedure methods do you find helpful, and do you utilize abdominal ultrasound guidance?

If using a 2.8-mm flexible diagnostic hysteroscope, we find 99.8%-99.9% of cervices can be successfully cannulated in the office, with rare exception, that is, following cryotherapy or chlamydia cervicitis. This is the equivalent of your dilator having a camera on the tip and fully articulating to adjust to the cervical path.

Transvaginal sonography prior to hysteroscopy where one maps the cervical lumen helps anticipate problems (along with being familiar with the patient’s history). For the rare dilation under anesthesia, concurrent sonography with a 2.8-mm flexible hysteroscope and intermittent dilator use has been sufficient for our exceptions without the need for lacrimal dilators, vasopressin, misoprostol, and other adjuncts. Of note, we use a 1080p flexible endoscope, as lower resolution would make this more challenging.
 

In patients with recurrent implantation failure following IVF, is hysteroscopy superior to 3D saline infusion sonogram?

At an American Society of Reproductive Medicine 2021 session, Ilan Tur-Kaspa, MD, and Dr. Parry debated the topic of 2D ultrasound combined with hysteroscopy vs. 3D saline infusion sonography. Core areas of agreement were that expert hands for any approach are better than nonexpert, and high-resolution technology is better than lower resolution. There was also agreement that extrauterine and myometrial disease, such as intramural fibroids and adenomyosis, are contributory factors.

So, sonography will always have a role. However, existing and forthcoming data show hysteroscopy to improve live birth rates for patients with recurrent implantation failure after IVF. Dr. Parry finds diagnostic hysteroscopy easier for identifying endometritis, sessile and cornual polyps, retained products of conception (which are often isoechogenic with the endometrium) and lateral adhesions.

The reality is that there is variability among physicians and midlevel providers in both sonographic and diagnostic hysteroscopic skill. If one wants to verify findings with another team member, acknowledging that there can be nuances to identifying these pathologies by sonography, it is easier to share and discuss findings through hysteroscopic video than sonographic records.
 

 

 

When is endometrial biopsy indicated during office hysteroscopy?

The patients of an REI are very unlikely to have endometrial cancer (or even hyperplasia) outside of polyps (or arguably hypervascular areas of overgrowth), so the focus is on resecting visualized pathology relative to random biopsy.

However, the threshold for biopsy should be adjusted to the patient population, as well as to individual findings and risk. RVUs are greatly increased (11.1 > 41.57) with biopsy, helping sustainability. Additionally, if one places the hysteroscope on endometrium and applies suction through the inflow channel, one can obtain a sample with small-caliber diagnostic hysteroscopes and without having to use forceps.
 

What is your threshold for fluid deficit in hysteroscopy?

We follow AAGL guidelines, which for operative hysteroscopy are 2,500 mL of isotonic fluids or 1,000 mL of hypotonic fluids in low-risk patients. This should be further reduced to 500 mL of isotonic fluids in the elderly and even 300 mL in those with cardiovascular compromise.6

For patients who request sedation for office hysteroscopy, which option do you recommend – paracervical block alone, nitrous oxide, or the combination?

For diagnostic, greater than 95% of our patients do not require even over-the-counter analgesic medications. For operative, we consider all permissible resources that allow for a safe combination that is appropriate to the pathology and clinical setting, such as paracervical blocks, nitrous oxide, NSAIDs such as ketorolac, anxiolytics, and more.

The goal is to optimize the patient experience. However, the top three criteria that influence successful operative office hysteroscopy for a conscious patient are a parous cervix, judicious patient selection, and pre- and intraoperative verbal analgesia. Informed consent and engagement improve the experience of both the patient and physician.

Dr. Parry is the founder of Positive Steps Fertility in Madison, Miss. Dr. Trolice is director of The IVF Center in Winter Park, Fla., and professor of obstetrics and gynecology at the University of Central Florida, Orlando.

References

1. Parry JP et al. J Minim Invasive Gynecol. 2017 May-Jun. doi: 10.1016/j.jmig.2017.02.010.

2. Wadhwa L et al. 2017 Apr-Jun. doi: 10.4103/jhrs.JHRS_123_16.

3. Parry JP et al. Fertil Steril. 2017 Oct. doi: 10.1016/j.fertnstert.2017.07.1159.

4. Penzias A et al. Fertil Steril. 2021 Nov. doi: 10.1016/j.fertnstert.2021.08.038.

5. Campo R et al. Hum Reprod. 2005 Jan;20(1):258-63. doi: 10.1093/humrep/deh559.

6. AAGL AAGL practice report: Practice guidelines for the management of hysteroscopic distending media. J Minim Invasive Gynecol. 2013 Mar-Apr. doi: 10.1016/j.jmig.2012.12.002.

Publications
Publications
Topics
Article Type
Sections
Teambase XML
<?xml version="1.0" encoding="UTF-8"?>
<!--$RCSfile: InCopy_agile.xsl,v $ $Revision: 1.35 $-->
<!--$RCSfile: drupal.xsl,v $ $Revision: 1.7 $-->
<root generator="drupal.xsl" gversion="1.7"> <header> <fileName>163585</fileName> <TBEID>0C04A4AC.SIG</TBEID> <TBUniqueIdentifier>MD_0C04A4AC</TBUniqueIdentifier> <newsOrJournal>News</newsOrJournal> <publisherName>Frontline Medical Communications</publisherName> <storyname/> <articleType>353</articleType> <TBLocation>QC Done-All Pubs</TBLocation> <QCDate>20230601T175552</QCDate> <firstPublished>20230602T090207</firstPublished> <LastPublished>20230602T090207</LastPublished> <pubStatus qcode="stat:"/> <embargoDate/> <killDate/> <CMSDate>20230602T090207</CMSDate> <articleSource/> <facebookInfo/> <meetingNumber/> <byline>Other Writer</byline> <bylineText>J. PRESTON PARRY MD, MPH; MARK TROLICE MD, MBA</bylineText> <bylineFull>J. PRESTON PARRY MD, MPH; MARK TROLICE MD, MBA</bylineFull> <bylineTitleText/> <USOrGlobal/> <wireDocType/> <newsDocType>Column</newsDocType> <journalDocType/> <linkLabel/> <pageRange/> <citation/> <quizID/> <indexIssueDate/> <itemClass qcode="ninat:text"/> <provider qcode="provider:imng"> <name>IMNG Medical Media</name> <rightsInfo> <copyrightHolder> <name>Frontline Medical News</name> </copyrightHolder> <copyrightNotice>Copyright (c) 2015 Frontline Medical News, a Frontline Medical Communications Inc. company. All rights reserved. This material may not be published, broadcast, copied, or otherwise reproduced or distributed without the prior written permission of Frontline Medical Communications Inc.</copyrightNotice> </rightsInfo> </provider> <abstract/> <metaDescription>Performed properly, office hysteroscopy can transform your practice by accurately, gently, and safely assessing the uterine cavity as well as assessing tubal pa</metaDescription> <articlePDF/> <teaserImage>295450</teaserImage> <teaser>Hysteroscopy is the gold standard for assessing the uterine cavity. </teaser> <title>Applications of office hysteroscopy for the infertility patient</title> <deck/> <disclaimer/> <AuthorList/> <articleURL/> <doi/> <pubMedID/> <publishXMLStatus/> <publishXMLVersion>1</publishXMLVersion> <useEISSN>0</useEISSN> <urgency/> <pubPubdateYear/> <pubPubdateMonth/> <pubPubdateDay/> <pubVolume/> <pubNumber/> <wireChannels/> <primaryCMSID/> <CMSIDs/> <keywords/> <seeAlsos/> <publications_g> <publicationData> <publicationCode>ob</publicationCode> <pubIssueName/> <pubArticleType/> <pubTopics/> <pubCategories/> <pubSections/> </publicationData> </publications_g> <publications> <term canonical="true">23</term> </publications> <sections> <term canonical="true">68807</term> <term>52</term> </sections> <topics> <term canonical="true">287</term> </topics> <links> <link> <itemClass qcode="ninat:picture"/> <altRep contenttype="image/jpeg">images/24011e3a.jpg</altRep> <description role="drol:caption">Dr. J. Preston Parry</description> <description role="drol:credit"/> </link> <link> <itemClass qcode="ninat:picture"/> <altRep contenttype="image/jpeg">images/24010881.jpg</altRep> <description role="drol:caption">Dr. Mark P. Trolice</description> <description role="drol:credit"/> </link> <link> <itemClass qcode="ninat:picture"/> <altRep contenttype="image/jpeg">images/24011e3b.jpg</altRep> <description role="drol:caption">Relative Stages of Difficulty for Operative Office Hysteroscopy</description> <description role="drol:credit">Dr. Parry and Dr. Trolice</description> </link> </links> </header> <itemSet> <newsItem> <itemMeta> <itemRole>Main</itemRole> <itemClass>text</itemClass> <title>Applications of office hysteroscopy for the infertility patient</title> <deck/> </itemMeta> <itemContent> <h2>What role does diagnostic office hysteroscopy play in an infertility evaluation?</h2> <p><span class="tag metaDescription">Performed properly, office hysteroscopy can transform your practice by accurately, gently, and safely assessing the uterine cavity as well as assessing tubal patency</span>.<sup>1</sup></p> <p>More specifically, hysteroscopy is the gold standard for assessing the uterine cavity. The sensitivity, specificity, and positive predictive and negative predictive values of hysterosalpingography (HSG) in evaluating uterine cavity abnormalities were 44.83%; 86.67%; 56.52%; and 80.25%, respectively.<sup>2</sup> Given the poor sensitivity of HSG, a diagnosis of endometrial polyps and/or chronic endometritis is more likely to be missed.<br/><br/>[[{"fid":"295450","view_mode":"medstat_image_flush_right","fields":{"format":"medstat_image_flush_right","field_file_image_alt_text[und][0][value]":"founder of Positive Steps Fertility in Madison, Miss.","field_file_image_credit[und][0][value]":"","field_file_image_caption[und][0][value]":"Dr. J. Preston Parry"},"type":"media","attributes":{"class":"media-element file-medstat_image_flush_right"}}]]Our crossover trial comparing HSG to office hysteroscopy for tubal patency showed that women were 110 times more likely to have the maximum level of pain with HSG than diagnostic hysteroscopy when using a 2.8-mm flexible hysteroscope.<sup>3</sup> Further, infection rates and vasovagal events were far lower with hysteroscopy.<sup>1</sup><br/><br/>Finally, compared with HSG, we showed 98%-100% sensitivity and 84% specificity for tubal occlusion with hysteroscopy by air-infused saline. Conversely, HSG typically is associated with 76%-96% sensitivity and 67%-100% specificity.<sup>4</sup> Additionally, we can often perform diagnostic hysteroscopies for approximately $35 per procedure for total fixed and disposable equipment costs.<br/><br/></p> <h2>How should physicians perform office hysteroscopy to minimize patient discomfort?</h2> <p>The classic paradigm has been to focus on paracervical blocks, anxiolytics, and a supportive environment (such as mood music). However, those are far more important when your hysteroscope is larger than the natural cervical lumen. If you can use small hysteroscopes (&lt; 3 mm for the nulliparous cervix, &lt; 4 mm for the parous cervix), most women will not require cervical dilation, which further enhances the patient experience.</p> <p>[[{"fid":"284599","view_mode":"medstat_image_flush_left","fields":{"format":"medstat_image_flush_left","field_file_image_alt_text[und][0][value]":"Dr. Trolice is director of The IVF Center in Winter Park, Fla., and professor of obstetrics and gynecology at the University of Central Florida, Orlando.","field_file_image_credit[und][0][value]":"","field_file_image_caption[und][0][value]":"Dr. Mark P. Trolice"},"type":"media","attributes":{"class":"media-element file-medstat_image_flush_left"}}]]Using a flexible hysteroscope for suspected pathology, making sure not to overdistend the uterus (particularly in high-risk patients such as those with tubal occlusion and cervical stenosis), and vaginoscopy can all minimize patient discomfort. We have published data showing that by using a 2.8-mm flexible diagnostic hysteroscope in a group of mostly nulliparous women, greater than 50% have no discomfort, and more than 90% will have mild to no discomfort.<sup>3</sup></p> <h2>What operative hysteroscopy procedures can be performed safely in a physician’s office, and what equipment is required?</h2> <p>Though highly dependent on experience and resources, reproductive endocrinology and infertility specialists (REIs) arguably have the easiest transition to operative office hysteroscopy by utilizing the analgesia and procedure room that is standard for oocyte retrieval and simply adding hysteroscopic procedures. The accompanying table stratifies general hysteroscopic procedures by difficulty. </p> <p>[[{"fid":"295451","view_mode":"medstat_image_full_text","fields":{"format":"medstat_image_full_text","field_file_image_alt_text[und][0][value]":"","field_file_image_credit[und][0][value]":"Dr. Parry and Dr. Trolice","field_file_image_caption[und][0][value]":"Relative Stages of Difficulty for Operative Office Hysteroscopy"},"type":"media","attributes":{"class":"media-element file-medstat_image_full_text"}}]]If one can use propofol or a similar level of sedation (which is routinely utilized for oocyte aspiration), there are few hysteroscopies that cannot be accomplished in the office. However, the less sedation and analgesia, the more judicious one must be in patient selection. Moreover, there are trade-offs between visualization, comfort, and instrumentation.<br/><br/>The greater the uterine distention and diameter of the hysteroscope, the more patients experience pain. One-third of patients (especially nulliparous) will discontinue a procedure with a 5-mm hysteroscope because of discomfort.<sup>5</sup> However, as one drops to 4.5 mm and smaller operative hysteroscopes, instruments often occupy the inflow channel, limiting distention and visualization, which also can affect completion rates and safety.<br/><br/></p> <h2>When is operative hysteroscopy best suited for the OR?</h2> <p>In addition to physician experience and clinical resources, the critical factors guiding our choices for selecting the OR rather than the office, include: </p> <ul class="body"> <li><strong>Loss of landmarks.</strong> Though Dr. Parry now does most severe intrauterine adhesion cases in the office with ultrasound guidance, when neither ostia can be visualized there is meaningful risk for perforation. Preoperative estrogen, development of planes with the diagnostic hysteroscope prior, and preparing the patient for a possible multistage procedure are all important.</li> <li><strong>Use of energy.</strong> There are many excellent hysteroscopic surgeons who use the resectoscope well in the office. However, with possible patient movement and potential perforation with energy leading to a bowel injury, there can be greater risk when using energy relative to other methods (such as forceps, scissors, and mechanical morcellation).</li> <li><strong>Deeper fibroids.</strong> Fibroids displace rather than invade the myometrium, and one can sonographically visualize the myometrium reapproximate over a fibroid as it herniates more into the uterine cavity. Nevertheless, the closer a fibroid comes to the serosa, the more mindful one should be of risks and balances for hysteroscopic removal.</li> </ul> <h2>In a patient with a severely stenotic cervix or tortuous endocervical canal, what preprocedure methods do you find helpful, and do you utilize abdominal ultrasound guidance?</h2> <p>If using a 2.8-mm flexible diagnostic hysteroscope, we find 99.8%-99.9% of cervices can be successfully cannulated in the office, with rare exception, that is, following cryotherapy or chlamydia cervicitis. This is the equivalent of your dilator having a camera on the tip and fully articulating to adjust to the cervical path.</p> <p>Transvaginal sonography prior to hysteroscopy where one maps the cervical lumen helps anticipate problems (along with being familiar with the patient’s history). For the rare dilation under anesthesia, concurrent sonography with a 2.8-mm flexible hysteroscope and intermittent dilator use has been sufficient for our exceptions without the need for lacrimal dilators, vasopressin, misoprostol, and other adjuncts. Of note, we use a 1080p flexible endoscope, as lower resolution would make this more challenging.<br/><br/></p> <h2>In patients with recurrent implantation failure following IVF, is hysteroscopy superior to 3D saline infusion sonogram?</h2> <p>At an American Society of Reproductive Medicine 2021 session, Ilan Tur-Kaspa, MD, and Dr. Parry debated the topic of 2D ultrasound combined with hysteroscopy vs. 3D saline infusion sonography. Core areas of agreement were that expert hands for any approach are better than nonexpert, and high-resolution technology is better than lower resolution. There was also agreement that extrauterine and myometrial disease, such as intramural fibroids and adenomyosis, are contributory factors.<br/><br/>So, sonography will always have a role. However, existing and forthcoming data show hysteroscopy to improve live birth rates for patients with recurrent implantation failure after IVF. Dr. Parry finds diagnostic hysteroscopy easier for identifying endometritis, sessile and cornual polyps, retained products of conception (which are often isoechogenic with the endometrium) and lateral adhesions.<br/><br/>The reality is that there is variability among physicians and midlevel providers in both sonographic and diagnostic hysteroscopic skill. If one wants to verify findings with another team member, acknowledging that there can be nuances to identifying these pathologies by sonography, it is easier to share and discuss findings through hysteroscopic video than sonographic records.<br/><br/></p> <h2>When is endometrial biopsy indicated during office hysteroscopy?</h2> <p>The patients of an REI are very unlikely to have endometrial cancer (or even hyperplasia) outside of polyps (or arguably hypervascular areas of overgrowth), so the focus is on resecting visualized pathology relative to random biopsy.</p> <p>However, the threshold for biopsy should be adjusted to the patient population, as well as to individual findings and risk. RVUs are greatly increased (11.1 &gt; 41.57) with biopsy, helping sustainability. Additionally, if one places the hysteroscope on endometrium and applies suction through the inflow channel, one can obtain a sample with small-caliber diagnostic hysteroscopes and without having to use forceps.<br/><br/></p> <h2>What is your threshold for fluid deficit in hysteroscopy?</h2> <p>We follow AAGL guidelines, which for operative hysteroscopy are 2,500 mL of isotonic fluids or 1,000 mL of hypotonic fluids in low-risk patients. This should be further reduced to 500 mL of isotonic fluids in the elderly and even 300 mL in those with cardiovascular compromise.<sup>6</sup></p> <h2>For patients who request sedation for office hysteroscopy, which option do you recommend – paracervical block alone, nitrous oxide, or the combination?</h2> <p>For diagnostic, greater than 95% of our patients do not require even over-the-counter analgesic medications. For operative, we consider all permissible resources that allow for a safe combination that is appropriate to the pathology and clinical setting, such as paracervical blocks, nitrous oxide, NSAIDs such as ketorolac, anxiolytics, and more.</p> <p>The goal is to optimize the patient experience. However, the top three criteria that influence successful operative office hysteroscopy for a conscious patient are a parous cervix, judicious patient selection, and pre- and intraoperative verbal analgesia. Informed consent and engagement improve the experience of both the patient and physician.<span class="end"/></p> <p> <em>Dr. Parry is the founder of Positive Steps Fertility in Madison, Miss. Dr. Trolice is director of The IVF Center in Winter Park, Fla., and professor of obstetrics and gynecology at the University of Central Florida, Orlando.</em> </p> <h2>References</h2> <p>1. Parry JP et al. J Minim Invasive Gynecol. 2017 May-Jun. doi: <span class="Hyperlink"><a href="https://www.jmig.org/article/S1553-4650(17)30138-3/fulltext">10.1016/j.jmig.2017.02.010</a></span>.<br/><br/>2. Wadhwa L et al. 2017 Apr-Jun. doi: <span class="Hyperlink"><a href="https://www.ncbi.nlm.nih.gov/pmc/articles/PMC5586093/">10.4103/jhrs.JHRS_123_16</a></span>.<br/><br/>3. Parry JP et al. Fertil Steril. 2017 Oct. doi: <span class="Hyperlink"><a href="https://www.fertstert.org/article/S0015-0282(17)31685-0/fulltext">10.1016/j.fertnstert.2017.07.1159</a></span>.<br/><br/>4. Penzias A et al. Fertil Steril. 2021 Nov. doi: <span class="Hyperlink"><a href="https://www.fertstert.org/article/S0015-0282(21)01984-1/fulltext">10.1016/j.fertnstert.2021.08.038</a></span>.<br/><br/>5. Campo R et al. Hum Reprod. 2005 Jan;20(1):258-63. doi: <span class="Hyperlink"><a href="https://academic.oup.com/humrep/article/20/1/258/671551">10.1093/humrep/deh559</a></span>.<br/><br/>6. AAGL AAGL practice report: Practice guidelines for the management of hysteroscopic distending media. J Minim Invasive Gynecol. 2013 Mar-Apr. doi: <span class="Hyperlink"><a href="https://www.jmig.org/article/S1553-4650(12)01327-1/fulltext">10.1016/j.jmig.2012.12.002</a></span>.<span class="end"/> </p> </itemContent> </newsItem> <newsItem> <itemMeta> <itemRole>teaser</itemRole> <itemClass>text</itemClass> <title/> <deck/> </itemMeta> <itemContent> </itemContent> </newsItem> </itemSet></root>
Disallow All Ads
Content Gating
No Gating (article Unlocked/Free)
Alternative CME
Disqus Comments
Default
Use ProPublica
Hide sidebar & use full width
render the right sidebar.
Conference Recap Checkbox
Not Conference Recap
Clinical Edge
Display the Slideshow in this Article
Medscape Article
Display survey writer
Reuters content
Disable Inline Native ads
WebMD Article

Understanding clinic-reported IVF success rates

Article Type
Changed
Wed, 05/03/2023 - 16:13

The field of assisted reproductive technologies (ART) continues to evolve from its first successful birth in 1978 in England, and then in 1981 in the United States. Over the last 6 years, the total number of cycles in the U.S. has increased by 44% to nearly 370,000.

The Society for Assisted Reproductive Technology, an affiliate of the American Society for Reproductive Medicine, maintains standards and provides resources and education to both professionals and patients. SART membership consists of more than 350 clinics throughout the United States, representing 80% of ART clinics. Over 95% of ART cycles in 2021 in the United States were performed in SART-member clinics.

Trolice_Mark _P_FLA_2022_web.jpg
Dr. Mark P. Trolice

SART is an invaluable resource for both patients and physicians. Their website includes a “Predict My Success” calculator that allows patients and physicians to enter individualized data to calculate the chance of having a baby over one or more complete cycles of IVF. To help us understand the pregnancy outcome data from ART – cycles per clinic along with national results – I posed the questions below to Amy Sparks, PhD, HCLD, director of the IVF and Andrology Laboratories and the Center for Advanced Reproductive Care at University of Iowa Hospitals and Clinics, Iowa City. Dr. Sparks is past president of SART and former chairperson of the SART Registry committee when the current Clinic Summary Report format was initially released.
 

Question: The Fertility Clinic Success Rate and Certification Act (FCSRCA) of 1992 mandated that all ART clinics report success rate data to the federal government, through the Centers for Disease Control and Prevention, in a standardized manner. As ART is the only field in medicine to be required to annually report their patient outcomes, that is, all initiated cycles and live births, why do you believe this law was enacted and is limited to reproductive medicine?

Answer:
The FCSRCA of 1992 was enacted in response to the lack of open and reliable pregnancy success rate information for patients seeking infertility care using assisted reproductive technologies. Success rates of 25%-50% were being advertised by independent clinics when, nationally, fewer than 15% of ART procedures led to live births. The Federal Trade Commission said such claims were deceptive and filed charges against five clinics, saying they misrepresented their success in helping women become pregnant. The government won one case by court order and the other four cases were settled out of court.

Sparks_Amy_IA_web.jpg
Dr. Amy Sparks

This field of medicine was in the spotlight as the majority of patients lacked insurance coverage for their ART cycles, and there was a strong desire to protect consumers paying out of pocket for relatively low success. Recognizing that the FTC’s mission is to ensure truth in advertising and not regulate medical care, Congress passed the FCSRCA, mandating that all centers providing ART services report all initiated cycles and their outcomes. The CDC was appointed as the agency responsible for collecting cycle data and reporting outcomes. Centers not reporting their cycles are listed as nonreporting centers.

This act also established standards for accreditation of embryology laboratories including personnel and traditional clinical laboratory management requirements. These standards serve as the foundation for embryology laboratory accrediting agencies.
 

 

 

Q: Why have live-birth rates on SART appeared to be focused on “per IVF cycle” as opposed to the CDC reporting of live births “per embryo transfer?”

A:
An ART cycle “start” is defined as the initiation of ovarian stimulation with medication that may or may not include administration of exogenous gonadotropins, followed by oocyte retrieval and embryo transfer. Not every patient beginning a cycle will undergo an oocyte retrieval and not all patients who undergo oocyte retrieval have an embryo transfer. The live-birth rates (LBR) for each of these steps of progression in the ART process are available in the SART and CDC reports.

In 2016, SART recognized that practices were foregoing fresh embryo transfer after oocyte retrieval, opting to cryopreserve all embryos to either accommodate genetic testing of the embryos prior to transfer or to avoid embryo transfer to an unfavorable uterine environment. In response to changes in practice and in an effort to deemphasize live birth per transfer, thereby alleviating a potential motivator or pressure for practitioners to transfer multiple embryos, SART moved to a report that displays the cumulative live-birth rate per cycle start for oocyte retrieval. The cumulative live-birth rate per cycle start for oocyte retrieval is the chance of live birth from transfers of embryos derived from the oocyte retrieval and performed within 1 year of the oocyte retrieval.

This change in reporting further reduced the pressure to transfer multiple embryos and encouraged elective, single-embryo transfer. The outcome per transfer is no longer the report’s primary focus.
 

Q: The latest pregnancy outcomes statistics are from the year 2020 and are finalized by the CDC. Why does the SART website have this same year labeled “preliminary” outcomes?

A:
Shortly after the 2016 SART report change, the CDC made similar changes to their report. The difference is that SART provides a “preliminary” report of outcomes within the year of the cycle start for oocyte retrieval. The cumulative outcome is not “finalized” until the following year as transfers may be performed as late as 12 months after the oocyte retrieval.

SART has opted to report both the “preliminary” or interim outcome and the “final” outcome a year later. The CDC has opted to limit their report to “final” outcomes. I’m happy to report that SART recently released the final report for 2021 cycles.
 

Q: Have national success rates in the United States continued to rise or have they plateaued?

A:
It appears that success rates have plateaued; however, we find ourselves at another point where practice patterns and patients’ approach to using ART for family building have changed.

Recognizing the impact of maternal aging on reproductive potential, patients are opting to undergo multiple ART cycles to cryopreserve embryos for family building before they attempt to get pregnant. This family-building path reduces the value of measuring the LBR per cycle start as we may not know the outcome for many years. SART leaders are deliberating intently as to how to best represent this growing patient population in outcome reporting.
 

 

 

Q: Can you comment on the reduction of multiple gestations with the increasing use of single-embryo transfer?

A:
The reduction in emphasis on live births per transfer, emphasis on singleton live-birth rates in both the SART and CDC reports, and American Society for Reproductive Medicine practice committee guidelines strongly supporting single embryo transfer have significantly reduced the rate of multiple gestations.

A decade ago, only a third of the transfers were single-embryo transfers and over 25% of live births resulted in a multiple birth. Today, the majority of embryo transfers are elective, single-embryo transfers, and the multiple birth rate has been reduced by nearly 80%. In 2020, 93% of live births from IVF were singletons.
 

Q: SART offers an online IVF calculator so both patients and physicians can plug in data for an approximate cumulative success rate for up to three IVF cycles. The calculator pools data from all U.S.-reporting IVF centers. Can you explain what an “IVF cycle” is and what patient information is required? Why do success rates increase over time?

A:
Each “IVF cycle” is a cycle start for an oocyte retrieval and all transfers of embryos from that cycle within a year of the oocyte retrieval. If the first cycle and subsequent transfers do not lead to a live birth, patients still have a chance to achieve a live birth with a second or third cycle. The success rate increases over time as it reflects the chance of success for a population of patients, with some achieving a live birth after the first cycle and additional patients who achieve success following their third cycle.

Q: The SART IVF calculator can be used with no prior IVF cycles or following an unsuccessful cycle. Are there data to support an estimation of outcome following two or even more unsuccessful cycles?

A:
The variables in the SART IVF calculator are based upon the cycle-specific data from patients seeking care at SART member clinics. The current predictor was built with data from cycles performed in 2015-2016. SART is adjusting the predictor and developing a calculator that will be routinely updated, accordingly.

Q: Only approximately 40% of states have some form of infertility coverage law in place; however the number of IVF cycles in the United States continues to increase on an annual basis. What do you think are the driving factors behind this?

A:
Advocacy efforts to improve patients’ access to infertility care have included giving patients tools to encourage their employers to include infertility care in their health care benefits package. More recently, the “Great Resignation” has led to the “Great Recruitment” and employers are recognizing that the addition of infertility care to health care benefits is a powerful recruitment tool.

Dr. Trolice is director of The IVF Center in Winter Park, Fla., and professor of obstetrics and gynecology at the University of Central Florida, Orlando.

Publications
Topics
Sections

The field of assisted reproductive technologies (ART) continues to evolve from its first successful birth in 1978 in England, and then in 1981 in the United States. Over the last 6 years, the total number of cycles in the U.S. has increased by 44% to nearly 370,000.

The Society for Assisted Reproductive Technology, an affiliate of the American Society for Reproductive Medicine, maintains standards and provides resources and education to both professionals and patients. SART membership consists of more than 350 clinics throughout the United States, representing 80% of ART clinics. Over 95% of ART cycles in 2021 in the United States were performed in SART-member clinics.

Trolice_Mark _P_FLA_2022_web.jpg
Dr. Mark P. Trolice

SART is an invaluable resource for both patients and physicians. Their website includes a “Predict My Success” calculator that allows patients and physicians to enter individualized data to calculate the chance of having a baby over one or more complete cycles of IVF. To help us understand the pregnancy outcome data from ART – cycles per clinic along with national results – I posed the questions below to Amy Sparks, PhD, HCLD, director of the IVF and Andrology Laboratories and the Center for Advanced Reproductive Care at University of Iowa Hospitals and Clinics, Iowa City. Dr. Sparks is past president of SART and former chairperson of the SART Registry committee when the current Clinic Summary Report format was initially released.
 

Question: The Fertility Clinic Success Rate and Certification Act (FCSRCA) of 1992 mandated that all ART clinics report success rate data to the federal government, through the Centers for Disease Control and Prevention, in a standardized manner. As ART is the only field in medicine to be required to annually report their patient outcomes, that is, all initiated cycles and live births, why do you believe this law was enacted and is limited to reproductive medicine?

Answer:
The FCSRCA of 1992 was enacted in response to the lack of open and reliable pregnancy success rate information for patients seeking infertility care using assisted reproductive technologies. Success rates of 25%-50% were being advertised by independent clinics when, nationally, fewer than 15% of ART procedures led to live births. The Federal Trade Commission said such claims were deceptive and filed charges against five clinics, saying they misrepresented their success in helping women become pregnant. The government won one case by court order and the other four cases were settled out of court.

Sparks_Amy_IA_web.jpg
Dr. Amy Sparks

This field of medicine was in the spotlight as the majority of patients lacked insurance coverage for their ART cycles, and there was a strong desire to protect consumers paying out of pocket for relatively low success. Recognizing that the FTC’s mission is to ensure truth in advertising and not regulate medical care, Congress passed the FCSRCA, mandating that all centers providing ART services report all initiated cycles and their outcomes. The CDC was appointed as the agency responsible for collecting cycle data and reporting outcomes. Centers not reporting their cycles are listed as nonreporting centers.

This act also established standards for accreditation of embryology laboratories including personnel and traditional clinical laboratory management requirements. These standards serve as the foundation for embryology laboratory accrediting agencies.
 

 

 

Q: Why have live-birth rates on SART appeared to be focused on “per IVF cycle” as opposed to the CDC reporting of live births “per embryo transfer?”

A:
An ART cycle “start” is defined as the initiation of ovarian stimulation with medication that may or may not include administration of exogenous gonadotropins, followed by oocyte retrieval and embryo transfer. Not every patient beginning a cycle will undergo an oocyte retrieval and not all patients who undergo oocyte retrieval have an embryo transfer. The live-birth rates (LBR) for each of these steps of progression in the ART process are available in the SART and CDC reports.

In 2016, SART recognized that practices were foregoing fresh embryo transfer after oocyte retrieval, opting to cryopreserve all embryos to either accommodate genetic testing of the embryos prior to transfer or to avoid embryo transfer to an unfavorable uterine environment. In response to changes in practice and in an effort to deemphasize live birth per transfer, thereby alleviating a potential motivator or pressure for practitioners to transfer multiple embryos, SART moved to a report that displays the cumulative live-birth rate per cycle start for oocyte retrieval. The cumulative live-birth rate per cycle start for oocyte retrieval is the chance of live birth from transfers of embryos derived from the oocyte retrieval and performed within 1 year of the oocyte retrieval.

This change in reporting further reduced the pressure to transfer multiple embryos and encouraged elective, single-embryo transfer. The outcome per transfer is no longer the report’s primary focus.
 

Q: The latest pregnancy outcomes statistics are from the year 2020 and are finalized by the CDC. Why does the SART website have this same year labeled “preliminary” outcomes?

A:
Shortly after the 2016 SART report change, the CDC made similar changes to their report. The difference is that SART provides a “preliminary” report of outcomes within the year of the cycle start for oocyte retrieval. The cumulative outcome is not “finalized” until the following year as transfers may be performed as late as 12 months after the oocyte retrieval.

SART has opted to report both the “preliminary” or interim outcome and the “final” outcome a year later. The CDC has opted to limit their report to “final” outcomes. I’m happy to report that SART recently released the final report for 2021 cycles.
 

Q: Have national success rates in the United States continued to rise or have they plateaued?

A:
It appears that success rates have plateaued; however, we find ourselves at another point where practice patterns and patients’ approach to using ART for family building have changed.

Recognizing the impact of maternal aging on reproductive potential, patients are opting to undergo multiple ART cycles to cryopreserve embryos for family building before they attempt to get pregnant. This family-building path reduces the value of measuring the LBR per cycle start as we may not know the outcome for many years. SART leaders are deliberating intently as to how to best represent this growing patient population in outcome reporting.
 

 

 

Q: Can you comment on the reduction of multiple gestations with the increasing use of single-embryo transfer?

A:
The reduction in emphasis on live births per transfer, emphasis on singleton live-birth rates in both the SART and CDC reports, and American Society for Reproductive Medicine practice committee guidelines strongly supporting single embryo transfer have significantly reduced the rate of multiple gestations.

A decade ago, only a third of the transfers were single-embryo transfers and over 25% of live births resulted in a multiple birth. Today, the majority of embryo transfers are elective, single-embryo transfers, and the multiple birth rate has been reduced by nearly 80%. In 2020, 93% of live births from IVF were singletons.
 

Q: SART offers an online IVF calculator so both patients and physicians can plug in data for an approximate cumulative success rate for up to three IVF cycles. The calculator pools data from all U.S.-reporting IVF centers. Can you explain what an “IVF cycle” is and what patient information is required? Why do success rates increase over time?

A:
Each “IVF cycle” is a cycle start for an oocyte retrieval and all transfers of embryos from that cycle within a year of the oocyte retrieval. If the first cycle and subsequent transfers do not lead to a live birth, patients still have a chance to achieve a live birth with a second or third cycle. The success rate increases over time as it reflects the chance of success for a population of patients, with some achieving a live birth after the first cycle and additional patients who achieve success following their third cycle.

Q: The SART IVF calculator can be used with no prior IVF cycles or following an unsuccessful cycle. Are there data to support an estimation of outcome following two or even more unsuccessful cycles?

A:
The variables in the SART IVF calculator are based upon the cycle-specific data from patients seeking care at SART member clinics. The current predictor was built with data from cycles performed in 2015-2016. SART is adjusting the predictor and developing a calculator that will be routinely updated, accordingly.

Q: Only approximately 40% of states have some form of infertility coverage law in place; however the number of IVF cycles in the United States continues to increase on an annual basis. What do you think are the driving factors behind this?

A:
Advocacy efforts to improve patients’ access to infertility care have included giving patients tools to encourage their employers to include infertility care in their health care benefits package. More recently, the “Great Resignation” has led to the “Great Recruitment” and employers are recognizing that the addition of infertility care to health care benefits is a powerful recruitment tool.

Dr. Trolice is director of The IVF Center in Winter Park, Fla., and professor of obstetrics and gynecology at the University of Central Florida, Orlando.

The field of assisted reproductive technologies (ART) continues to evolve from its first successful birth in 1978 in England, and then in 1981 in the United States. Over the last 6 years, the total number of cycles in the U.S. has increased by 44% to nearly 370,000.

The Society for Assisted Reproductive Technology, an affiliate of the American Society for Reproductive Medicine, maintains standards and provides resources and education to both professionals and patients. SART membership consists of more than 350 clinics throughout the United States, representing 80% of ART clinics. Over 95% of ART cycles in 2021 in the United States were performed in SART-member clinics.

Trolice_Mark _P_FLA_2022_web.jpg
Dr. Mark P. Trolice

SART is an invaluable resource for both patients and physicians. Their website includes a “Predict My Success” calculator that allows patients and physicians to enter individualized data to calculate the chance of having a baby over one or more complete cycles of IVF. To help us understand the pregnancy outcome data from ART – cycles per clinic along with national results – I posed the questions below to Amy Sparks, PhD, HCLD, director of the IVF and Andrology Laboratories and the Center for Advanced Reproductive Care at University of Iowa Hospitals and Clinics, Iowa City. Dr. Sparks is past president of SART and former chairperson of the SART Registry committee when the current Clinic Summary Report format was initially released.
 

Question: The Fertility Clinic Success Rate and Certification Act (FCSRCA) of 1992 mandated that all ART clinics report success rate data to the federal government, through the Centers for Disease Control and Prevention, in a standardized manner. As ART is the only field in medicine to be required to annually report their patient outcomes, that is, all initiated cycles and live births, why do you believe this law was enacted and is limited to reproductive medicine?

Answer:
The FCSRCA of 1992 was enacted in response to the lack of open and reliable pregnancy success rate information for patients seeking infertility care using assisted reproductive technologies. Success rates of 25%-50% were being advertised by independent clinics when, nationally, fewer than 15% of ART procedures led to live births. The Federal Trade Commission said such claims were deceptive and filed charges against five clinics, saying they misrepresented their success in helping women become pregnant. The government won one case by court order and the other four cases were settled out of court.

Sparks_Amy_IA_web.jpg
Dr. Amy Sparks

This field of medicine was in the spotlight as the majority of patients lacked insurance coverage for their ART cycles, and there was a strong desire to protect consumers paying out of pocket for relatively low success. Recognizing that the FTC’s mission is to ensure truth in advertising and not regulate medical care, Congress passed the FCSRCA, mandating that all centers providing ART services report all initiated cycles and their outcomes. The CDC was appointed as the agency responsible for collecting cycle data and reporting outcomes. Centers not reporting their cycles are listed as nonreporting centers.

This act also established standards for accreditation of embryology laboratories including personnel and traditional clinical laboratory management requirements. These standards serve as the foundation for embryology laboratory accrediting agencies.
 

 

 

Q: Why have live-birth rates on SART appeared to be focused on “per IVF cycle” as opposed to the CDC reporting of live births “per embryo transfer?”

A:
An ART cycle “start” is defined as the initiation of ovarian stimulation with medication that may or may not include administration of exogenous gonadotropins, followed by oocyte retrieval and embryo transfer. Not every patient beginning a cycle will undergo an oocyte retrieval and not all patients who undergo oocyte retrieval have an embryo transfer. The live-birth rates (LBR) for each of these steps of progression in the ART process are available in the SART and CDC reports.

In 2016, SART recognized that practices were foregoing fresh embryo transfer after oocyte retrieval, opting to cryopreserve all embryos to either accommodate genetic testing of the embryos prior to transfer or to avoid embryo transfer to an unfavorable uterine environment. In response to changes in practice and in an effort to deemphasize live birth per transfer, thereby alleviating a potential motivator or pressure for practitioners to transfer multiple embryos, SART moved to a report that displays the cumulative live-birth rate per cycle start for oocyte retrieval. The cumulative live-birth rate per cycle start for oocyte retrieval is the chance of live birth from transfers of embryos derived from the oocyte retrieval and performed within 1 year of the oocyte retrieval.

This change in reporting further reduced the pressure to transfer multiple embryos and encouraged elective, single-embryo transfer. The outcome per transfer is no longer the report’s primary focus.
 

Q: The latest pregnancy outcomes statistics are from the year 2020 and are finalized by the CDC. Why does the SART website have this same year labeled “preliminary” outcomes?

A:
Shortly after the 2016 SART report change, the CDC made similar changes to their report. The difference is that SART provides a “preliminary” report of outcomes within the year of the cycle start for oocyte retrieval. The cumulative outcome is not “finalized” until the following year as transfers may be performed as late as 12 months after the oocyte retrieval.

SART has opted to report both the “preliminary” or interim outcome and the “final” outcome a year later. The CDC has opted to limit their report to “final” outcomes. I’m happy to report that SART recently released the final report for 2021 cycles.
 

Q: Have national success rates in the United States continued to rise or have they plateaued?

A:
It appears that success rates have plateaued; however, we find ourselves at another point where practice patterns and patients’ approach to using ART for family building have changed.

Recognizing the impact of maternal aging on reproductive potential, patients are opting to undergo multiple ART cycles to cryopreserve embryos for family building before they attempt to get pregnant. This family-building path reduces the value of measuring the LBR per cycle start as we may not know the outcome for many years. SART leaders are deliberating intently as to how to best represent this growing patient population in outcome reporting.
 

 

 

Q: Can you comment on the reduction of multiple gestations with the increasing use of single-embryo transfer?

A:
The reduction in emphasis on live births per transfer, emphasis on singleton live-birth rates in both the SART and CDC reports, and American Society for Reproductive Medicine practice committee guidelines strongly supporting single embryo transfer have significantly reduced the rate of multiple gestations.

A decade ago, only a third of the transfers were single-embryo transfers and over 25% of live births resulted in a multiple birth. Today, the majority of embryo transfers are elective, single-embryo transfers, and the multiple birth rate has been reduced by nearly 80%. In 2020, 93% of live births from IVF were singletons.
 

Q: SART offers an online IVF calculator so both patients and physicians can plug in data for an approximate cumulative success rate for up to three IVF cycles. The calculator pools data from all U.S.-reporting IVF centers. Can you explain what an “IVF cycle” is and what patient information is required? Why do success rates increase over time?

A:
Each “IVF cycle” is a cycle start for an oocyte retrieval and all transfers of embryos from that cycle within a year of the oocyte retrieval. If the first cycle and subsequent transfers do not lead to a live birth, patients still have a chance to achieve a live birth with a second or third cycle. The success rate increases over time as it reflects the chance of success for a population of patients, with some achieving a live birth after the first cycle and additional patients who achieve success following their third cycle.

Q: The SART IVF calculator can be used with no prior IVF cycles or following an unsuccessful cycle. Are there data to support an estimation of outcome following two or even more unsuccessful cycles?

A:
The variables in the SART IVF calculator are based upon the cycle-specific data from patients seeking care at SART member clinics. The current predictor was built with data from cycles performed in 2015-2016. SART is adjusting the predictor and developing a calculator that will be routinely updated, accordingly.

Q: Only approximately 40% of states have some form of infertility coverage law in place; however the number of IVF cycles in the United States continues to increase on an annual basis. What do you think are the driving factors behind this?

A:
Advocacy efforts to improve patients’ access to infertility care have included giving patients tools to encourage their employers to include infertility care in their health care benefits package. More recently, the “Great Resignation” has led to the “Great Recruitment” and employers are recognizing that the addition of infertility care to health care benefits is a powerful recruitment tool.

Dr. Trolice is director of The IVF Center in Winter Park, Fla., and professor of obstetrics and gynecology at the University of Central Florida, Orlando.

Publications
Publications
Topics
Article Type
Sections
Teambase XML
<?xml version="1.0" encoding="UTF-8"?>
<!--$RCSfile: InCopy_agile.xsl,v $ $Revision: 1.35 $-->
<!--$RCSfile: drupal.xsl,v $ $Revision: 1.7 $-->
<root generator="drupal.xsl" gversion="1.7"> <header> <fileName>163235</fileName> <TBEID>0C049D40.SIG</TBEID> <TBUniqueIdentifier>MD_0C049D40</TBUniqueIdentifier> <newsOrJournal>News</newsOrJournal> <publisherName>Frontline Medical Communications</publisherName> <storyname>Reproductive Rounds column</storyname> <articleType>353</articleType> <TBLocation>QC Done-All Pubs</TBLocation> <QCDate>20230428T163941</QCDate> <firstPublished>20230428T164052</firstPublished> <LastPublished>20230428T164052</LastPublished> <pubStatus qcode="stat:"/> <embargoDate/> <killDate/> <CMSDate>20230428T164052</CMSDate> <articleSource/> <facebookInfo/> <meetingNumber/> <byline>Other Writer</byline> <bylineText>MARK TROLICE MD, MBA</bylineText> <bylineFull>MARK TROLICE MD, MBA</bylineFull> <bylineTitleText/> <USOrGlobal/> <wireDocType/> <newsDocType>Column</newsDocType> <journalDocType/> <linkLabel/> <pageRange/> <citation/> <quizID/> <indexIssueDate/> <itemClass qcode="ninat:text"/> <provider qcode="provider:imng"> <name>IMNG Medical Media</name> <rightsInfo> <copyrightHolder> <name>Frontline Medical News</name> </copyrightHolder> <copyrightNotice>Copyright (c) 2015 Frontline Medical News, a Frontline Medical Communications Inc. company. All rights reserved. This material may not be published, broadcast, copied, or otherwise reproduced or distributed without the prior written permission of Frontline Medical Communications Inc.</copyrightNotice> </rightsInfo> </provider> <abstract/> <metaDescription>The Society for Assisted Reproductive Technology, an affiliate of the American Society for Reproductive Medicine, maintains standards and provides resources and</metaDescription> <articlePDF/> <teaserImage>284599</teaserImage> <teaser>Patients are opting to undergo multiple ART cycles to cryopreserve embryos for family building before they attempt to get pregnant, says Dr. Amy Sparks.</teaser> <title>Understanding clinic-reported IVF success rates</title> <deck/> <disclaimer/> <AuthorList/> <articleURL/> <doi/> <pubMedID/> <publishXMLStatus/> <publishXMLVersion>1</publishXMLVersion> <useEISSN>0</useEISSN> <urgency/> <pubPubdateYear/> <pubPubdateMonth/> <pubPubdateDay/> <pubVolume/> <pubNumber/> <wireChannels/> <primaryCMSID/> <CMSIDs/> <keywords/> <seeAlsos/> <publications_g> <publicationData> <publicationCode>ob</publicationCode> <pubIssueName/> <pubArticleType/> <pubTopics/> <pubCategories/> <pubSections/> </publicationData> </publications_g> <publications> <term canonical="true">23</term> </publications> <sections> <term canonical="true">68807</term> <term>52</term> </sections> <topics> <term canonical="true">287</term> <term>262</term> </topics> <links> <link> <itemClass qcode="ninat:picture"/> <altRep contenttype="image/jpeg">images/24010881.jpg</altRep> <description role="drol:caption">Dr. Mark P. Trolice</description> <description role="drol:credit">Fertility CARE</description> </link> <link> <itemClass qcode="ninat:picture"/> <altRep contenttype="image/jpeg">images/24011cad.jpg</altRep> <description role="drol:caption">Dr. Amy Sparks</description> <description role="drol:credit">University of Iowa</description> </link> </links> </header> <itemSet> <newsItem> <itemMeta> <itemRole>Main</itemRole> <itemClass>text</itemClass> <title>Understanding clinic-reported IVF success rates</title> <deck/> </itemMeta> <itemContent> <p>The field of assisted reproductive technologies (ART) continues to evolve from its first successful birth in 1978 in England, and then in 1981 in the United States. Over the last 6 years, the total number of cycles in the U.S. has increased by 44% to nearly 370,000.</p> <p><span class="tag metaDescription">The <span class="Hyperlink"><a href="http://sart.org/">Society for Assisted Reproductive Technology</a></span>, an affiliate of the <span class="Hyperlink"><a href="C:\Users\DrT\Dropbox\Ob.Gyn.News\SART Q&amp;A Sparks\asrm.org">American Society for Reproductive Medicine</a></span>, maintains standards and provides resources and education to both professionals and patients.</span> SART membership consists of more than 350 clinics throughout the United States, representing 80% of ART clinics. Over 95% of ART cycles in 2021 in the United States were performed in SART-member clinics.<br/><br/>[[{"fid":"284599","view_mode":"medstat_image_flush_left","fields":{"format":"medstat_image_flush_left","field_file_image_alt_text[und][0][value]":"Dr. Trolice is director of The IVF Center in Winter Park, Fla., and professor of obstetrics and gynecology at the University of Central Florida, Orlando.","field_file_image_credit[und][0][value]":"Fertility CARE","field_file_image_caption[und][0][value]":"Dr. Mark P. Trolice"},"type":"media","attributes":{"class":"media-element file-medstat_image_flush_left"}}]]SART is an invaluable resource for both patients and physicians. Their website includes a “Predict My Success” calculator that allows patients and physicians to enter individualized data to calculate the chance of having a baby over one or more complete cycles of IVF. To help us understand the pregnancy outcome data from ART – cycles per clinic along with national results – I posed the questions below to Amy Sparks, PhD, HCLD, director of the IVF and Andrology Laboratories and the Center for Advanced Reproductive Care at University of Iowa Hospitals and Clinics, Iowa City. Dr. Sparks is past president of SART and former chairperson of the SART Registry committee when the current Clinic Summary Report format was initially released.<br/><br/></p> <p><strong>Question: The Fertility Clinic Success Rate and Certification Act (FCSRCA) of 1992 mandated that all ART clinics report success rate data to the federal government, through the Centers for Disease Control and Prevention, in a standardized manner. As ART is the only field in medicine to be required to annually report their patient outcomes, that is, all initiated cycles and live births, why do you believe this law was enacted and is limited to reproductive medicine?<br/><br/>Answer:</strong> The FCSRCA of 1992 was enacted in response to the lack of open and reliable pregnancy success rate information for patients seeking infertility care using assisted reproductive technologies. Success rates of 25%-50% were being advertised by independent clinics when, nationally, fewer than 15% of ART procedures led to live births. The Federal Trade Commission said such claims were deceptive and filed charges against five clinics, saying they misrepresented their success in helping women become pregnant. The government won one case by court order and the other four cases were settled out of court.</p> <p>[[{"fid":"294642","view_mode":"medstat_image_flush_left","fields":{"format":"medstat_image_flush_left","field_file_image_alt_text[und][0][value]":"Obstetrics and Gynecology, University of Iowa; Director, In Vitro Fertilization and Reproductive Testing Laboratories; past president of SART and former chairperson the SART Registry committee","field_file_image_credit[und][0][value]":"University of Iowa","field_file_image_caption[und][0][value]":"Dr. Amy Sparks"},"type":"media","attributes":{"class":"media-element file-medstat_image_flush_left"}}]]This field of medicine was in the spotlight as the majority of patients lacked insurance coverage for their ART cycles, and there was a strong desire to protect consumers paying out of pocket for relatively low success. Recognizing that the FTC’s mission is to ensure truth in advertising and not regulate medical care, Congress passed the FCSRCA, mandating that all centers providing ART services report all initiated cycles and their outcomes. The CDC was appointed as the agency responsible for collecting cycle data and reporting outcomes. Centers not reporting their cycles are listed as nonreporting centers.<br/><br/>This act also established standards for accreditation of embryology laboratories including personnel and traditional clinical laboratory management requirements. These standards serve as the foundation for embryology laboratory accrediting agencies.<br/><br/></p> <p><strong>Q: Why have <a href="https://www.sartcorsonline.com/members/Search">live-birth rates</a> on SART appeared to be focused on “per IVF cycle” as opposed to <a href="https://www.cdc.gov/art/artdata/index.html">the CDC reporting</a> of live births “per embryo transfer?”<br/><br/>A: </strong>An ART cycle “start” is defined as the initiation of ovarian stimulation with medication that may or may not include administration of exogenous gonadotropins, followed by oocyte retrieval and embryo transfer. Not every patient beginning a cycle will undergo an oocyte retrieval and not all patients who undergo oocyte retrieval have an embryo transfer. The live-birth rates (LBR) for each of these steps of progression in the ART process are available in the SART and CDC reports.</p> <p>In 2016, SART recognized that practices were foregoing fresh embryo transfer after oocyte retrieval, opting to cryopreserve all embryos to either accommodate genetic testing of the embryos prior to transfer or to avoid embryo transfer to an unfavorable uterine environment. In response to changes in practice and in an effort to deemphasize live birth per transfer, thereby alleviating a potential motivator or pressure for practitioners to transfer multiple embryos, SART moved to a report that displays the cumulative live-birth rate per cycle start for oocyte retrieval. The cumulative live-birth rate per cycle start for oocyte retrieval is the chance of live birth from transfers of embryos derived from the oocyte retrieval and performed within 1 year of the oocyte retrieval.<br/><br/>This change in reporting further reduced the pressure to transfer multiple embryos and encouraged elective, single-embryo transfer. The outcome per transfer is no longer the report’s primary focus.<br/><br/></p> <p><strong>Q: The latest pregnancy outcomes statistics are from the year 2020 and are finalized by the CDC. Why does the SART website have this same year labeled “preliminary” outcomes?<br/><br/>A: </strong>Shortly after the 2016 SART report change, the CDC made similar changes to their report. The difference is that SART provides a “preliminary” report of outcomes within the year of the cycle start for oocyte retrieval. The cumulative outcome is not “finalized” until the following year as transfers may be performed as late as 12 months after the oocyte retrieval.</p> <p>SART has opted to report both the “preliminary” or interim outcome and the “final” outcome a year later. The CDC has opted to limit their report to “final” outcomes. I’m happy to report that SART recently released the final report for 2021 cycles.<br/><br/></p> <p><strong>Q: Have national success rates in the United States continued to rise or have they plateaued?<br/><br/>A: </strong>It appears that success rates have plateaued; however, we find ourselves at another point where practice patterns and patients’ approach to using ART for family building have changed.</p> <p>Recognizing the impact of maternal aging on reproductive potential, patients are opting to undergo multiple ART cycles to cryopreserve embryos for family building before they attempt to get pregnant. This family-building path reduces the value of measuring the LBR per cycle start as we may not know the outcome for many years. SART leaders are deliberating intently as to how to best represent this growing patient population in outcome reporting.<br/><br/></p> <p><strong>Q: Can you comment on the reduction of multiple gestations with the increasing use of single-embryo transfer?<br/><br/>A: </strong>The reduction in emphasis on live births per transfer, emphasis on singleton live-birth rates in both the SART and CDC reports, and American Society for Reproductive Medicine practice committee guidelines strongly supporting single embryo transfer have significantly reduced the rate of multiple gestations.</p> <p>A decade ago, only a third of the transfers were single-embryo transfers and over 25% of live births resulted in a multiple birth. Today, the majority of embryo transfers are elective, single-embryo transfers, and the multiple birth rate has been reduced by nearly 80%. In 2020, 93% of live births from IVF were singletons. <br/><br/></p> <p><strong>Q: SART offers an <a href="https://w3.abdn.ac.uk/clsm/SARTIVF/">online IVF calculator</a> so both patients and physicians can plug in data for an approximate cumulative success rate for up to three IVF cycles. The calculator pools data from all U.S.-reporting IVF centers. Can you explain what an “IVF cycle” is and what patient information is required? Why do success rates increase over time?<br/><br/>A: </strong>Each “IVF cycle” is a cycle start for an oocyte retrieval and all transfers of embryos from that cycle within a year of the oocyte retrieval. If the first cycle and subsequent transfers do not lead to a live birth, patients still have a chance to achieve a live birth with a second or third cycle. The success rate increases over time as it reflects the chance of success for a population of patients, with some achieving a live birth after the first cycle and additional patients who achieve success following their third cycle.</p> <p><strong>Q: The SART IVF calculator can be used with no prior IVF cycles or following an unsuccessful cycle. Are there data to support an estimation of outcome following two or even more unsuccessful cycles?<br/><br/>A: </strong>The variables in the SART IVF calculator are based upon the cycle-specific data from patients seeking care at SART member clinics. The current predictor was built with data from cycles performed in 2015-2016. SART is adjusting the predictor and developing a calculator that will be routinely updated, accordingly.</p> <p><strong>Q: Only approximately 40% of states have some form of infertility coverage law in place; however the number of IVF cycles in the United States continues to increase on an annual basis. What do you think are the driving factors behind this?<br/><br/>A: </strong>Advocacy efforts to improve patients’ access to infertility care have included giving patients tools to encourage their employers to include infertility care in their health care benefits package. More recently, the “Great Resignation” has led to the “Great Recruitment” and employers are recognizing that the addition of infertility care to health care benefits is a powerful recruitment tool.<span class="end"/></p> <p> <em>Dr. Trolice is director of The IVF Center in Winter Park, Fla., and professor of obstetrics and gynecology at the University of Central Florida, Orlando.</em> </p> </itemContent> </newsItem> <newsItem> <itemMeta> <itemRole>teaser</itemRole> <itemClass>text</itemClass> <title/> <deck/> </itemMeta> <itemContent> </itemContent> </newsItem> </itemSet></root>
Disallow All Ads
Content Gating
No Gating (article Unlocked/Free)
Alternative CME
Disqus Comments
Default
Use ProPublica
Hide sidebar & use full width
render the right sidebar.
Conference Recap Checkbox
Not Conference Recap
Clinical Edge
Display the Slideshow in this Article
Medscape Article
Display survey writer
Reuters content
Disable Inline Native ads
WebMD Article

The physician as leader

Article Type
Changed
Fri, 03/31/2023 - 11:09

 

Physicians are placed in positions of leadership by the medical team, by the community, and by society, particularly during times of crisis such as the COVID pandemic. They are looked to by the media at times of health care news such as the overturning of Roe v. Wade.1 In a 2015 survey of resident physicians, two-thirds agreed that a formalized leadership curriculum would help them become better supervisors and clinicians.2 While all physicians are viewed as leaders, the concept of leadership is rarely, if ever, described or developed as a part of medical training. This month’s column will provide insights into defining leadership as a physician in the medical and administrative settings.

Benefits of effective leadership

Physicians, whether they are clinicians, researchers, administrators, or teachers, are expected to oversee and engage their teams. A report by the Institute of Medicine recommended that academic health centers “develop leaders at all levels who can manage the organizational and system changes necessary to improve health through innovation in health professions education, patient care, and research.”3 Hospitals with higher-rated management practices and more highly rated boards of directors have been shown to deliver higher-quality care and better clinical outcomes, including lower mortality.

Trolice_Mark _P_FLA_2022_web.jpg
Dr. Mark P. Trolice

To illustrate, the clinicians at the Mayo Clinic annually rate their supervisors on a Leader Index, a simple 12-question survey of five leadership domains: truthfulness, transparency, character, capability, and partnership. All supervisors were physicians and scientists. Their findings revealed that for each one-point increase in composite leadership score, there was a 3.3% decrease in the likelihood of burnout and a 9.0% increase in the likelihood of satisfaction in the physicians supervised.4

Interprofessional teamwork and engagement are vital skills for a leader to create a successful team. Enhanced management practices have also been associated with higher patient approval ratings and better financial performance. Effective leadership additionally affects physician well-being, with stronger leadership associated with less physician burnout and higher satisfaction.5

Leadership styles enhance quality measures in health care.6 The most effective leadership styles are ones in which the staff feels they are part of a team, are engaged, and are mentored.7 While leadership styles can vary, the common theme is staff engagement. An authoritative style leader is one who mobilizes the team toward a vision, that is, “Come with me.” An affiliative style leader creates harmony and builds emotional bonds where “people come first.” Democratic leaders forge a consensus through staff participation by asking, “What do you think?” Finally, a leader who uses a coaching style helps staff to identify their strengths and weaknesses and work toward improvement. These leadership behaviors are in contradistinction to the unsuccessful coercive leader who demands immediate compliance, that is, “Do what I tell you.”

Five fundamental leadership principles are shown in Table 1.8

Effective leaders have an open (growth) mindset, unwavering attention to diversity, equity, and inclusion, and to building relationships and trust; they practice effective communication and listening, focus on results, and cocreate support structures.

162629a_graphic_web.png

A growth mindset is the belief that one’s abilities are not innate but can improve through effort and learning.9

 

 

Emotional intelligence

A survey of business senior managers rated the qualities found in the most outstanding leaders. Using objective criteria like profitability the study psychologists interviewed the highest-rated leaders to compare their capabilities. While intellects and cognitive skills were important, the results showed that emotional intelligence (EI) was twice as important as technical skills and IQ.10 As an example, in a 1996 study, when senior managers had an optimal level of EI, their division’s yearly earnings were 20% higher than estimated.11

EI is a leadership competency that deals with the ability to understand and manage your own emotions and your interactions with others.10 At the Cleveland Clinic, EI is exemplified by the acronym HEART, whereby the team strives to improve the patient experience, mainly when an error occurs. The health care team is using EI by showing its the ability to Hear, Empathize, Apologize, Reply, and Thank. When an untoward event occurs, the physician, as the leader of the team, must lead by example when communicating with staff and patients. EI consists of five components (Table 2).13

162629b_graphic_web.png

  • Self-awareness is insight by which you can improve. Maintaining a journal of your daily thoughts may assist with this as well as simply pausing to pay attention during times of heightened emotions.
  • Self-regulation shows control, that is, behaving according to your values, and being accountable and calm when challenged.
  • Purpose, knowing your “why,” produces motivation and helps maintain optimism.
  • Empathy shows the ability to understand the emotions of other people.
  • Social skill is the ability to establish mutually rewarding relationships.

Given all the above benefits, it is no surprise that companies are actively trying use artificial intelligence to improve EI.12
 

Learning to be a leader

In medical school, students are expected to develop skills to handle and resolve conflicts, learn to share leadership, take mutual responsibility, and monitor their own performance.13 Although training of young physicians in leadership is not unprecedented, a systemic review revealed a lack of analytic studies to evaluate the effectiveness of the teaching methods.14 During undergraduate medical education, standard curricula and methods of instruction on leadership are not established, resulting in variable outcomes.

The Association of American Medical Colleges offers a curriculum, “Preparing Medical Students to Be Physician Leaders: A Leadership Training Program for Students Designed and Led by Students.”15 The objectives of this training are to help students identify their “personal style of leadership, recognize strengths and weaknesses, utilize effective communication strategies, appropriately delegate team member responsibilities, and provide constructive feedback to help improve team function.”
 

Take-home points

Following the completion of formal medical education, physicians are thrust into leadership roles. The key to being an effective leader is using EI to mentor the team and make staff feel connected to the team’s meaning and purpose, so they feel valued.

 

Dr. Trolice is director of The IVF Center in Winter Park, Fla., and professor of obstetrics and gynecology at the University of Central Florida, Orlando.

References

1. Carsen S and Xia C. McGill J Med. 2006 Jan;9(1):1-2.

2. Jardine D et al. J Grad Med Educ. 2015;7(2):307-9.

3. Institute of Medicine. Acad Emerg Med. July 2004;11(7):802-6.

4. Shanafelt TD et al. Mayo Clin Proc. April 2015;90(4):432-40.

5. Rotenstein LS et al. Harv Bus Rev. Oct. 17, 2018.

6. Sfantou SF. Healthcare 2017;5(4):73.


7. Goleman D. Harv Bus Rev. March-April 2000.

8. Collins-Nakai R. McGill J Med [Internet]. 2020 Dec. 1 [cited 2023 Mar. 28];9(1).

9. Dweck C. Harv Bus Rev. Jan. 13, 2016.


10. Goleman D. Harv Bus Rev. 1998 Nov-Dec;76(6):93-102..

11. Goleman D et al. Primal leadership: Realizing the power of emotional intelligence. Boston: Harvard Business School Publishing, 2002.12. Limon D and Plaster B. Harv Bus Rev. Jan. 25, 2022.

13. Chen T-Y. Tzu Chi Med J. Apr–Jun 2018;30(2):66-70.

14. Kumar B et al. BMC Med Educ. 2020;20:175.

15. Richards K et al. Med Ed Portal. Dec. 13 2019.

Publications
Topics
Sections

 

Physicians are placed in positions of leadership by the medical team, by the community, and by society, particularly during times of crisis such as the COVID pandemic. They are looked to by the media at times of health care news such as the overturning of Roe v. Wade.1 In a 2015 survey of resident physicians, two-thirds agreed that a formalized leadership curriculum would help them become better supervisors and clinicians.2 While all physicians are viewed as leaders, the concept of leadership is rarely, if ever, described or developed as a part of medical training. This month’s column will provide insights into defining leadership as a physician in the medical and administrative settings.

Benefits of effective leadership

Physicians, whether they are clinicians, researchers, administrators, or teachers, are expected to oversee and engage their teams. A report by the Institute of Medicine recommended that academic health centers “develop leaders at all levels who can manage the organizational and system changes necessary to improve health through innovation in health professions education, patient care, and research.”3 Hospitals with higher-rated management practices and more highly rated boards of directors have been shown to deliver higher-quality care and better clinical outcomes, including lower mortality.

Trolice_Mark _P_FLA_2022_web.jpg
Dr. Mark P. Trolice

To illustrate, the clinicians at the Mayo Clinic annually rate their supervisors on a Leader Index, a simple 12-question survey of five leadership domains: truthfulness, transparency, character, capability, and partnership. All supervisors were physicians and scientists. Their findings revealed that for each one-point increase in composite leadership score, there was a 3.3% decrease in the likelihood of burnout and a 9.0% increase in the likelihood of satisfaction in the physicians supervised.4

Interprofessional teamwork and engagement are vital skills for a leader to create a successful team. Enhanced management practices have also been associated with higher patient approval ratings and better financial performance. Effective leadership additionally affects physician well-being, with stronger leadership associated with less physician burnout and higher satisfaction.5

Leadership styles enhance quality measures in health care.6 The most effective leadership styles are ones in which the staff feels they are part of a team, are engaged, and are mentored.7 While leadership styles can vary, the common theme is staff engagement. An authoritative style leader is one who mobilizes the team toward a vision, that is, “Come with me.” An affiliative style leader creates harmony and builds emotional bonds where “people come first.” Democratic leaders forge a consensus through staff participation by asking, “What do you think?” Finally, a leader who uses a coaching style helps staff to identify their strengths and weaknesses and work toward improvement. These leadership behaviors are in contradistinction to the unsuccessful coercive leader who demands immediate compliance, that is, “Do what I tell you.”

Five fundamental leadership principles are shown in Table 1.8

Effective leaders have an open (growth) mindset, unwavering attention to diversity, equity, and inclusion, and to building relationships and trust; they practice effective communication and listening, focus on results, and cocreate support structures.

162629a_graphic_web.png

A growth mindset is the belief that one’s abilities are not innate but can improve through effort and learning.9

 

 

Emotional intelligence

A survey of business senior managers rated the qualities found in the most outstanding leaders. Using objective criteria like profitability the study psychologists interviewed the highest-rated leaders to compare their capabilities. While intellects and cognitive skills were important, the results showed that emotional intelligence (EI) was twice as important as technical skills and IQ.10 As an example, in a 1996 study, when senior managers had an optimal level of EI, their division’s yearly earnings were 20% higher than estimated.11

EI is a leadership competency that deals with the ability to understand and manage your own emotions and your interactions with others.10 At the Cleveland Clinic, EI is exemplified by the acronym HEART, whereby the team strives to improve the patient experience, mainly when an error occurs. The health care team is using EI by showing its the ability to Hear, Empathize, Apologize, Reply, and Thank. When an untoward event occurs, the physician, as the leader of the team, must lead by example when communicating with staff and patients. EI consists of five components (Table 2).13

162629b_graphic_web.png

  • Self-awareness is insight by which you can improve. Maintaining a journal of your daily thoughts may assist with this as well as simply pausing to pay attention during times of heightened emotions.
  • Self-regulation shows control, that is, behaving according to your values, and being accountable and calm when challenged.
  • Purpose, knowing your “why,” produces motivation and helps maintain optimism.
  • Empathy shows the ability to understand the emotions of other people.
  • Social skill is the ability to establish mutually rewarding relationships.

Given all the above benefits, it is no surprise that companies are actively trying use artificial intelligence to improve EI.12
 

Learning to be a leader

In medical school, students are expected to develop skills to handle and resolve conflicts, learn to share leadership, take mutual responsibility, and monitor their own performance.13 Although training of young physicians in leadership is not unprecedented, a systemic review revealed a lack of analytic studies to evaluate the effectiveness of the teaching methods.14 During undergraduate medical education, standard curricula and methods of instruction on leadership are not established, resulting in variable outcomes.

The Association of American Medical Colleges offers a curriculum, “Preparing Medical Students to Be Physician Leaders: A Leadership Training Program for Students Designed and Led by Students.”15 The objectives of this training are to help students identify their “personal style of leadership, recognize strengths and weaknesses, utilize effective communication strategies, appropriately delegate team member responsibilities, and provide constructive feedback to help improve team function.”
 

Take-home points

Following the completion of formal medical education, physicians are thrust into leadership roles. The key to being an effective leader is using EI to mentor the team and make staff feel connected to the team’s meaning and purpose, so they feel valued.

 

Dr. Trolice is director of The IVF Center in Winter Park, Fla., and professor of obstetrics and gynecology at the University of Central Florida, Orlando.

References

1. Carsen S and Xia C. McGill J Med. 2006 Jan;9(1):1-2.

2. Jardine D et al. J Grad Med Educ. 2015;7(2):307-9.

3. Institute of Medicine. Acad Emerg Med. July 2004;11(7):802-6.

4. Shanafelt TD et al. Mayo Clin Proc. April 2015;90(4):432-40.

5. Rotenstein LS et al. Harv Bus Rev. Oct. 17, 2018.

6. Sfantou SF. Healthcare 2017;5(4):73.


7. Goleman D. Harv Bus Rev. March-April 2000.

8. Collins-Nakai R. McGill J Med [Internet]. 2020 Dec. 1 [cited 2023 Mar. 28];9(1).

9. Dweck C. Harv Bus Rev. Jan. 13, 2016.


10. Goleman D. Harv Bus Rev. 1998 Nov-Dec;76(6):93-102..

11. Goleman D et al. Primal leadership: Realizing the power of emotional intelligence. Boston: Harvard Business School Publishing, 2002.12. Limon D and Plaster B. Harv Bus Rev. Jan. 25, 2022.

13. Chen T-Y. Tzu Chi Med J. Apr–Jun 2018;30(2):66-70.

14. Kumar B et al. BMC Med Educ. 2020;20:175.

15. Richards K et al. Med Ed Portal. Dec. 13 2019.

 

Physicians are placed in positions of leadership by the medical team, by the community, and by society, particularly during times of crisis such as the COVID pandemic. They are looked to by the media at times of health care news such as the overturning of Roe v. Wade.1 In a 2015 survey of resident physicians, two-thirds agreed that a formalized leadership curriculum would help them become better supervisors and clinicians.2 While all physicians are viewed as leaders, the concept of leadership is rarely, if ever, described or developed as a part of medical training. This month’s column will provide insights into defining leadership as a physician in the medical and administrative settings.

Benefits of effective leadership

Physicians, whether they are clinicians, researchers, administrators, or teachers, are expected to oversee and engage their teams. A report by the Institute of Medicine recommended that academic health centers “develop leaders at all levels who can manage the organizational and system changes necessary to improve health through innovation in health professions education, patient care, and research.”3 Hospitals with higher-rated management practices and more highly rated boards of directors have been shown to deliver higher-quality care and better clinical outcomes, including lower mortality.

Trolice_Mark _P_FLA_2022_web.jpg
Dr. Mark P. Trolice

To illustrate, the clinicians at the Mayo Clinic annually rate their supervisors on a Leader Index, a simple 12-question survey of five leadership domains: truthfulness, transparency, character, capability, and partnership. All supervisors were physicians and scientists. Their findings revealed that for each one-point increase in composite leadership score, there was a 3.3% decrease in the likelihood of burnout and a 9.0% increase in the likelihood of satisfaction in the physicians supervised.4

Interprofessional teamwork and engagement are vital skills for a leader to create a successful team. Enhanced management practices have also been associated with higher patient approval ratings and better financial performance. Effective leadership additionally affects physician well-being, with stronger leadership associated with less physician burnout and higher satisfaction.5

Leadership styles enhance quality measures in health care.6 The most effective leadership styles are ones in which the staff feels they are part of a team, are engaged, and are mentored.7 While leadership styles can vary, the common theme is staff engagement. An authoritative style leader is one who mobilizes the team toward a vision, that is, “Come with me.” An affiliative style leader creates harmony and builds emotional bonds where “people come first.” Democratic leaders forge a consensus through staff participation by asking, “What do you think?” Finally, a leader who uses a coaching style helps staff to identify their strengths and weaknesses and work toward improvement. These leadership behaviors are in contradistinction to the unsuccessful coercive leader who demands immediate compliance, that is, “Do what I tell you.”

Five fundamental leadership principles are shown in Table 1.8

Effective leaders have an open (growth) mindset, unwavering attention to diversity, equity, and inclusion, and to building relationships and trust; they practice effective communication and listening, focus on results, and cocreate support structures.

162629a_graphic_web.png

A growth mindset is the belief that one’s abilities are not innate but can improve through effort and learning.9

 

 

Emotional intelligence

A survey of business senior managers rated the qualities found in the most outstanding leaders. Using objective criteria like profitability the study psychologists interviewed the highest-rated leaders to compare their capabilities. While intellects and cognitive skills were important, the results showed that emotional intelligence (EI) was twice as important as technical skills and IQ.10 As an example, in a 1996 study, when senior managers had an optimal level of EI, their division’s yearly earnings were 20% higher than estimated.11

EI is a leadership competency that deals with the ability to understand and manage your own emotions and your interactions with others.10 At the Cleveland Clinic, EI is exemplified by the acronym HEART, whereby the team strives to improve the patient experience, mainly when an error occurs. The health care team is using EI by showing its the ability to Hear, Empathize, Apologize, Reply, and Thank. When an untoward event occurs, the physician, as the leader of the team, must lead by example when communicating with staff and patients. EI consists of five components (Table 2).13

162629b_graphic_web.png

  • Self-awareness is insight by which you can improve. Maintaining a journal of your daily thoughts may assist with this as well as simply pausing to pay attention during times of heightened emotions.
  • Self-regulation shows control, that is, behaving according to your values, and being accountable and calm when challenged.
  • Purpose, knowing your “why,” produces motivation and helps maintain optimism.
  • Empathy shows the ability to understand the emotions of other people.
  • Social skill is the ability to establish mutually rewarding relationships.

Given all the above benefits, it is no surprise that companies are actively trying use artificial intelligence to improve EI.12
 

Learning to be a leader

In medical school, students are expected to develop skills to handle and resolve conflicts, learn to share leadership, take mutual responsibility, and monitor their own performance.13 Although training of young physicians in leadership is not unprecedented, a systemic review revealed a lack of analytic studies to evaluate the effectiveness of the teaching methods.14 During undergraduate medical education, standard curricula and methods of instruction on leadership are not established, resulting in variable outcomes.

The Association of American Medical Colleges offers a curriculum, “Preparing Medical Students to Be Physician Leaders: A Leadership Training Program for Students Designed and Led by Students.”15 The objectives of this training are to help students identify their “personal style of leadership, recognize strengths and weaknesses, utilize effective communication strategies, appropriately delegate team member responsibilities, and provide constructive feedback to help improve team function.”
 

Take-home points

Following the completion of formal medical education, physicians are thrust into leadership roles. The key to being an effective leader is using EI to mentor the team and make staff feel connected to the team’s meaning and purpose, so they feel valued.

 

Dr. Trolice is director of The IVF Center in Winter Park, Fla., and professor of obstetrics and gynecology at the University of Central Florida, Orlando.

References

1. Carsen S and Xia C. McGill J Med. 2006 Jan;9(1):1-2.

2. Jardine D et al. J Grad Med Educ. 2015;7(2):307-9.

3. Institute of Medicine. Acad Emerg Med. July 2004;11(7):802-6.

4. Shanafelt TD et al. Mayo Clin Proc. April 2015;90(4):432-40.

5. Rotenstein LS et al. Harv Bus Rev. Oct. 17, 2018.

6. Sfantou SF. Healthcare 2017;5(4):73.


7. Goleman D. Harv Bus Rev. March-April 2000.

8. Collins-Nakai R. McGill J Med [Internet]. 2020 Dec. 1 [cited 2023 Mar. 28];9(1).

9. Dweck C. Harv Bus Rev. Jan. 13, 2016.


10. Goleman D. Harv Bus Rev. 1998 Nov-Dec;76(6):93-102..

11. Goleman D et al. Primal leadership: Realizing the power of emotional intelligence. Boston: Harvard Business School Publishing, 2002.12. Limon D and Plaster B. Harv Bus Rev. Jan. 25, 2022.

13. Chen T-Y. Tzu Chi Med J. Apr–Jun 2018;30(2):66-70.

14. Kumar B et al. BMC Med Educ. 2020;20:175.

15. Richards K et al. Med Ed Portal. Dec. 13 2019.

Publications
Publications
Topics
Article Type
Sections
Teambase XML
<?xml version="1.0" encoding="UTF-8"?>
<!--$RCSfile: InCopy_agile.xsl,v $ $Revision: 1.35 $-->
<!--$RCSfile: drupal.xsl,v $ $Revision: 1.7 $-->
<root generator="drupal.xsl" gversion="1.7"> <header> <fileName>162629</fileName> <TBEID>0C048F24.SIG</TBEID> <TBUniqueIdentifier>MD_0C048F24</TBUniqueIdentifier> <newsOrJournal>News</newsOrJournal> <publisherName>Frontline Medical Communications</publisherName> <storyname/> <articleType>353</articleType> <TBLocation>Published-All Pubs</TBLocation> <QCDate>20230330T175600</QCDate> <firstPublished>20230331T105226</firstPublished> <LastPublished>20230331T110236</LastPublished> <pubStatus qcode="stat:"/> <embargoDate/> <killDate/> <CMSDate>20230331T105226</CMSDate> <articleSource/> <facebookInfo/> <meetingNumber/> <byline>Mark P Trolice</byline> <bylineText>MARK P. TROLICE MD, MBA</bylineText> <bylineFull>MARK P. TROLICE MD, MBA</bylineFull> <bylineTitleText/> <USOrGlobal/> <wireDocType/> <newsDocType/> <journalDocType/> <linkLabel/> <pageRange/> <citation/> <quizID/> <indexIssueDate/> <itemClass qcode="ninat:text"/> <provider qcode="provider:imng"> <name>IMNG Medical Media</name> <rightsInfo> <copyrightHolder> <name>Frontline Medical News</name> </copyrightHolder> <copyrightNotice>Copyright (c) 2015 Frontline Medical News, a Frontline Medical Communications Inc. company. All rights reserved. This material may not be published, broadcast, copied, or otherwise reproduced or distributed without the prior written permission of Frontline Medical Communications Inc.</copyrightNotice> </rightsInfo> </provider> <abstract/> <metaDescription>Physicians are placed in positions of leadership by the medical team, by the community, and by society, particularly during times of crisis such as the COVID pa</metaDescription> <articlePDF/> <teaserImage>284599</teaserImage> <teaser>While all physicians are viewed as leaders, the concept of leadership is rarely, if ever, described or developed as a part of medical training.</teaser> <title>The physician as leader</title> <deck/> <disclaimer/> <AuthorList/> <articleURL/> <doi/> <pubMedID/> <publishXMLStatus/> <publishXMLVersion>4</publishXMLVersion> <useEISSN>0</useEISSN> <urgency/> <pubPubdateYear/> <pubPubdateMonth/> <pubPubdateDay/> <pubVolume/> <pubNumber/> <wireChannels/> <primaryCMSID/> <CMSIDs/> <keywords/> <seeAlsos/> <publications_g> <publicationData> <publicationCode>ob</publicationCode> <pubIssueName/> <pubArticleType/> <pubTopics/> <pubCategories/> <pubSections/> </publicationData> </publications_g> <publications> <term canonical="true">23</term> </publications> <sections> <term canonical="true">68807</term> </sections> <topics> <term canonical="true">38029</term> </topics> <links> <link> <itemClass qcode="ninat:picture"/> <altRep contenttype="image/jpeg">images/24010881.jpg</altRep> <description role="drol:caption">Dr. Mark P. Trolice</description> <description role="drol:credit">Fertility CARE</description> </link> <link> <itemClass qcode="ninat:picture"/> <altRep contenttype="image/png">images/24011b82.png</altRep> <description role="drol:caption"/> <description role="drol:credit"/> </link> <link> <itemClass qcode="ninat:picture"/> <altRep contenttype="image/png">images/24011b83.png</altRep> <description role="drol:caption"/> <description role="drol:credit"/> </link> </links> </header> <itemSet> <newsItem> <itemMeta> <itemRole>Main</itemRole> <itemClass>text</itemClass> <title>The physician as leader</title> <deck/> </itemMeta> <itemContent> <p>Physicians are placed in positions of leadership by the medical team, by the community, and by society, particularly during times of crisis such as the COVID pandemic. They are looked to by the media at times of health care news such as the overturning of <em>Roe v. Wad</em>e.<sup>1</sup> In a 2015 survey of resident physicians, two-thirds agreed that a formalized leadership curriculum would help them become better supervisors and clinicians.<sup>2</sup> While all physicians are viewed as leaders, the concept of leadership is rarely, if ever, described or developed as a part of medical training. This month’s column will provide insights into defining leadership as a physician in the medical and administrative settings.</p> <h2>Benefits of effective leadership</h2> <p>Physicians, whether they are clinicians, researchers, administrators, or teachers, are expected to oversee and engage their teams. A report by the Institute of Medicine recommended that academic health centers “develop leaders at all levels who can manage the organizational and system changes necessary to improve health through innovation in health professions education, patient care, and research.”<sup>3</sup> Hospitals with higher-rated management practices and more highly rated boards of directors have been shown to deliver higher-quality care and better clinical outcomes, including lower mortality. [[{"fid":"284599","view_mode":"medstat_image_flush_left","fields":{"format":"medstat_image_flush_left","field_file_image_alt_text[und][0][value]":"Dr. Trolice is director of The IVF Center in Winter Park, Fla., and professor of obstetrics and gynecology at the University of Central Florida, Orlando.","field_file_image_credit[und][0][value]":"Fertility CARE","field_file_image_caption[und][0][value]":"Dr. Mark P. Trolice"},"type":"media","attributes":{"class":"media-element file-medstat_image_flush_left"}}]]</p> <p>To illustrate, the clinicians at the Mayo Clinic annually rate their supervisors on a Leader Index, a simple 12-question survey of five leadership domains: truthfulness, transparency, character, capability, and partnership. All supervisors were physicians and scientists. Their findings revealed that for each one-point increase in composite leadership score, there was a 3.3% decrease in the likelihood of burnout and a 9.0% increase in the likelihood of satisfaction in the physicians supervised.<sup>4</sup> <br/><br/>Interprofessional teamwork and engagement are vital skills for a leader to create a successful team. Enhanced management practices have also been associated with higher patient approval ratings and better financial performance. Effective leadership additionally affects physician well-being, with stronger leadership associated with less physician burnout and higher satisfaction.<sup>5</sup><br/><br/>Leadership styles enhance quality measures in health care.<sup>6</sup> The most effective leadership styles are ones in which the staff feels they are part of a team, are engaged, and are mentored.<sup>7</sup> While leadership styles can vary, the common theme is staff engagement. An authoritative style leader is one who mobilizes the team toward a vision, that is, “Come with me.” An affiliative style leader creates harmony and builds emotional bonds where “people come first.” Democratic leaders forge a consensus through staff participation by asking, “What do you think?” Finally, a leader who uses a coaching style helps staff to identify their strengths and weaknesses and work toward improvement. These leadership behaviors are in contradistinction to the unsuccessful coercive leader who demands immediate compliance, that is, “Do what I tell you.” <br/><br/>Five fundamental leadership principles are shown in Table 1.<sup>8</sup> <br/><br/>Effective leaders have an open (growth) mindset, unwavering attention to diversity, equity, and inclusion, and to building relationships and trust; they practice effective communication and listening, focus on results, and cocreate support structures. [[{"fid":"294036","view_mode":"medstat_image_flush_right","fields":{"format":"medstat_image_flush_right","field_file_image_alt_text[und][0][value]":"Table 1. Five fundamental leadership principles","field_file_image_credit[und][0][value]":"","field_file_image_caption[und][0][value]":""},"type":"media","attributes":{"class":"media-element file-medstat_image_flush_right"}}]]A growth mindset is the belief that one’s abilities are not innate but can improve through effort and learning.<sup>9</sup></p> <h2>Emotional intelligence</h2> <p>A survey of business senior managers rated the qualities found in the most outstanding leaders. Using objective criteria like profitability the study psychologists interviewed the highest-rated leaders to compare their capabilities. While intellects and cognitive skills were important, the results showed that emotional intelligence (EI) was twice as important as technical skills and IQ.<sup>10</sup> As an example, in a 1996 study, when senior managers had an optimal level of EI, their division’s yearly earnings were 20% higher than estimated.<sup>11</sup></p> <p>EI is a leadership competency that deals with the ability to understand and manage your own emotions and your interactions with others.<sup>10</sup> At the Cleveland Clinic, EI is exemplified by the acronym HEART, whereby the team strives to improve the patient experience, mainly when an error occurs. The health care team is using EI by showing its the ability to Hear, Empathize, Apologize, Reply, and Thank. When an untoward event occurs, the physician, as the leader of the team, must lead by example when communicating with staff and patients. EI consists of five components (Table 2).<sup>13</sup> <br/><br/>[[{"fid":"294037","view_mode":"medstat_image_full_text","fields":{"format":"medstat_image_full_text","field_file_image_alt_text[und][0][value]":"Table 2. The five components of emotional intelligence at work","field_file_image_credit[und][0][value]":"","field_file_image_caption[und][0][value]":""},"type":"media","attributes":{"class":"media-element file-medstat_image_full_text"}}]]</p> <ul class="body"> <li>Self-awareness is insight by which you can improve. Maintaining a journal of your daily thoughts may assist with this as well as simply pausing to pay attention during times of heightened emotions. </li> <li>Self-regulation shows control, that is, behaving according to your values, and being accountable and calm when challenged. </li> <li>Purpose, knowing your “why,” produces motivation and helps maintain optimism.</li> <li>Empathy shows the ability to understand the emotions of other people. </li> <li>Social skill is the ability to establish mutually rewarding relationships. </li> </ul> <p>Given all the above benefits, it is no surprise that companies are actively trying use artificial intelligence to improve EI.<sup>12</sup> <br/><br/></p> <h2>Learning to be a leader</h2> <p>In medical school, students are expected to develop skills to handle and resolve conflicts, learn to share leadership, take mutual responsibility, and monitor their own performance.<sup>13</sup> Although training of young physicians in leadership is not unprecedented, a systemic review revealed a lack of analytic studies to evaluate the effectiveness of the teaching methods.<sup>14</sup> During undergraduate medical education, standard curricula and methods of instruction on leadership are not established, resulting in variable outcomes. </p> <p>The Association of American Medical Colleges offers a curriculum, “Preparing Medical Students to Be Physician Leaders: A Leadership Training Program for Students Designed and Led by Students.”<sup>15</sup> The objectives of this training are to help students identify their “personal style of leadership, recognize strengths and weaknesses, utilize effective communication strategies, appropriately delegate team member responsibilities, and provide constructive feedback to help improve team function.” <br/><br/></p> <h2>Take-home points</h2> <p>Following the completion of formal medical education, physicians are thrust into leadership roles. The key to being an effective leader is using EI to mentor the team and make staff feel connected to the team’s meaning and purpose, so they feel valued.<span class="end"/></p> <p> <em>Dr. Trolice is director of The IVF Center in Winter Park, Fla., and professor of obstetrics and gynecology at the University of Central Florida, Orlando. </em> </p> <h2>References</h2> <p>1. Carsen S and Xia C. <span class="Hyperlink"><a href="https://pubmed.ncbi.nlm.nih.gov/19529799/">McGill J Med. 2006 Jan;9(1):1-2</a></span>.<br/><br/><span class="Hyperlink">2. Jardine D et al. <a href="https://doi.org/10.4300/JGME-07-02-31">J Grad Med Educ. 2015;7(2):307-9</a>. <br/><br/></span><span class="Hyperlink">3. Institute of Medicine. <a href="https://doi.org/10.1197/j.aem.2004.02.007">Acad Emerg Med. July 2004;11(7):802-6</a>.<br/><br/>4. Shanafelt TD et al. <a href="https://doi.org/10.1016/j.mayocp.2015.01.012">Mayo Clin Proc. April 2015;90(4):432-40</a>. <br/><br/>5. Rotenstein LS et al. <a href="https://hbr.org/2018/10/why-doctors-need-leadership-training">Harv Bus Rev. Oct. 17, 2018</a>.<br/><br/>6. Sfantou SF. <a href="https://www.mdpi.com/2227-9032/5/4/73">Healthcare 2017;5(4):73</a>. <br/><br/></span>7. Goleman D. <span class="Hyperlink"><a href="&#13;&#13;https://hbr.org/2000/03/leadership-that-gets-results">Harv Bus Rev. March-April 2000</a></span>. <br/><br/><span class="Hyperlink">8. Collins-Nakai R.<a href="https://doi.org/10.26443/mjm.v9i1.497"> McGill J Med [Internet]. 2020 Dec. 1 [cited 2023 Mar. 28];9(1)</a>. <br/><br/>9. Dweck C. <a href="https://hbr.org/2016/01/what-having-a-growth-mindset-actually-means">Harv Bus Rev. Jan. 13, 2016.</a> </span><br/><br/>10. Goleman D. <span class="Hyperlink"><a href="https://hbr.org/2004/01/what-makes-a-leader">Harv Bus Rev. 1998 Nov-Dec;76(6):93-102.</a></span>.<br/><br/>11. Goleman D et al. Primal leadership: Realizing the power of emotional intelligence. <span class="Hyperlink"><a href="https://www.amazon.com/Primal-Leadership-Realizing-Emotional-Intelligence/dp/157851486X">Boston: Harvard Business School Publishing, 2002.</a></span><span class="Hyperlink">12. </span>Limon D and Plaster B.<span class="Hyperlink"><a href="https://hbr.org/2022/01/can-ai-teach-us-how-to-become-more-emotionally-intelligent"> Harv Bus Rev. Jan. 25, 2022</a>.<br/><br/>13. Chen T-Y. <a href="https://doi.org/10.4103/tcmj.tcmj_26_18">Tzu Chi Med J. Apr–Jun 2018;30(2):66-70</a>. <br/><br/>14. Kumar B et al. <a href="https://doi.org/10.1186/s12909-020-02089-2">BMC Med Educ. 2020;20:175</a>. <br/><br/>15. Richards K et al.<a href="https://doi.org/10.15766/mep_2374-8265.10863"> Med Ed Portal. Dec. 13 2019</a>. </span></p> </itemContent> </newsItem> <newsItem> <itemMeta> <itemRole>teaser</itemRole> <itemClass>text</itemClass> <title/> <deck/> </itemMeta> <itemContent> </itemContent> </newsItem> </itemSet></root>
Disallow All Ads
Content Gating
No Gating (article Unlocked/Free)
Alternative CME
Disqus Comments
Default
Use ProPublica
Hide sidebar & use full width
render the right sidebar.
Conference Recap Checkbox
Not Conference Recap
Clinical Edge
Display the Slideshow in this Article
Medscape Article
Display survey writer
Reuters content
Disable Inline Native ads
WebMD Article

Is there still a role for tubal surgery in the modern world of IVF?

Article Type
Changed
Fri, 02/17/2023 - 12:14

According to the Centers for Disease Control and Preventions, in 2019 2.1% of all infants born in the United States were conceived by assisted reproductive technology (ART). Now 45 years old, ART, namely in vitro fertilization (IVF), is offered in nearly 500 clinics in the United States, contributing to over 300,000 treatment cycles per year.

A tubal factor is responsible for 30% of female infertility and may involve proximal and/or distal tubal occlusion, irrespective of pelvic adhesions.1 Before the advent of IVF, the sole approach to the treatment of a tubal factor had been surgery. Given its success and minimal invasiveness, IVF is increasingly being offered to circumvent a tubal factor for infertility. This month we examine the utility of surgical treatment of tubal factor infertility. The options for fertility with a history of bilateral tubal ligation was covered in a prior Reproductive Rounds column.

Trolice_Mark _P_FLA_2022_web.jpg
Dr. Mark P. Trolice

Tubal disease and pelvic adhesions prevent the normal transport of the oocyte and sperm through the fallopian tube. The primary etiology of tubal factor infertility is pelvic inflammatory disease, mainly caused by chlamydia or gonorrhea. Other conditions that may interfere with tubal transport include severe endometriosis, adhesions from previous surgery, or nontubal infection (for example, appendicitis, inflammatory bowel disease), pelvic tuberculosis, and salpingitis isthmica nodosa (that is, diverticulosis of the fallopian tube).
 

Proximal tubal occlusion

During a hysterosalpingogram (HSG), transient uterine cornual spasm can result if a woman experiences significant uterine cramping, thereby resulting in a false-positive diagnosis of proximal tubal occlusion. When a repeat HSG is gently performed with slow instillation of contrast, uterine cramping is less likely, and the tubal patency rate is 60%. PTO may also result from plugs of mucus and amorphous debris, but this is not true occlusion.2 In cases with unilateral PTO, controlled ovarian hyperstimulation with intrauterine insemination has resulted in pregnancy rates similar to those in patients with unexplained infertility.3

Reconstructive surgery for bilateral PTO has limited effectiveness and the risk of subsequent ectopic pregnancy is as high as 20%.4 A more successful option is fluoroscopic tubal catheterization (FTC), an outpatient procedure performed in a radiology or infertility center. FTC uses a coaxial catheter system where the outer catheter is guided through the tubal ostium and an inner catheter is atraumatically advanced to overcome the blockage. This procedure is 85% successful for tubal patency with 50% of patients conceiving in the first 12 months; one-third of time the tubes reocclude. After the reestablishment of patency with FTC, the chance of achieving a live birth is 22% and the risk of ectopic pregnancy is 4%.5

Treatment of distal tubal occlusion – the hydrosalpinx

Surgery for treating tubal factor infertility is most successful in women with distal tubal obstruction (DTO), often caused by a hydrosalpinx. Fimbrioplasty is the lysis of fimbrial adhesions or dilatation of fimbrial strictures; the tube is patent, but there are adhesive bands that surround the terminal end with preserved tubal rugae. Gentle introduction of an alligator laparoscopic forceps into the tubal ostium followed by opening and withdrawal of the forceps helps to stretch the tube and release minor degrees of fimbrial agglutination.6

A hydrosalpinx is diagnosed by DTO with dilation and intraluminal fluid accumulation along with the reduction/loss of endothelial cilia. Left untreated, a hydrosalpinx can lead to a 50% reduction in IVF pregnancy rates.7 Tube-sparing treatment involves neosalpingostomy to create a new tubal opening. A nonsurgical approach, ultrasound-guided aspiration of hydrosalpinges, has not been shown to significantly increase the rate of clinical pregnancy. Efficacy for improving fertility is generally poor, but depends upon tubal wall thickness, ampullary dilation, presence of mucosal folds, percentage of ciliated cells in the fimbrial end, and peritubal adhesions.8

Evidence supports that laparoscopic salpingectomy in women with hydrosalpinges improves the outcomes of IVF treatment, compared with no surgical intervention.9 The improvement in pregnancy and live birth rates likely stems from the elimination of the retrograde flow of embryotoxic fluid that disrupts implantation. Endometrial receptivity markers (endometrial cell adhesion molecules, integrins, and HOXA10) have been shown to be reduced in the presence of hydrosalpinx.10 A small, randomized trial demonstrated that bipolar diathermy prior to IVF improved pregnancy outcomes.11 PTO was not more effective than salpingectomy. Conceptions, without IVF, have been reported following salpingectomy for unilateral hydrosalpinx.12

In a series including 434 patients with DTO who underwent laparoscopic fimbrioplasty (enlargement of the ostium) or neosalpingostomy (creation of a new ostium) by a single surgeon, 5-year actuarial delivery rates decreased as the severity of tubal occlusion increased; the ectopic rate was stable at approximately 15%.13 A prospective study reported that the relative increase in the pregnancy rate after salpingectomy was greatest in women with a large hydrosalpinx visible on ultrasound.14

Because of the possible risks of decreased ovarian reserve secondary to interruption of ovarian blood supply, salpingectomy should be done with minimal thermal injury and very close to the fallopian tube.

162178_graphic_web.png

 

Summary

Surgery may be considered for young women with mild distal tubal disease as one surgical procedure can lead to several pregnancies whereas IVF must be performed each time pregnancy is desired. IVF is more likely than surgery to be successful in women with bilateral hydrosalpinx, in those with pelvic adhesions, in older reproductive aged women, and for both proximal and distal tubal occlusion.15 An online prediction calculator from the Society for Assisted Reproductive Technology (SART) can be helpful in counseling patients on personalized expectations for IVF pregnancy outcomes.

Dr. Trolice is director of The IVF Center in Winter Park, Fla., and professor of obstetrics and gynecology at the University of Central Florida, Orlando.
 

References

1. Ambildhuke K et al. Cureus. 2022;1:14(11):e30990.

2. Fatemeh Z et al. Br J Radiol. 2021 Jun 1;94(1122):20201386.

3. Farhi J et al. Fertil Steril. 2007 Aug;88(2):396.

4. Honoré GM et al. Fertil Steril. 1999;71(5):785.

5. De Silva PM et al. Hum Reprod. 2017;32(4):836.

6. Namnoum A and Murphy A. “Diagnostic and Operative Laparoscopy,” in Te Linde’s Operative Gynecology, 8th ed. Philadelphia: Lippincott-Raven, 1997, pp. 389.

7. Camus E et al.Hum Reprod. 1999;14(5):1243.

8. Marana R et al. Hum Reprod. 1999;14(12):2991-5.

9. Johnson N et al. Cochrane Database Syst Rev. 2010 Jan 20;2010(1):CD002125.

10. Savaris RF et al. Fertil Steril. 2006 Jan;85(1):188.

11. Kontoravdis A et al. Fertil Steril. 2006;86(6):1642.

12. Sagoskin AW et al. Hum Reprod. 2003;18(12):2634.

13. Audebert A et al. Fertil Steril. 2014;102(4):1203.

14. Bildirici I et al. Hum Reprod. 2001;16(11):2422.

15. Practice Committee of the American Society for Reproductive Medicine. Fertil Steril. 2012;97(3):539.

Publications
Topics
Sections

According to the Centers for Disease Control and Preventions, in 2019 2.1% of all infants born in the United States were conceived by assisted reproductive technology (ART). Now 45 years old, ART, namely in vitro fertilization (IVF), is offered in nearly 500 clinics in the United States, contributing to over 300,000 treatment cycles per year.

A tubal factor is responsible for 30% of female infertility and may involve proximal and/or distal tubal occlusion, irrespective of pelvic adhesions.1 Before the advent of IVF, the sole approach to the treatment of a tubal factor had been surgery. Given its success and minimal invasiveness, IVF is increasingly being offered to circumvent a tubal factor for infertility. This month we examine the utility of surgical treatment of tubal factor infertility. The options for fertility with a history of bilateral tubal ligation was covered in a prior Reproductive Rounds column.

Trolice_Mark _P_FLA_2022_web.jpg
Dr. Mark P. Trolice

Tubal disease and pelvic adhesions prevent the normal transport of the oocyte and sperm through the fallopian tube. The primary etiology of tubal factor infertility is pelvic inflammatory disease, mainly caused by chlamydia or gonorrhea. Other conditions that may interfere with tubal transport include severe endometriosis, adhesions from previous surgery, or nontubal infection (for example, appendicitis, inflammatory bowel disease), pelvic tuberculosis, and salpingitis isthmica nodosa (that is, diverticulosis of the fallopian tube).
 

Proximal tubal occlusion

During a hysterosalpingogram (HSG), transient uterine cornual spasm can result if a woman experiences significant uterine cramping, thereby resulting in a false-positive diagnosis of proximal tubal occlusion. When a repeat HSG is gently performed with slow instillation of contrast, uterine cramping is less likely, and the tubal patency rate is 60%. PTO may also result from plugs of mucus and amorphous debris, but this is not true occlusion.2 In cases with unilateral PTO, controlled ovarian hyperstimulation with intrauterine insemination has resulted in pregnancy rates similar to those in patients with unexplained infertility.3

Reconstructive surgery for bilateral PTO has limited effectiveness and the risk of subsequent ectopic pregnancy is as high as 20%.4 A more successful option is fluoroscopic tubal catheterization (FTC), an outpatient procedure performed in a radiology or infertility center. FTC uses a coaxial catheter system where the outer catheter is guided through the tubal ostium and an inner catheter is atraumatically advanced to overcome the blockage. This procedure is 85% successful for tubal patency with 50% of patients conceiving in the first 12 months; one-third of time the tubes reocclude. After the reestablishment of patency with FTC, the chance of achieving a live birth is 22% and the risk of ectopic pregnancy is 4%.5

Treatment of distal tubal occlusion – the hydrosalpinx

Surgery for treating tubal factor infertility is most successful in women with distal tubal obstruction (DTO), often caused by a hydrosalpinx. Fimbrioplasty is the lysis of fimbrial adhesions or dilatation of fimbrial strictures; the tube is patent, but there are adhesive bands that surround the terminal end with preserved tubal rugae. Gentle introduction of an alligator laparoscopic forceps into the tubal ostium followed by opening and withdrawal of the forceps helps to stretch the tube and release minor degrees of fimbrial agglutination.6

A hydrosalpinx is diagnosed by DTO with dilation and intraluminal fluid accumulation along with the reduction/loss of endothelial cilia. Left untreated, a hydrosalpinx can lead to a 50% reduction in IVF pregnancy rates.7 Tube-sparing treatment involves neosalpingostomy to create a new tubal opening. A nonsurgical approach, ultrasound-guided aspiration of hydrosalpinges, has not been shown to significantly increase the rate of clinical pregnancy. Efficacy for improving fertility is generally poor, but depends upon tubal wall thickness, ampullary dilation, presence of mucosal folds, percentage of ciliated cells in the fimbrial end, and peritubal adhesions.8

Evidence supports that laparoscopic salpingectomy in women with hydrosalpinges improves the outcomes of IVF treatment, compared with no surgical intervention.9 The improvement in pregnancy and live birth rates likely stems from the elimination of the retrograde flow of embryotoxic fluid that disrupts implantation. Endometrial receptivity markers (endometrial cell adhesion molecules, integrins, and HOXA10) have been shown to be reduced in the presence of hydrosalpinx.10 A small, randomized trial demonstrated that bipolar diathermy prior to IVF improved pregnancy outcomes.11 PTO was not more effective than salpingectomy. Conceptions, without IVF, have been reported following salpingectomy for unilateral hydrosalpinx.12

In a series including 434 patients with DTO who underwent laparoscopic fimbrioplasty (enlargement of the ostium) or neosalpingostomy (creation of a new ostium) by a single surgeon, 5-year actuarial delivery rates decreased as the severity of tubal occlusion increased; the ectopic rate was stable at approximately 15%.13 A prospective study reported that the relative increase in the pregnancy rate after salpingectomy was greatest in women with a large hydrosalpinx visible on ultrasound.14

Because of the possible risks of decreased ovarian reserve secondary to interruption of ovarian blood supply, salpingectomy should be done with minimal thermal injury and very close to the fallopian tube.

162178_graphic_web.png

 

Summary

Surgery may be considered for young women with mild distal tubal disease as one surgical procedure can lead to several pregnancies whereas IVF must be performed each time pregnancy is desired. IVF is more likely than surgery to be successful in women with bilateral hydrosalpinx, in those with pelvic adhesions, in older reproductive aged women, and for both proximal and distal tubal occlusion.15 An online prediction calculator from the Society for Assisted Reproductive Technology (SART) can be helpful in counseling patients on personalized expectations for IVF pregnancy outcomes.

Dr. Trolice is director of The IVF Center in Winter Park, Fla., and professor of obstetrics and gynecology at the University of Central Florida, Orlando.
 

References

1. Ambildhuke K et al. Cureus. 2022;1:14(11):e30990.

2. Fatemeh Z et al. Br J Radiol. 2021 Jun 1;94(1122):20201386.

3. Farhi J et al. Fertil Steril. 2007 Aug;88(2):396.

4. Honoré GM et al. Fertil Steril. 1999;71(5):785.

5. De Silva PM et al. Hum Reprod. 2017;32(4):836.

6. Namnoum A and Murphy A. “Diagnostic and Operative Laparoscopy,” in Te Linde’s Operative Gynecology, 8th ed. Philadelphia: Lippincott-Raven, 1997, pp. 389.

7. Camus E et al.Hum Reprod. 1999;14(5):1243.

8. Marana R et al. Hum Reprod. 1999;14(12):2991-5.

9. Johnson N et al. Cochrane Database Syst Rev. 2010 Jan 20;2010(1):CD002125.

10. Savaris RF et al. Fertil Steril. 2006 Jan;85(1):188.

11. Kontoravdis A et al. Fertil Steril. 2006;86(6):1642.

12. Sagoskin AW et al. Hum Reprod. 2003;18(12):2634.

13. Audebert A et al. Fertil Steril. 2014;102(4):1203.

14. Bildirici I et al. Hum Reprod. 2001;16(11):2422.

15. Practice Committee of the American Society for Reproductive Medicine. Fertil Steril. 2012;97(3):539.

According to the Centers for Disease Control and Preventions, in 2019 2.1% of all infants born in the United States were conceived by assisted reproductive technology (ART). Now 45 years old, ART, namely in vitro fertilization (IVF), is offered in nearly 500 clinics in the United States, contributing to over 300,000 treatment cycles per year.

A tubal factor is responsible for 30% of female infertility and may involve proximal and/or distal tubal occlusion, irrespective of pelvic adhesions.1 Before the advent of IVF, the sole approach to the treatment of a tubal factor had been surgery. Given its success and minimal invasiveness, IVF is increasingly being offered to circumvent a tubal factor for infertility. This month we examine the utility of surgical treatment of tubal factor infertility. The options for fertility with a history of bilateral tubal ligation was covered in a prior Reproductive Rounds column.

Trolice_Mark _P_FLA_2022_web.jpg
Dr. Mark P. Trolice

Tubal disease and pelvic adhesions prevent the normal transport of the oocyte and sperm through the fallopian tube. The primary etiology of tubal factor infertility is pelvic inflammatory disease, mainly caused by chlamydia or gonorrhea. Other conditions that may interfere with tubal transport include severe endometriosis, adhesions from previous surgery, or nontubal infection (for example, appendicitis, inflammatory bowel disease), pelvic tuberculosis, and salpingitis isthmica nodosa (that is, diverticulosis of the fallopian tube).
 

Proximal tubal occlusion

During a hysterosalpingogram (HSG), transient uterine cornual spasm can result if a woman experiences significant uterine cramping, thereby resulting in a false-positive diagnosis of proximal tubal occlusion. When a repeat HSG is gently performed with slow instillation of contrast, uterine cramping is less likely, and the tubal patency rate is 60%. PTO may also result from plugs of mucus and amorphous debris, but this is not true occlusion.2 In cases with unilateral PTO, controlled ovarian hyperstimulation with intrauterine insemination has resulted in pregnancy rates similar to those in patients with unexplained infertility.3

Reconstructive surgery for bilateral PTO has limited effectiveness and the risk of subsequent ectopic pregnancy is as high as 20%.4 A more successful option is fluoroscopic tubal catheterization (FTC), an outpatient procedure performed in a radiology or infertility center. FTC uses a coaxial catheter system where the outer catheter is guided through the tubal ostium and an inner catheter is atraumatically advanced to overcome the blockage. This procedure is 85% successful for tubal patency with 50% of patients conceiving in the first 12 months; one-third of time the tubes reocclude. After the reestablishment of patency with FTC, the chance of achieving a live birth is 22% and the risk of ectopic pregnancy is 4%.5

Treatment of distal tubal occlusion – the hydrosalpinx

Surgery for treating tubal factor infertility is most successful in women with distal tubal obstruction (DTO), often caused by a hydrosalpinx. Fimbrioplasty is the lysis of fimbrial adhesions or dilatation of fimbrial strictures; the tube is patent, but there are adhesive bands that surround the terminal end with preserved tubal rugae. Gentle introduction of an alligator laparoscopic forceps into the tubal ostium followed by opening and withdrawal of the forceps helps to stretch the tube and release minor degrees of fimbrial agglutination.6

A hydrosalpinx is diagnosed by DTO with dilation and intraluminal fluid accumulation along with the reduction/loss of endothelial cilia. Left untreated, a hydrosalpinx can lead to a 50% reduction in IVF pregnancy rates.7 Tube-sparing treatment involves neosalpingostomy to create a new tubal opening. A nonsurgical approach, ultrasound-guided aspiration of hydrosalpinges, has not been shown to significantly increase the rate of clinical pregnancy. Efficacy for improving fertility is generally poor, but depends upon tubal wall thickness, ampullary dilation, presence of mucosal folds, percentage of ciliated cells in the fimbrial end, and peritubal adhesions.8

Evidence supports that laparoscopic salpingectomy in women with hydrosalpinges improves the outcomes of IVF treatment, compared with no surgical intervention.9 The improvement in pregnancy and live birth rates likely stems from the elimination of the retrograde flow of embryotoxic fluid that disrupts implantation. Endometrial receptivity markers (endometrial cell adhesion molecules, integrins, and HOXA10) have been shown to be reduced in the presence of hydrosalpinx.10 A small, randomized trial demonstrated that bipolar diathermy prior to IVF improved pregnancy outcomes.11 PTO was not more effective than salpingectomy. Conceptions, without IVF, have been reported following salpingectomy for unilateral hydrosalpinx.12

In a series including 434 patients with DTO who underwent laparoscopic fimbrioplasty (enlargement of the ostium) or neosalpingostomy (creation of a new ostium) by a single surgeon, 5-year actuarial delivery rates decreased as the severity of tubal occlusion increased; the ectopic rate was stable at approximately 15%.13 A prospective study reported that the relative increase in the pregnancy rate after salpingectomy was greatest in women with a large hydrosalpinx visible on ultrasound.14

Because of the possible risks of decreased ovarian reserve secondary to interruption of ovarian blood supply, salpingectomy should be done with minimal thermal injury and very close to the fallopian tube.

162178_graphic_web.png

 

Summary

Surgery may be considered for young women with mild distal tubal disease as one surgical procedure can lead to several pregnancies whereas IVF must be performed each time pregnancy is desired. IVF is more likely than surgery to be successful in women with bilateral hydrosalpinx, in those with pelvic adhesions, in older reproductive aged women, and for both proximal and distal tubal occlusion.15 An online prediction calculator from the Society for Assisted Reproductive Technology (SART) can be helpful in counseling patients on personalized expectations for IVF pregnancy outcomes.

Dr. Trolice is director of The IVF Center in Winter Park, Fla., and professor of obstetrics and gynecology at the University of Central Florida, Orlando.
 

References

1. Ambildhuke K et al. Cureus. 2022;1:14(11):e30990.

2. Fatemeh Z et al. Br J Radiol. 2021 Jun 1;94(1122):20201386.

3. Farhi J et al. Fertil Steril. 2007 Aug;88(2):396.

4. Honoré GM et al. Fertil Steril. 1999;71(5):785.

5. De Silva PM et al. Hum Reprod. 2017;32(4):836.

6. Namnoum A and Murphy A. “Diagnostic and Operative Laparoscopy,” in Te Linde’s Operative Gynecology, 8th ed. Philadelphia: Lippincott-Raven, 1997, pp. 389.

7. Camus E et al.Hum Reprod. 1999;14(5):1243.

8. Marana R et al. Hum Reprod. 1999;14(12):2991-5.

9. Johnson N et al. Cochrane Database Syst Rev. 2010 Jan 20;2010(1):CD002125.

10. Savaris RF et al. Fertil Steril. 2006 Jan;85(1):188.

11. Kontoravdis A et al. Fertil Steril. 2006;86(6):1642.

12. Sagoskin AW et al. Hum Reprod. 2003;18(12):2634.

13. Audebert A et al. Fertil Steril. 2014;102(4):1203.

14. Bildirici I et al. Hum Reprod. 2001;16(11):2422.

15. Practice Committee of the American Society for Reproductive Medicine. Fertil Steril. 2012;97(3):539.

Publications
Publications
Topics
Article Type
Sections
Teambase XML
<?xml version="1.0" encoding="UTF-8"?>
<!--$RCSfile: InCopy_agile.xsl,v $ $Revision: 1.35 $-->
<!--$RCSfile: drupal.xsl,v $ $Revision: 1.7 $-->
<root generator="drupal.xsl" gversion="1.7"> <header> <fileName>162178</fileName> <TBEID>0C0484D2.SIG</TBEID> <TBUniqueIdentifier>MD_0C0484D2</TBUniqueIdentifier> <newsOrJournal>News</newsOrJournal> <publisherName>Frontline Medical Communications</publisherName> <storyname/> <articleType>353</articleType> <TBLocation>QC Done-All Pubs</TBLocation> <QCDate>20230217T124310</QCDate> <firstPublished>20230217T120848</firstPublished> <LastPublished>20230217T120848</LastPublished> <pubStatus qcode="stat:"/> <embargoDate/> <killDate/> <CMSDate>20230217T120848</CMSDate> <articleSource/> <facebookInfo/> <meetingNumber/> <byline>Mark P Trolice</byline> <bylineText>MARK P. TROLICE, MD, MBA</bylineText> <bylineFull>MARK P. TROLICE, MD, MBA</bylineFull> <bylineTitleText/> <USOrGlobal/> <wireDocType/> <newsDocType/> <journalDocType/> <linkLabel/> <pageRange/> <citation/> <quizID/> <indexIssueDate/> <itemClass qcode="ninat:text"/> <provider qcode="provider:imng"> <name>IMNG Medical Media</name> <rightsInfo> <copyrightHolder> <name>Frontline Medical News</name> </copyrightHolder> <copyrightNotice>Copyright (c) 2015 Frontline Medical News, a Frontline Medical Communications Inc. company. All rights reserved. This material may not be published, broadcast, copied, or otherwise reproduced or distributed without the prior written permission of Frontline Medical Communications Inc.</copyrightNotice> </rightsInfo> </provider> <abstract/> <metaDescription>According to the Centers for Disease Control and Preventions, in 2019 2.1% of all infants born in the United States were conceived by assisted reproductive tech</metaDescription> <articlePDF/> <teaserImage>284599</teaserImage> <teaser>Surgery for treating tubal factor infertility is most successful in younger women with distal tubal obstruction, often caused by a hydrosalpinx. </teaser> <title>Is there still a role for tubal surgery in the modern world of IVF?</title> <deck/> <disclaimer/> <AuthorList/> <articleURL/> <doi/> <pubMedID/> <publishXMLStatus/> <publishXMLVersion>1</publishXMLVersion> <useEISSN>0</useEISSN> <urgency/> <pubPubdateYear/> <pubPubdateMonth/> <pubPubdateDay/> <pubVolume/> <pubNumber/> <wireChannels/> <primaryCMSID/> <CMSIDs/> <keywords/> <seeAlsos/> <publications_g> <publicationData> <publicationCode>mdsurg</publicationCode> <pubIssueName/> <pubArticleType/> <pubTopics/> <pubCategories/> <pubSections/> <journalTitle/> <journalFullTitle/> <copyrightStatement>2018 Frontline Medical Communications Inc.,</copyrightStatement> </publicationData> <publicationData> <publicationCode>ob</publicationCode> <pubIssueName/> <pubArticleType/> <pubTopics/> <pubCategories/> <pubSections/> </publicationData> </publications_g> <publications> <term>52226</term> <term canonical="true">23</term> </publications> <sections> <term>52</term> <term>41022</term> <term canonical="true">68807</term> </sections> <topics> <term>352</term> <term canonical="true">302</term> <term>28881</term> <term>218</term> </topics> <links> <link> <itemClass qcode="ninat:picture"/> <altRep contenttype="image/jpeg">images/24010881.jpg</altRep> <description role="drol:caption">Dr. Mark P. Trolice</description> <description role="drol:credit"/> </link> <link> <itemClass qcode="ninat:picture"/> <altRep contenttype="image/jpeg">images/24011935.jpg</altRep> <description role="drol:caption"/> <description role="drol:credit"/> </link> </links> </header> <itemSet> <newsItem> <itemMeta> <itemRole>Main</itemRole> <itemClass>text</itemClass> <title>Is there still a role for tubal surgery in the modern world of IVF?</title> <deck/> </itemMeta> <itemContent> <p>According to the Centers for Disease Control and Preventions, in 2019 2.1% of all infants born in the United States were conceived by assisted reproductive technology (ART). Now 45 years old, ART, namely in vitro fertilization (IVF), is offered in nearly 500 clinics in the United States, contributing to over 300,000 treatment cycles per year.</p> <p>A tubal factor is responsible for 30% of female infertility and may involve proximal and/or distal tubal occlusion, irrespective of pelvic adhesions.<sup>1</sup> Before the advent of IVF, the sole approach to the treatment of a tubal factor had been surgery. Given its success and minimal invasiveness, IVF is increasingly being offered to circumvent a tubal factor for infertility. This month we examine the utility of surgical treatment of tubal factor infertility. The options for fertility with a history of bilateral tubal ligation was covered in a <span class="Hyperlink"><a href="https://www.mdedge.com/obgyn/article/253241/surgery/fertility-after-tubal-ligation-its-matter-age">prior Reproductive Rounds column</a></span>.<br/><br/>[[{"fid":"284599","view_mode":"medstat_image_flush_left","fields":{"format":"medstat_image_flush_left","field_file_image_alt_text[und][0][value]":"Dr. Trolice is director of The IVF Center in Winter Park, Fla., and professor of obstetrics and gynecology at the University of Central Florida, Orlando.","field_file_image_credit[und][0][value]":"","field_file_image_caption[und][0][value]":"Dr. Mark P. Trolice"},"type":"media","attributes":{"class":"media-element file-medstat_image_flush_left"}}]]Tubal disease and pelvic adhesions prevent the normal transport of the oocyte and sperm through the fallopian tube. The primary etiology of tubal factor infertility is pelvic inflammatory disease, mainly caused by chlamydia or gonorrhea. Other conditions that may interfere with tubal transport include severe endometriosis, adhesions from previous surgery, or nontubal infection (for example, appendicitis, inflammatory bowel disease), pelvic tuberculosis, and salpingitis isthmica nodosa (that is, diverticulosis of the fallopian tube). <br/><br/></p> <h2>Proximal tubal occlusion</h2> <p>During a hysterosalpingogram (HSG), transient uterine cornual spasm can result if a woman experiences significant uterine cramping, thereby resulting in a false-positive diagnosis of proximal tubal occlusion. When a repeat HSG is gently performed with slow instillation of contrast, uterine cramping is less likely, and the tubal patency rate is 60%. PTO may also result from plugs of mucus and amorphous debris, but this is not true occlusion.<sup>2</sup> In cases with unilateral PTO, controlled ovarian hyperstimulation with intrauterine insemination has resulted in pregnancy rates similar to those in patients with unexplained infertility.<sup>3</sup></p> <p>Reconstructive surgery for bilateral PTO has limited effectiveness and the risk of subsequent ectopic pregnancy is as high as 20%.<sup>4</sup> A more successful option is fluoroscopic tubal catheterization (FTC), an outpatient procedure performed in a radiology or infertility center. FTC uses a coaxial catheter system where the outer catheter is guided through the tubal ostium and an inner catheter is atraumatically advanced to overcome the blockage. This procedure is 85% successful for tubal patency with 50% of patients conceiving in the first 12 months; one-third of time the tubes reocclude. After the reestablishment of patency with FTC, the chance of achieving a live birth is 22% and the risk of ectopic pregnancy is 4%.<sup>5</sup></p> <h2>Treatment of distal tubal occlusion – the hydrosalpinx</h2> <p>Surgery for treating tubal factor infertility is most successful in women with distal tubal obstruction (DTO), often caused by a hydrosalpinx. Fimbrioplasty is the lysis of fimbrial adhesions or dilatation of fimbrial strictures; the tube is patent, but there are adhesive bands that surround the terminal end with preserved tubal rugae. Gentle introduction of an alligator laparoscopic forceps into the tubal ostium followed by opening and withdrawal of the forceps helps to stretch the tube and release minor degrees of fimbrial agglutination.<sup>6</sup></p> <p>A hydrosalpinx is diagnosed by DTO with dilation and intraluminal fluid accumulation along with the reduction/loss of endothelial cilia. Left untreated, a hydrosalpinx can lead to a 50% reduction in IVF pregnancy rates.<sup>7</sup> Tube-sparing treatment involves neosalpingostomy to create a new tubal opening. A nonsurgical approach, ultrasound-guided aspiration of hydrosalpinges, has not been shown to significantly increase the rate of clinical pregnancy. Efficacy for improving fertility is generally poor, but depends upon tubal wall thickness, ampullary dilation, presence of mucosal folds, percentage of ciliated cells in the fimbrial end, and peritubal adhesions.<sup>8</sup><br/><br/>Evidence supports that laparoscopic salpingectomy in women with hydrosalpinges improves the outcomes of IVF treatment, compared with no surgical intervention.<sup>9</sup> The improvement in pregnancy and live birth rates likely stems from the elimination of the retrograde flow of embryotoxic fluid that disrupts implantation. Endometrial receptivity markers (endometrial cell adhesion molecules, integrins, and HOXA10) have been shown to be reduced in the presence of hydrosalpinx.<sup>10</sup> A small, randomized trial demonstrated that bipolar diathermy prior to IVF improved pregnancy outcomes.<sup>11</sup> PTO was not more effective than salpingectomy. Conceptions, without IVF, have been reported following salpingectomy for unilateral hydrosalpinx.<sup>12</sup><br/><br/>In a series including 434 patients with DTO who underwent laparoscopic fimbrioplasty (enlargement of the ostium) or neosalpingostomy (creation of a new ostium) by a single surgeon, 5-year actuarial delivery rates decreased as the severity of tubal occlusion increased; the ectopic rate was stable at approximately 15%.<sup>13</sup> A prospective study reported that the relative increase in the pregnancy rate after salpingectomy was greatest in women with a large hydrosalpinx visible on ultrasound.<sup>14</sup><br/><br/>Because of the possible risks of decreased ovarian reserve secondary to interruption of ovarian blood supply, salpingectomy should be done with minimal thermal injury and very close to the fallopian tube.<br/><br/>[[{"fid":"293032","view_mode":"medstat_image_full_text","fields":{"format":"medstat_image_full_text","field_file_image_alt_text[und][0][value]":"Pros and cons of in vitro fertilization over tubal surgery","field_file_image_credit[und][0][value]":"","field_file_image_caption[und][0][value]":""},"type":"media","attributes":{"class":"media-element file-medstat_image_full_text"}}]]<br/><br/></p> <h2>Summary</h2> <p>Surgery may be considered for young women with mild distal tubal disease as one surgical procedure can lead to several pregnancies whereas IVF must be performed each time pregnancy is desired. IVF is more likely than surgery to be successful in women with bilateral hydrosalpinx, in those with pelvic adhesions, in older reproductive aged women, and for both proximal and distal tubal occlusion.<sup>15</sup> An online <span class="Hyperlink"><a href="https://w3.abdn.ac.uk/clsm/SARTIVF/">prediction calculator</a></span> from the <span class="Hyperlink"><a href="http://sart.org">Society for Assisted Reproductive Technology (SART)</a></span> can be helpful in counseling patients on personalized expectations for IVF pregnancy outcomes. </p> <p> <em>Dr. Trolice is director of The IVF Center in Winter Park, Fla., and professor of obstetrics and gynecology at the University of Central Florida, Orlando. <br/><br/></em> </p> <h2>References</h2> <p>1. Ambildhuke K et al. <span class="Hyperlink"><a href="https://www.cureus.com/articles/121570-a-review-of-tubal-factors-affecting-fertility-and-its-management#!">Cureus. 2022;1:14(11):e30990</a></span>.<br/><br/>2. Fatemeh Z et al. <span class="Hyperlink"><a href="https://www.birpublications.org/doi/10.1259/bjr.20201386">Br J Radiol. 2021 Jun 1;94(1122):20201386</a></span>.<br/><br/>3. Farhi J et al. <span class="Hyperlink"><a href="https://www.fertstert.org/article/S0015-0282(06)04659-0/fulltext">Fertil Steril. 2007 Aug;88(2):396</a></span>.<br/><br/>4. Honoré GM et al. <span class="Hyperlink"><a href="https://www.fertstert.org/article/S0015-0282(99)00014-X/fulltext">Fertil Steril. 1999;71(5):785</a></span>.<br/><br/>5. De Silva PM et al. <span class="Hyperlink"><a href="https://academic.oup.com/humrep/article/32/4/836/2981924?login=false">Hum Reprod. 2017;32(4):836</a></span>.<br/><br/>6. Namnoum A and Murphy A. “Diagnostic and Operative Laparoscopy,” in <span class="Hyperlink"><a href="https://obgyn.lwwhealthlibrary.com/book.aspx?bookid=2677">Te Linde’s Operative Gynecology, 8th ed. Philadelphia: Lippincott-Raven, 1997, pp. 389</a></span>.<br/><br/>7. Camus E et al.<span class="Hyperlink"><a href="https://academic.oup.com/humrep/article/14/5/1243/766154?login=false">Hum Reprod. 1999;14(5):1243</a></span>.<br/><br/>8. Marana R et al. <span class="Hyperlink"><a href="https://academic.oup.com/humrep/article/14/12/2991/2913066?login=false">Hum Reprod. 1999;14(12):299</a>1-5</span>.<br/><br/>9. Johnson N et al. <span class="Hyperlink"><a href="https://www.cochranelibrary.com/cdsr/doi/10.1002/14651858.CD002125.pub3/full">Cochrane Database Syst Rev. 2010 Jan 20;2010(1):CD002125</a></span>.<br/><br/>10. Savaris RF et al. <span class="Hyperlink"><a href="https://www.fertstert.org/article/S0015-0282(05)03405-9/fulltext">Fertil Steril. 2006 Jan;85(1):188</a>.</span><br/><br/>11. Kontoravdis A et al. <span class="Hyperlink"><a href="https://www.fertstert.org/article/S0015-0282(06)03038-X/fulltext">Fertil Steril. 2006;86(6):1642</a>.</span><br/><br/>12. Sagoskin AW et al. <span class="Hyperlink"><a href="https://academic.oup.com/humrep/article/18/12/2634/2913534?login=false">Hum Reprod. 2003;18(12):2634</a>.</span><br/><br/>13. Audebert A et al. <span class="Hyperlink"><a href="https://www.fertstert.org/article/S0015-0282(14)00593-7/fulltext">Fertil Steril. 2014;102(4):1203</a></span>.<br/><br/>14. Bildirici I et al. <span class="Hyperlink"><a href="https://academic.oup.com/humrep/article/16/11/2422/681162?login=false"> Hum Reprod. 2001;16(11):2422</a>.</span><br/><br/>15. Practice Committee of the American Society for Reproductive Medicine. <span class="Hyperlink"><a href="https://www.fertstert.org/article/S0015-0282(11)02915-3/fulltext">Fertil Steril. 2012;97(3):539</a></span>.</p> </itemContent> </newsItem> <newsItem> <itemMeta> <itemRole>teaser</itemRole> <itemClass>text</itemClass> <title/> <deck/> </itemMeta> <itemContent> </itemContent> </newsItem> </itemSet></root>
Disallow All Ads
Content Gating
No Gating (article Unlocked/Free)
Alternative CME
Disqus Comments
Default
Use ProPublica
Hide sidebar & use full width
render the right sidebar.
Conference Recap Checkbox
Not Conference Recap
Clinical Edge
Display the Slideshow in this Article
Medscape Article
Display survey writer
Reuters content
Disable Inline Native ads
WebMD Article

Advances in fertility preservation: Q & A

Article Type
Changed
Tue, 01/24/2023 - 10:05

From the first obscure reference until the 19th century, the maternal mortality rate from an ectopic pregnancy was nearly 100%. In the past 140 years, because of early detection and prompt surgical management, the mortality rate from an ectopic pregnancy declined from 72%-90% in 1880 to 0.48% from 2004 to 2008.1 Given this remarkable reduction in mortality, the 20th-century approach to ectopic pregnancy evolved from preserving the life of the mother to preserving fertility by utilizing conservative treatment with methotrexate and/or tubal surgery.

Why the reference to ectopic pregnancy? Advances in oncology have comparably affected our approach to cancer patients. The increase in survival rates following a cancer diagnosis has fostered revolutionary developments in fertility preservation to obviate the effect of gonadotoxic therapy. We have evolved from shielding and transposing ovaries to ovarian tissue cryopreservation2,3 with rapid implementation.

Trolice_Mark_P_FLA_new_web.jpg
Dr. Mark P. Trolice

One of the leaders in the field of female fertility preservation is Kutluk Oktay, MD, of Yale University, New Haven, Conn. I posed the following salient questions to him on the state of fertility preservation as well as expectations for the future.

Oktay_Kutluk_CN_web.jpg
Dr. Kutluk Oktay

 

Q1. What medication/treatment is gonadotoxic that warrants a consultation for fertility preservation?

A: While new drugs for cancer treatment continue to be approved and require testing for gonadotoxicity, evidence is clear on the damaging effects of alkylating agents such as cyclophosphamide, ifosfamide, chlorambucil, and melphalan on primordial follicle reserve.4 A useful tool to determine the risk of alkylating agents affecting fertility is the Cyclophosphamide Equivalent Dose (CED) Calculator. Likewise, topoisomerase inhibitors, such as doxorubicin4 induce ovarian reserve damage by causing double-strand DNA breaks (DSBs) in oocytes.5-7 Contrary to common belief, chemotherapy exposure suppresses the mechanisms that can initiate follicle growth.6 When DSBs occur, some oocytes may be able to repair such damage, otherwise apoptosis is triggered, which results in irreversible ovarian reserve loss.7 Younger individuals have much higher repair capacity, the magnitude of damage can be hard to predict, and it is variable.8,9 So, prior exposure to gonadotoxic drugs does not preclude consideration of fertility preservation.10

In addition, pelvic radiation, in a dose-dependent manner, causes severe DSBs and triggers the same cell suicide mechanisms while also potentially damaging uterine function. Additional information can be found in the American Society of Clinical Oncology Fertility Preservation Guidelines.4
 

Q2. What are the current options for fertility preservation in patients who will be exposed to gonadotoxic medication/treatment?

A: The current fertility preservation options for female patients faced with gonadotoxic treatments are embryo, oocyte, and ovarian tissue cryopreservation (OTC). Selection of fertility preservation is typically contingent upon the timetable of treatment. Oocyte and embryo cryopreservation have been the standard of care. Recently, OTC had its experimental designation removed by American Society for Reproductive Medicine11 with the advantage of not requiring ovarian stimulation or sexual maturity; and it may to be performed while patients are receiving chemotherapy. If successful, OTC followed by orthotopic transplantation has the potential to restore natural ovarian function, thereby allowing spontaneous conception.10 Especially in young adults, ovarian reserve loss is fractional and can remain at reasonable levels after a few courses of chemotherapy. Ovarian stimulation is risky after the initiation of chemotherapy because of the severe DNA damage to oocytes of developing follicles and the associated poor response.7 Hence, ovarian stimulation should be initiated and completed before the initiation of chemotherapy.

 

 

Q3. How successful are the approved fertility preservation options in obtaining oocytes for future utilization by ART?

A:
We have decades of experience with embryo cryopreservation and proven success rates that patients can check on the SART.org website for individual clinics. For oocyte cryopreservation, models are used to provide calculation estimates because the technique is less established.12 Although success rates are approaching those with fresh oocytes, they are still not equal.13 OTC followed by orthotopic tissue transplantation has the least outcomes data (approximately 200 reported livebirths to date with a 25% live birth rate per recipient worldwide10 since the first success was reported in 2000.2,14

With our robotic surgical approach to orthotopic and heterotopic ovarian tissue transplantation and the utility of neovascularizing agents, we have found that ovarian graft longevity is extended. Oocytes/embryos can be obtained and has resulted in one to two livebirths in all our recipients to date.10 Unfortunately, if any of the critical steps are not up to standards (freezing, thawing, or transplantation), success rates can dramatically decline. Therefore, providers and patients should seek centers with experience in all three stages of this procedure to maximize outcomes.
 

Q4. Are there concerns of increasing recurrence/mortality with fertility preservation given hormonal exposure?

A: Yes, this concern exists, at least in theory for estrogen-sensitive cancers, most commonly breast cancer. We developed ovarian stimulation protocols supplemented with anti-estrogen treatments (tamoxifen, an estrogen-receptor antagonist, and letrozole, an aromatase inhibitor) that appear equally effective and reduce estrogen exposure in any susceptible cancer.15,16 Even in estrogen receptor–negative tumors, high estrogen exposure may activate non–estrogen receptor–dependent pathways. In addition, even those tumors that are practically deemed estrogen receptor negative may still contain a small percentage of estrogen receptors, which may become active at high estrogen levels.

Therefore, when we approach women with estrogen-sensitive cancers, e.g., breast and endometrial, we do not alter our approach based on receptor status. One exception occurs in women with BRCA mutations, especially the BRCA1, as they have 25% lower serum anti-müllerian hormone (AMH) levels,8,17 yield fewer oocytes in response to ovarian stimulation,18,19 and have lower fertilization rates and embryo numbers20 compared with those without the mutations.
 

Q5. Are all reproductive centers capable of offering fertility preservation? If not, how does a patient find a center?

A: All IVF clinics offer embryo and, presumably, oocyte cryopreservation. Pregnancy outcomes vary based on the center’s experience. Globally, major differences exist in the availability and competency of OTC along with the subsequent transplantation approach. A limited number of centers have competency in all aspects of OTC, i.e., cryopreservation, thawing, and transplantation. In general, fertility preservation patients have a multitude of medical issues that necessitate management expertise and the bandwidth to coordinate with cancer health professionals. The reproductive centers offering fertility preservation should be prepared to respond immediately and accommodate patients about to undergo gonadotoxic treatment.
 

Q6. How should a patient be counseled before proceeding with fertility preservation?

A:
The candidate should be counseled on the likelihood of damage from gonadotoxic therapy and all fertility preservation options, on the basis of the urgency of treatment and the woman’s long-term goals. For example, the desire for a large family may compel a patient to undergo multiple cycles of ovarian stimulation or a combination of oocyte/embryo cryopreservation with OTC. In patients who are undergoing embryo cryopreservation, I recommend preimplantation genetic testing for aneuploidies, although there are limitations to its application. Other novel pieces of information we are using in counseling are baseline AMH levels and BRCA mutation status for women with breast cancer. In an 8-year-long NIH-funded prospective longitudinal study we found that women with both baseline AMH < 2 ng/mL and BRCA mutations are at significantly higher risk of losing their ovarian reserve and developing amenorrhea.21 Because the oocytes of women with BRCA mutations are deficient in DNA repair as we have previously shown,19 they are more liable to death upon exposure to DNA-damaging cancer drugs such as cyclophosphamide and doxorubicin.22

 

 

Q7. What is the time limit for use of cryopreserved oocytes/tissue?

A: Under optimal storage conditions, cryopreserved oocytes/tissue can be utilized indefinitely without a negative effect on pregnancy outcomes.
 

Q8. What does the future hold for fertility preservation?

A: The future holds promise for both the medical and nonmedical (planned) utility of fertility preservation. With the former, we will see that the utility of OTC and orthotopic and heterotopic tissue transplantation increase as success rates improve. Improved neovascularizing agents will make the transplants last longer and enhance pregnancy outcomes.23,24 I see planned fertility preservation increasing, based on the experience gained from cancer patients and some preliminary experience with planned OTC, especially for healthy women who wish to consider delaying menopause.25,26

Because of attrition from apoptosis, approximately 2,000 oocytes are wasted per ovulation. Through calculation models, we predict that if an equivalent of one-third of a woman’s ovarian cortex can be cryopreserved (which may not significantly affect the age at natural menopause) before age 40 years, transplantation at perimenopause may provide sufficient primordial follicles to delay menopause for 5 years or longer.26 Because ovarian tissue can also be transplanted subcutaneously under local anesthesia, as we have shown,27,28 repeated heterotopic transplants can be performed in an office setting at reduced cost, invasiveness, and with enhanced effectiveness. We can expect increasing reports and progress on this planned use of OTC and transplantation in the future. 
 

Dr. Oktay is professor of obstetrics & gynecology and reproductive sciences and director of the Laboratory of Molecular Reproduction and Fertility Preservation at Yale University, New Haven, Conn. Dr. Trolice is director of The IVF Center in Winter Park, Fla., and professor of obstetrics and gynecology at the University of Central Florida, Orlando.

References

1. Lurie S. Eur J Obstet Gynecol Reprod Biol. 1992 Jan 9;43(1):1-7.

2. Oktay K and Karlikaya G. N Engl J Med. 2000 Jun 22;342(25):1919.

3. Sonmezer and Oktay K. Hum Reprod Update. 2004;10(3):251-66.

4. Oktay K et al. J Clin Oncol. 2018 Jul 1;36(19):1994-2001.

5. Goldfarb SB et al. Breast Cancer Res Treat. 2021;185:165-73.

6. Titus S et al. Sci Rep. 2021 Jan 11;11(1):407.

7. Soleimani R et al. Aging (Albany NY). 2011 Aug;3(8):782-93.

8. Titus S et al. Sci Transl Med. 2013 Feb 13;5(172):172ra21.

9. Oktay KH et al. Fertil Steril. 2022 Jan 5:S0015-0282(21)02293-7.

10. Oktay K et al. Fertil Steril. 2022;117(1):181-92.

11. Practice Committee of the American Society for Reproductive Medicine. Fertil Steril. 2019;112(6):1022–33.

12. Cil A et al. Fertil Steril. 2013 Aug;100(2):492-9.e3.

13. Goldman KN et al. Fertil Steril. 2013 Sep;100(3):712-7.

14. Marin L and Oktay K. Scientific history of ovarian tissue cryopreservation and transplantation. In: Oktay K (ed.), Principles and Practice of Ovarian Tissue Cryopreservation and Transplantation. Elsevier;2022:1-10.

15. Oktay K et al. J Clin Oncol. 2005 Jul 1;23(19):4347-53.

16. Kim JY et al. J Clin Endocrinol Metab. 2016 Apr;101(4):1364-71.

17. Turan V et al. J Clin Oncol. 2021;39:18.

18. Oktay K et al. J Clin Oncol. 2010 Jan 10;28(2):240-4.

19. Lin W et al. J Clin Endocrinol Metab. 2017;102(10):3839-47.

20. Turan V et al. Reprod Sci. 2018;(25):26-32.

21. Oktay K et al. Presence of BRCA mutations and a pre-chemotherapy AMH level of < 2ng/mL strongly predict risk of amenorrhea in women with breast cancer P-291. Presented at the American Society for Reproductive Medicine 78th annual meeting, Anaheim, Calif. Oct. 22-26, 2022.

22. Oktay KH et al. Fertil Steril. 2020;113(6):1251‐60.e1.

23. Soleimani R et al. PLoS One. 2011 Apr 29;6(4):e19475.

24. Marin L et al. Future aspects of ovarian cryopreservation and transplantation. In: Oktay K (ed.). Principles and Practice of Ovarian Tissue Cryopreservation and Transplantation. Elsevier; 2022;223-30.

25. Oktay KH et al. Trends Mol Med. 2021;27(8):753-61.

26. Oktay K and Marin L. Ovarian tissue cryopreservation for delaying childbearing and menopause. In: Oktay, K. (Ed.), Principles and Practice of Ovarian Tissue Cryopreservation and Transplantation. Elsevier;2022:195-204.

27. Oktay K et al. JAMA. 2001 Sep 26;286(12):1490-3.

28. Oktay K et al. Lancet. 2004 Mar 13;363(9412):837-40.

Publications
Topics
Sections

From the first obscure reference until the 19th century, the maternal mortality rate from an ectopic pregnancy was nearly 100%. In the past 140 years, because of early detection and prompt surgical management, the mortality rate from an ectopic pregnancy declined from 72%-90% in 1880 to 0.48% from 2004 to 2008.1 Given this remarkable reduction in mortality, the 20th-century approach to ectopic pregnancy evolved from preserving the life of the mother to preserving fertility by utilizing conservative treatment with methotrexate and/or tubal surgery.

Why the reference to ectopic pregnancy? Advances in oncology have comparably affected our approach to cancer patients. The increase in survival rates following a cancer diagnosis has fostered revolutionary developments in fertility preservation to obviate the effect of gonadotoxic therapy. We have evolved from shielding and transposing ovaries to ovarian tissue cryopreservation2,3 with rapid implementation.

Trolice_Mark_P_FLA_new_web.jpg
Dr. Mark P. Trolice

One of the leaders in the field of female fertility preservation is Kutluk Oktay, MD, of Yale University, New Haven, Conn. I posed the following salient questions to him on the state of fertility preservation as well as expectations for the future.

Oktay_Kutluk_CN_web.jpg
Dr. Kutluk Oktay

 

Q1. What medication/treatment is gonadotoxic that warrants a consultation for fertility preservation?

A: While new drugs for cancer treatment continue to be approved and require testing for gonadotoxicity, evidence is clear on the damaging effects of alkylating agents such as cyclophosphamide, ifosfamide, chlorambucil, and melphalan on primordial follicle reserve.4 A useful tool to determine the risk of alkylating agents affecting fertility is the Cyclophosphamide Equivalent Dose (CED) Calculator. Likewise, topoisomerase inhibitors, such as doxorubicin4 induce ovarian reserve damage by causing double-strand DNA breaks (DSBs) in oocytes.5-7 Contrary to common belief, chemotherapy exposure suppresses the mechanisms that can initiate follicle growth.6 When DSBs occur, some oocytes may be able to repair such damage, otherwise apoptosis is triggered, which results in irreversible ovarian reserve loss.7 Younger individuals have much higher repair capacity, the magnitude of damage can be hard to predict, and it is variable.8,9 So, prior exposure to gonadotoxic drugs does not preclude consideration of fertility preservation.10

In addition, pelvic radiation, in a dose-dependent manner, causes severe DSBs and triggers the same cell suicide mechanisms while also potentially damaging uterine function. Additional information can be found in the American Society of Clinical Oncology Fertility Preservation Guidelines.4
 

Q2. What are the current options for fertility preservation in patients who will be exposed to gonadotoxic medication/treatment?

A: The current fertility preservation options for female patients faced with gonadotoxic treatments are embryo, oocyte, and ovarian tissue cryopreservation (OTC). Selection of fertility preservation is typically contingent upon the timetable of treatment. Oocyte and embryo cryopreservation have been the standard of care. Recently, OTC had its experimental designation removed by American Society for Reproductive Medicine11 with the advantage of not requiring ovarian stimulation or sexual maturity; and it may to be performed while patients are receiving chemotherapy. If successful, OTC followed by orthotopic transplantation has the potential to restore natural ovarian function, thereby allowing spontaneous conception.10 Especially in young adults, ovarian reserve loss is fractional and can remain at reasonable levels after a few courses of chemotherapy. Ovarian stimulation is risky after the initiation of chemotherapy because of the severe DNA damage to oocytes of developing follicles and the associated poor response.7 Hence, ovarian stimulation should be initiated and completed before the initiation of chemotherapy.

 

 

Q3. How successful are the approved fertility preservation options in obtaining oocytes for future utilization by ART?

A:
We have decades of experience with embryo cryopreservation and proven success rates that patients can check on the SART.org website for individual clinics. For oocyte cryopreservation, models are used to provide calculation estimates because the technique is less established.12 Although success rates are approaching those with fresh oocytes, they are still not equal.13 OTC followed by orthotopic tissue transplantation has the least outcomes data (approximately 200 reported livebirths to date with a 25% live birth rate per recipient worldwide10 since the first success was reported in 2000.2,14

With our robotic surgical approach to orthotopic and heterotopic ovarian tissue transplantation and the utility of neovascularizing agents, we have found that ovarian graft longevity is extended. Oocytes/embryos can be obtained and has resulted in one to two livebirths in all our recipients to date.10 Unfortunately, if any of the critical steps are not up to standards (freezing, thawing, or transplantation), success rates can dramatically decline. Therefore, providers and patients should seek centers with experience in all three stages of this procedure to maximize outcomes.
 

Q4. Are there concerns of increasing recurrence/mortality with fertility preservation given hormonal exposure?

A: Yes, this concern exists, at least in theory for estrogen-sensitive cancers, most commonly breast cancer. We developed ovarian stimulation protocols supplemented with anti-estrogen treatments (tamoxifen, an estrogen-receptor antagonist, and letrozole, an aromatase inhibitor) that appear equally effective and reduce estrogen exposure in any susceptible cancer.15,16 Even in estrogen receptor–negative tumors, high estrogen exposure may activate non–estrogen receptor–dependent pathways. In addition, even those tumors that are practically deemed estrogen receptor negative may still contain a small percentage of estrogen receptors, which may become active at high estrogen levels.

Therefore, when we approach women with estrogen-sensitive cancers, e.g., breast and endometrial, we do not alter our approach based on receptor status. One exception occurs in women with BRCA mutations, especially the BRCA1, as they have 25% lower serum anti-müllerian hormone (AMH) levels,8,17 yield fewer oocytes in response to ovarian stimulation,18,19 and have lower fertilization rates and embryo numbers20 compared with those without the mutations.
 

Q5. Are all reproductive centers capable of offering fertility preservation? If not, how does a patient find a center?

A: All IVF clinics offer embryo and, presumably, oocyte cryopreservation. Pregnancy outcomes vary based on the center’s experience. Globally, major differences exist in the availability and competency of OTC along with the subsequent transplantation approach. A limited number of centers have competency in all aspects of OTC, i.e., cryopreservation, thawing, and transplantation. In general, fertility preservation patients have a multitude of medical issues that necessitate management expertise and the bandwidth to coordinate with cancer health professionals. The reproductive centers offering fertility preservation should be prepared to respond immediately and accommodate patients about to undergo gonadotoxic treatment.
 

Q6. How should a patient be counseled before proceeding with fertility preservation?

A:
The candidate should be counseled on the likelihood of damage from gonadotoxic therapy and all fertility preservation options, on the basis of the urgency of treatment and the woman’s long-term goals. For example, the desire for a large family may compel a patient to undergo multiple cycles of ovarian stimulation or a combination of oocyte/embryo cryopreservation with OTC. In patients who are undergoing embryo cryopreservation, I recommend preimplantation genetic testing for aneuploidies, although there are limitations to its application. Other novel pieces of information we are using in counseling are baseline AMH levels and BRCA mutation status for women with breast cancer. In an 8-year-long NIH-funded prospective longitudinal study we found that women with both baseline AMH < 2 ng/mL and BRCA mutations are at significantly higher risk of losing their ovarian reserve and developing amenorrhea.21 Because the oocytes of women with BRCA mutations are deficient in DNA repair as we have previously shown,19 they are more liable to death upon exposure to DNA-damaging cancer drugs such as cyclophosphamide and doxorubicin.22

 

 

Q7. What is the time limit for use of cryopreserved oocytes/tissue?

A: Under optimal storage conditions, cryopreserved oocytes/tissue can be utilized indefinitely without a negative effect on pregnancy outcomes.
 

Q8. What does the future hold for fertility preservation?

A: The future holds promise for both the medical and nonmedical (planned) utility of fertility preservation. With the former, we will see that the utility of OTC and orthotopic and heterotopic tissue transplantation increase as success rates improve. Improved neovascularizing agents will make the transplants last longer and enhance pregnancy outcomes.23,24 I see planned fertility preservation increasing, based on the experience gained from cancer patients and some preliminary experience with planned OTC, especially for healthy women who wish to consider delaying menopause.25,26

Because of attrition from apoptosis, approximately 2,000 oocytes are wasted per ovulation. Through calculation models, we predict that if an equivalent of one-third of a woman’s ovarian cortex can be cryopreserved (which may not significantly affect the age at natural menopause) before age 40 years, transplantation at perimenopause may provide sufficient primordial follicles to delay menopause for 5 years or longer.26 Because ovarian tissue can also be transplanted subcutaneously under local anesthesia, as we have shown,27,28 repeated heterotopic transplants can be performed in an office setting at reduced cost, invasiveness, and with enhanced effectiveness. We can expect increasing reports and progress on this planned use of OTC and transplantation in the future. 
 

Dr. Oktay is professor of obstetrics & gynecology and reproductive sciences and director of the Laboratory of Molecular Reproduction and Fertility Preservation at Yale University, New Haven, Conn. Dr. Trolice is director of The IVF Center in Winter Park, Fla., and professor of obstetrics and gynecology at the University of Central Florida, Orlando.

References

1. Lurie S. Eur J Obstet Gynecol Reprod Biol. 1992 Jan 9;43(1):1-7.

2. Oktay K and Karlikaya G. N Engl J Med. 2000 Jun 22;342(25):1919.

3. Sonmezer and Oktay K. Hum Reprod Update. 2004;10(3):251-66.

4. Oktay K et al. J Clin Oncol. 2018 Jul 1;36(19):1994-2001.

5. Goldfarb SB et al. Breast Cancer Res Treat. 2021;185:165-73.

6. Titus S et al. Sci Rep. 2021 Jan 11;11(1):407.

7. Soleimani R et al. Aging (Albany NY). 2011 Aug;3(8):782-93.

8. Titus S et al. Sci Transl Med. 2013 Feb 13;5(172):172ra21.

9. Oktay KH et al. Fertil Steril. 2022 Jan 5:S0015-0282(21)02293-7.

10. Oktay K et al. Fertil Steril. 2022;117(1):181-92.

11. Practice Committee of the American Society for Reproductive Medicine. Fertil Steril. 2019;112(6):1022–33.

12. Cil A et al. Fertil Steril. 2013 Aug;100(2):492-9.e3.

13. Goldman KN et al. Fertil Steril. 2013 Sep;100(3):712-7.

14. Marin L and Oktay K. Scientific history of ovarian tissue cryopreservation and transplantation. In: Oktay K (ed.), Principles and Practice of Ovarian Tissue Cryopreservation and Transplantation. Elsevier;2022:1-10.

15. Oktay K et al. J Clin Oncol. 2005 Jul 1;23(19):4347-53.

16. Kim JY et al. J Clin Endocrinol Metab. 2016 Apr;101(4):1364-71.

17. Turan V et al. J Clin Oncol. 2021;39:18.

18. Oktay K et al. J Clin Oncol. 2010 Jan 10;28(2):240-4.

19. Lin W et al. J Clin Endocrinol Metab. 2017;102(10):3839-47.

20. Turan V et al. Reprod Sci. 2018;(25):26-32.

21. Oktay K et al. Presence of BRCA mutations and a pre-chemotherapy AMH level of < 2ng/mL strongly predict risk of amenorrhea in women with breast cancer P-291. Presented at the American Society for Reproductive Medicine 78th annual meeting, Anaheim, Calif. Oct. 22-26, 2022.

22. Oktay KH et al. Fertil Steril. 2020;113(6):1251‐60.e1.

23. Soleimani R et al. PLoS One. 2011 Apr 29;6(4):e19475.

24. Marin L et al. Future aspects of ovarian cryopreservation and transplantation. In: Oktay K (ed.). Principles and Practice of Ovarian Tissue Cryopreservation and Transplantation. Elsevier; 2022;223-30.

25. Oktay KH et al. Trends Mol Med. 2021;27(8):753-61.

26. Oktay K and Marin L. Ovarian tissue cryopreservation for delaying childbearing and menopause. In: Oktay, K. (Ed.), Principles and Practice of Ovarian Tissue Cryopreservation and Transplantation. Elsevier;2022:195-204.

27. Oktay K et al. JAMA. 2001 Sep 26;286(12):1490-3.

28. Oktay K et al. Lancet. 2004 Mar 13;363(9412):837-40.

From the first obscure reference until the 19th century, the maternal mortality rate from an ectopic pregnancy was nearly 100%. In the past 140 years, because of early detection and prompt surgical management, the mortality rate from an ectopic pregnancy declined from 72%-90% in 1880 to 0.48% from 2004 to 2008.1 Given this remarkable reduction in mortality, the 20th-century approach to ectopic pregnancy evolved from preserving the life of the mother to preserving fertility by utilizing conservative treatment with methotrexate and/or tubal surgery.

Why the reference to ectopic pregnancy? Advances in oncology have comparably affected our approach to cancer patients. The increase in survival rates following a cancer diagnosis has fostered revolutionary developments in fertility preservation to obviate the effect of gonadotoxic therapy. We have evolved from shielding and transposing ovaries to ovarian tissue cryopreservation2,3 with rapid implementation.

Trolice_Mark_P_FLA_new_web.jpg
Dr. Mark P. Trolice

One of the leaders in the field of female fertility preservation is Kutluk Oktay, MD, of Yale University, New Haven, Conn. I posed the following salient questions to him on the state of fertility preservation as well as expectations for the future.

Oktay_Kutluk_CN_web.jpg
Dr. Kutluk Oktay

 

Q1. What medication/treatment is gonadotoxic that warrants a consultation for fertility preservation?

A: While new drugs for cancer treatment continue to be approved and require testing for gonadotoxicity, evidence is clear on the damaging effects of alkylating agents such as cyclophosphamide, ifosfamide, chlorambucil, and melphalan on primordial follicle reserve.4 A useful tool to determine the risk of alkylating agents affecting fertility is the Cyclophosphamide Equivalent Dose (CED) Calculator. Likewise, topoisomerase inhibitors, such as doxorubicin4 induce ovarian reserve damage by causing double-strand DNA breaks (DSBs) in oocytes.5-7 Contrary to common belief, chemotherapy exposure suppresses the mechanisms that can initiate follicle growth.6 When DSBs occur, some oocytes may be able to repair such damage, otherwise apoptosis is triggered, which results in irreversible ovarian reserve loss.7 Younger individuals have much higher repair capacity, the magnitude of damage can be hard to predict, and it is variable.8,9 So, prior exposure to gonadotoxic drugs does not preclude consideration of fertility preservation.10

In addition, pelvic radiation, in a dose-dependent manner, causes severe DSBs and triggers the same cell suicide mechanisms while also potentially damaging uterine function. Additional information can be found in the American Society of Clinical Oncology Fertility Preservation Guidelines.4
 

Q2. What are the current options for fertility preservation in patients who will be exposed to gonadotoxic medication/treatment?

A: The current fertility preservation options for female patients faced with gonadotoxic treatments are embryo, oocyte, and ovarian tissue cryopreservation (OTC). Selection of fertility preservation is typically contingent upon the timetable of treatment. Oocyte and embryo cryopreservation have been the standard of care. Recently, OTC had its experimental designation removed by American Society for Reproductive Medicine11 with the advantage of not requiring ovarian stimulation or sexual maturity; and it may to be performed while patients are receiving chemotherapy. If successful, OTC followed by orthotopic transplantation has the potential to restore natural ovarian function, thereby allowing spontaneous conception.10 Especially in young adults, ovarian reserve loss is fractional and can remain at reasonable levels after a few courses of chemotherapy. Ovarian stimulation is risky after the initiation of chemotherapy because of the severe DNA damage to oocytes of developing follicles and the associated poor response.7 Hence, ovarian stimulation should be initiated and completed before the initiation of chemotherapy.

 

 

Q3. How successful are the approved fertility preservation options in obtaining oocytes for future utilization by ART?

A:
We have decades of experience with embryo cryopreservation and proven success rates that patients can check on the SART.org website for individual clinics. For oocyte cryopreservation, models are used to provide calculation estimates because the technique is less established.12 Although success rates are approaching those with fresh oocytes, they are still not equal.13 OTC followed by orthotopic tissue transplantation has the least outcomes data (approximately 200 reported livebirths to date with a 25% live birth rate per recipient worldwide10 since the first success was reported in 2000.2,14

With our robotic surgical approach to orthotopic and heterotopic ovarian tissue transplantation and the utility of neovascularizing agents, we have found that ovarian graft longevity is extended. Oocytes/embryos can be obtained and has resulted in one to two livebirths in all our recipients to date.10 Unfortunately, if any of the critical steps are not up to standards (freezing, thawing, or transplantation), success rates can dramatically decline. Therefore, providers and patients should seek centers with experience in all three stages of this procedure to maximize outcomes.
 

Q4. Are there concerns of increasing recurrence/mortality with fertility preservation given hormonal exposure?

A: Yes, this concern exists, at least in theory for estrogen-sensitive cancers, most commonly breast cancer. We developed ovarian stimulation protocols supplemented with anti-estrogen treatments (tamoxifen, an estrogen-receptor antagonist, and letrozole, an aromatase inhibitor) that appear equally effective and reduce estrogen exposure in any susceptible cancer.15,16 Even in estrogen receptor–negative tumors, high estrogen exposure may activate non–estrogen receptor–dependent pathways. In addition, even those tumors that are practically deemed estrogen receptor negative may still contain a small percentage of estrogen receptors, which may become active at high estrogen levels.

Therefore, when we approach women with estrogen-sensitive cancers, e.g., breast and endometrial, we do not alter our approach based on receptor status. One exception occurs in women with BRCA mutations, especially the BRCA1, as they have 25% lower serum anti-müllerian hormone (AMH) levels,8,17 yield fewer oocytes in response to ovarian stimulation,18,19 and have lower fertilization rates and embryo numbers20 compared with those without the mutations.
 

Q5. Are all reproductive centers capable of offering fertility preservation? If not, how does a patient find a center?

A: All IVF clinics offer embryo and, presumably, oocyte cryopreservation. Pregnancy outcomes vary based on the center’s experience. Globally, major differences exist in the availability and competency of OTC along with the subsequent transplantation approach. A limited number of centers have competency in all aspects of OTC, i.e., cryopreservation, thawing, and transplantation. In general, fertility preservation patients have a multitude of medical issues that necessitate management expertise and the bandwidth to coordinate with cancer health professionals. The reproductive centers offering fertility preservation should be prepared to respond immediately and accommodate patients about to undergo gonadotoxic treatment.
 

Q6. How should a patient be counseled before proceeding with fertility preservation?

A:
The candidate should be counseled on the likelihood of damage from gonadotoxic therapy and all fertility preservation options, on the basis of the urgency of treatment and the woman’s long-term goals. For example, the desire for a large family may compel a patient to undergo multiple cycles of ovarian stimulation or a combination of oocyte/embryo cryopreservation with OTC. In patients who are undergoing embryo cryopreservation, I recommend preimplantation genetic testing for aneuploidies, although there are limitations to its application. Other novel pieces of information we are using in counseling are baseline AMH levels and BRCA mutation status for women with breast cancer. In an 8-year-long NIH-funded prospective longitudinal study we found that women with both baseline AMH < 2 ng/mL and BRCA mutations are at significantly higher risk of losing their ovarian reserve and developing amenorrhea.21 Because the oocytes of women with BRCA mutations are deficient in DNA repair as we have previously shown,19 they are more liable to death upon exposure to DNA-damaging cancer drugs such as cyclophosphamide and doxorubicin.22

 

 

Q7. What is the time limit for use of cryopreserved oocytes/tissue?

A: Under optimal storage conditions, cryopreserved oocytes/tissue can be utilized indefinitely without a negative effect on pregnancy outcomes.
 

Q8. What does the future hold for fertility preservation?

A: The future holds promise for both the medical and nonmedical (planned) utility of fertility preservation. With the former, we will see that the utility of OTC and orthotopic and heterotopic tissue transplantation increase as success rates improve. Improved neovascularizing agents will make the transplants last longer and enhance pregnancy outcomes.23,24 I see planned fertility preservation increasing, based on the experience gained from cancer patients and some preliminary experience with planned OTC, especially for healthy women who wish to consider delaying menopause.25,26

Because of attrition from apoptosis, approximately 2,000 oocytes are wasted per ovulation. Through calculation models, we predict that if an equivalent of one-third of a woman’s ovarian cortex can be cryopreserved (which may not significantly affect the age at natural menopause) before age 40 years, transplantation at perimenopause may provide sufficient primordial follicles to delay menopause for 5 years or longer.26 Because ovarian tissue can also be transplanted subcutaneously under local anesthesia, as we have shown,27,28 repeated heterotopic transplants can be performed in an office setting at reduced cost, invasiveness, and with enhanced effectiveness. We can expect increasing reports and progress on this planned use of OTC and transplantation in the future. 
 

Dr. Oktay is professor of obstetrics & gynecology and reproductive sciences and director of the Laboratory of Molecular Reproduction and Fertility Preservation at Yale University, New Haven, Conn. Dr. Trolice is director of The IVF Center in Winter Park, Fla., and professor of obstetrics and gynecology at the University of Central Florida, Orlando.

References

1. Lurie S. Eur J Obstet Gynecol Reprod Biol. 1992 Jan 9;43(1):1-7.

2. Oktay K and Karlikaya G. N Engl J Med. 2000 Jun 22;342(25):1919.

3. Sonmezer and Oktay K. Hum Reprod Update. 2004;10(3):251-66.

4. Oktay K et al. J Clin Oncol. 2018 Jul 1;36(19):1994-2001.

5. Goldfarb SB et al. Breast Cancer Res Treat. 2021;185:165-73.

6. Titus S et al. Sci Rep. 2021 Jan 11;11(1):407.

7. Soleimani R et al. Aging (Albany NY). 2011 Aug;3(8):782-93.

8. Titus S et al. Sci Transl Med. 2013 Feb 13;5(172):172ra21.

9. Oktay KH et al. Fertil Steril. 2022 Jan 5:S0015-0282(21)02293-7.

10. Oktay K et al. Fertil Steril. 2022;117(1):181-92.

11. Practice Committee of the American Society for Reproductive Medicine. Fertil Steril. 2019;112(6):1022–33.

12. Cil A et al. Fertil Steril. 2013 Aug;100(2):492-9.e3.

13. Goldman KN et al. Fertil Steril. 2013 Sep;100(3):712-7.

14. Marin L and Oktay K. Scientific history of ovarian tissue cryopreservation and transplantation. In: Oktay K (ed.), Principles and Practice of Ovarian Tissue Cryopreservation and Transplantation. Elsevier;2022:1-10.

15. Oktay K et al. J Clin Oncol. 2005 Jul 1;23(19):4347-53.

16. Kim JY et al. J Clin Endocrinol Metab. 2016 Apr;101(4):1364-71.

17. Turan V et al. J Clin Oncol. 2021;39:18.

18. Oktay K et al. J Clin Oncol. 2010 Jan 10;28(2):240-4.

19. Lin W et al. J Clin Endocrinol Metab. 2017;102(10):3839-47.

20. Turan V et al. Reprod Sci. 2018;(25):26-32.

21. Oktay K et al. Presence of BRCA mutations and a pre-chemotherapy AMH level of < 2ng/mL strongly predict risk of amenorrhea in women with breast cancer P-291. Presented at the American Society for Reproductive Medicine 78th annual meeting, Anaheim, Calif. Oct. 22-26, 2022.

22. Oktay KH et al. Fertil Steril. 2020;113(6):1251‐60.e1.

23. Soleimani R et al. PLoS One. 2011 Apr 29;6(4):e19475.

24. Marin L et al. Future aspects of ovarian cryopreservation and transplantation. In: Oktay K (ed.). Principles and Practice of Ovarian Tissue Cryopreservation and Transplantation. Elsevier; 2022;223-30.

25. Oktay KH et al. Trends Mol Med. 2021;27(8):753-61.

26. Oktay K and Marin L. Ovarian tissue cryopreservation for delaying childbearing and menopause. In: Oktay, K. (Ed.), Principles and Practice of Ovarian Tissue Cryopreservation and Transplantation. Elsevier;2022:195-204.

27. Oktay K et al. JAMA. 2001 Sep 26;286(12):1490-3.

28. Oktay K et al. Lancet. 2004 Mar 13;363(9412):837-40.

Publications
Publications
Topics
Article Type
Sections
Teambase XML
<?xml version="1.0" encoding="UTF-8"?>
<!--$RCSfile: InCopy_agile.xsl,v $ $Revision: 1.35 $-->
<!--$RCSfile: drupal.xsl,v $ $Revision: 1.7 $-->
<root generator="drupal.xsl" gversion="1.7"> <header> <fileName>161716</fileName> <TBEID>0C047975.SIG</TBEID> <TBUniqueIdentifier>MD_0C047975</TBUniqueIdentifier> <newsOrJournal>News</newsOrJournal> <publisherName>Frontline Medical Communications</publisherName> <storyname>Reproductive Rounds</storyname> <articleType>353</articleType> <TBLocation>QC Done-All Pubs</TBLocation> <QCDate>20230123T150549</QCDate> <firstPublished>20230124T100110</firstPublished> <LastPublished>20230124T100110</LastPublished> <pubStatus qcode="stat:"/> <embargoDate/> <killDate/> <CMSDate>20230124T100109</CMSDate> <articleSource/> <facebookInfo/> <meetingNumber/> <byline>Trolice and Oktay</byline> <bylineText>MARK P. TROLICE, MD, MBA, WITH KUTLUK OKTAY, MD, PHD</bylineText> <bylineFull>MARK P. TROLICE, MD, MBA, WITH KUTLUK OKTAY, MD, PHD</bylineFull> <bylineTitleText/> <USOrGlobal/> <wireDocType/> <newsDocType/> <journalDocType/> <linkLabel/> <pageRange/> <citation/> <quizID/> <indexIssueDate/> <itemClass qcode="ninat:text"/> <provider qcode="provider:imng"> <name>IMNG Medical Media</name> <rightsInfo> <copyrightHolder> <name>Frontline Medical News</name> </copyrightHolder> <copyrightNotice>Copyright (c) 2015 Frontline Medical News, a Frontline Medical Communications Inc. company. All rights reserved. This material may not be published, broadcast, copied, or otherwise reproduced or distributed without the prior written permission of Frontline Medical Communications Inc.</copyrightNotice> </rightsInfo> </provider> <abstract/> <metaDescription>From the first obscure reference until the 19th century, the maternal mortality rate from an ectopic pregnancy was nearly 100%. In the past 140 years, because o</metaDescription> <articlePDF/> <teaserImage>275553</teaserImage> <teaser>The current fertility preservation options for female patients faced with gonadotoxic treatments include embryo and oocyte cryopreservation; ovarian tissue cryopreservation is the newest modality.</teaser> <title>Advances in fertility preservation: Q &amp; A</title> <deck/> <disclaimer/> <AuthorList/> <articleURL/> <doi/> <pubMedID/> <publishXMLStatus/> <publishXMLVersion>1</publishXMLVersion> <useEISSN>0</useEISSN> <urgency/> <pubPubdateYear/> <pubPubdateMonth/> <pubPubdateDay/> <pubVolume/> <pubNumber/> <wireChannels/> <primaryCMSID/> <CMSIDs/> <keywords/> <seeAlsos/> <publications_g> <publicationData> <publicationCode>ob</publicationCode> <pubIssueName/> <pubArticleType/> <pubTopics/> <pubCategories/> <pubSections/> </publicationData> </publications_g> <publications> <term canonical="true">23</term> </publications> <sections> <term canonical="true">68807</term> </sections> <topics> <term canonical="true">218</term> </topics> <links> <link> <itemClass qcode="ninat:picture"/> <altRep contenttype="image/jpeg">images/2400f92a.jpg</altRep> <description role="drol:caption">Dr. Mark P. Trolice</description> <description role="drol:credit"/> </link> <link> <itemClass qcode="ninat:picture"/> <altRep contenttype="image/jpeg">images/24011830.jpg</altRep> <description role="drol:caption">Dr. Kutluk Oktay</description> <description role="drol:credit"/> </link> </links> </header> <itemSet> <newsItem> <itemMeta> <itemRole>Main</itemRole> <itemClass>text</itemClass> <title>Advances in fertility preservation: Q &amp; A</title> <deck/> </itemMeta> <itemContent> <p>From the first obscure reference until the 19th century, the maternal mortality rate from an ectopic pregnancy was nearly 100%. In the past 140 years, because of early detection and prompt surgical management, the mortality rate from an ectopic pregnancy declined from 72%-90% in 1880 to 0.48% from 2004 to 2008.<sup>1</sup> Given this remarkable reduction in mortality, the 20th-century approach to ectopic pregnancy evolved from preserving the life of the mother to preserving fertility by utilizing conservative treatment with methotrexate and/or tubal surgery.</p> <p>Why the reference to ectopic pregnancy? Advances in oncology have comparably affected our approach to cancer patients. The increase in survival rates following a cancer diagnosis has fostered revolutionary developments in fertility preservation to obviate the effect of gonadotoxic therapy. We have evolved from shielding and transposing ovaries to ovarian tissue cryopreservation<sup>2,3</sup> with rapid implementation. <br/><br/>[[{"fid":"275553","view_mode":"medstat_image_flush_left","fields":{"format":"medstat_image_flush_left","field_file_image_alt_text[und][0][value]":"Dr. Mark P. Trolice, director, Fertility CARE: The IVF Center, and professor, UCF College of Medicine, Orlando","field_file_image_credit[und][0][value]":"","field_file_image_caption[und][0][value]":"Dr. Mark P. Trolice"},"type":"media","attributes":{"class":"media-element file-medstat_image_flush_left"}}]]One of the leaders in the field of female fertility preservation is Kutluk Oktay, MD, of Yale University, New Haven, Conn. I posed the following salient questions to him on the state of fertility preservation as well as expectations for the future.[[{"fid":"292456","view_mode":"medstat_image_flush_left","fields":{"format":"medstat_image_flush_left","field_file_image_alt_text[und][0][value]":"Dr. Kutluk Oktay, professor of obstetrics &amp; gynecology and reproductive sciences and director of the Laboratory of Molecular Reproduction and Fertility Preservation at Yale University, New Haven, Conn.","field_file_image_credit[und][0][value]":"","field_file_image_caption[und][0][value]":"Dr. Kutluk Oktay"},"type":"media","attributes":{"class":"media-element file-medstat_image_flush_left"}}]]<br/><br/></p> <p><strong>Q1. What medication/treatment is gonadotoxic that warrants a consultation for fertility preservation?</strong> <br/><br/><strong>A:</strong> While new drugs for cancer treatment continue to be approved and require testing for gonadotoxicity, evidence is clear on the damaging effects of alkylating agents such as cyclophosphamide, ifosfamide, chlorambucil, and melphalan on primordial follicle reserve.<sup>4</sup> A useful tool to determine the risk of alkylating agents affecting fertility is the Cyclophosphamide Equivalent Dose (<span class="Hyperlink"><a href="https://fertilitypreservationpittsburgh.org/fertility-resources/fertility-risk-calculator/">CED</a></span>) Calculator. Likewise, topoisomerase inhibitors, such as doxorubicin<sup>4</sup> induce ovarian reserve damage by causing double-strand DNA breaks (DSBs) in oocytes.<sup>5-7</sup> Contrary to common belief, chemotherapy exposure suppresses the mechanisms that can initiate follicle growth.<sup>6</sup> When DSBs occur, some oocytes may be able to repair such damage, otherwise apoptosis is triggered, which results in irreversible ovarian reserve loss.<sup>7</sup> Younger individuals have much higher repair capacity, the magnitude of damage can be hard to predict, and it is variable.<sup>8,9</sup> So, prior exposure to gonadotoxic drugs does not preclude consideration of fertility preservation.<sup>10</sup> </p> <p>In addition, pelvic radiation, in a dose-dependent manner, causes severe DSBs and triggers the same cell suicide mechanisms while also potentially damaging uterine function. Additional information can be found in the American Society of Clinical Oncology Fertility Preservation Guidelines.<sup>4</sup> <br/><br/></p> <p><strong>Q2. What are the current options for fertility preservation in patients who will be exposed to gonadotoxic medication/treatment?</strong> <br/><br/><strong>A:</strong> The current fertility preservation options for female patients faced with gonadotoxic treatments are embryo, oocyte, and ovarian tissue cryopreservation (OTC). Selection of fertility preservation is typically contingent upon the timetable of treatment. Oocyte and embryo cryopreservation have been the standard of care. Recently, OTC had its experimental designation removed by American Society for Reproductive Medicine<sup>11</sup> with the advantage of not requiring ovarian stimulation or sexual maturity; and it may to be performed while patients are receiving chemotherapy. If successful, OTC followed by orthotopic transplantation has the potential to restore natural ovarian function, thereby allowing spontaneous conception.<sup>10 </sup>Especially in young adults, ovarian reserve loss is fractional and can remain at reasonable levels after a few courses of chemotherapy. Ovarian stimulation is risky after the initiation of chemotherapy because of the severe DNA damage to oocytes of developing follicles and the associated poor response.<sup>7</sup> Hence, ovarian stimulation should be initiated and completed before the initiation of chemotherapy.</p> <p><strong>Q3. How successful are the approved fertility preservation options in obtaining oocytes for future utilization by ART?<br/><br/>A:</strong> We have decades of experience with embryo cryopreservation and proven success rates that patients can check on the SART.org website for individual clinics. For oocyte cryopreservation, models are used to provide <span class="Hyperlink"><a href="https://www.fertilitypreservation.org/contents/probability-calculator">calculation estimates</a></span> because the technique is less established.<sup>12</sup> Although success rates are approaching those with fresh oocytes, they are still not equal.<sup>13 </sup>OTC followed by orthotopic tissue transplantation has the least outcomes data (approximately 200 reported livebirths to date with a 25% live birth rate per recipient worldwide<sup>10</sup> since the first success was reported in 2000.<sup>2,14</sup></p> <p>With our robotic surgical approach to orthotopic and heterotopic ovarian tissue transplantation and the utility of neovascularizing agents, we have found that ovarian graft longevity is extended. Oocytes/embryos can be obtained and has resulted in one to two livebirths in all our recipients to date.<sup>10 </sup>Unfortunately, if any of the critical steps are not up to standards (freezing, thawing, or transplantation), success rates can dramatically decline. Therefore, providers and patients should seek centers with experience in all three stages of this procedure to maximize outcomes.<br/><br/></p> <p><strong>Q4. Are there concerns of increasing recurrence/mortality with fertility preservation given hormonal exposure?</strong> <br/><br/><strong>A:</strong> Yes, this concern exists, at least in theory for estrogen-sensitive cancers, most commonly breast cancer. We developed ovarian stimulation protocols supplemented with anti-estrogen treatments (tamoxifen, an estrogen-receptor antagonist, and letrozole, an aromatase inhibitor) that appear equally effective and reduce estrogen exposure in any susceptible cancer.<sup>15,16</sup> Even in estrogen receptor–negative tumors, high estrogen exposure may activate non–estrogen receptor–dependent pathways. In addition, even those tumors that are practically deemed estrogen receptor negative may still contain a small percentage of estrogen receptors, which may become active at high estrogen levels. </p> <p>Therefore, when we approach women with estrogen-sensitive cancers, e.g., breast and endometrial, we do not alter our approach based on receptor status. One exception occurs in women with BRCA mutations, especially the BRCA1, as they have 25% lower serum anti-müllerian hormone (AMH) levels,<sup>8,17</sup> yield fewer oocytes in response to ovarian stimulation,<sup>18,19</sup> and have lower fertilization rates and embryo numbers<sup>20 </sup>compared with those without the mutations. <br/><br/></p> <p><strong>Q5. Are all reproductive centers capable of offering fertility preservation? If not, how does a patient find a center?</strong> </p> <p><strong>A:</strong> All IVF clinics offer embryo and, presumably, oocyte cryopreservation. Pregnancy outcomes vary based on the center’s experience. Globally, major differences exist in the availability and competency of OTC along with the subsequent transplantation approach. A limited number of centers have competency in all aspects of OTC, i.e., cryopreservation, thawing, and transplantation. In general, fertility preservation patients have a multitude of medical issues that necessitate management expertise and the bandwidth to coordinate with cancer health professionals. The reproductive centers offering fertility preservation should be prepared to respond immediately and accommodate patients about to undergo gonadotoxic treatment. <br/><br/></p> <p><strong>Q6. How should a patient be counseled before proceeding with fertility preservation? <br/><br/>A:</strong> The candidate should be counseled on the likelihood of damage from gonadotoxic therapy and all fertility preservation options, on the basis of the urgency of treatment and the woman’s long-term goals. For example, the desire for a large family may compel a patient to undergo multiple cycles of ovarian stimulation or a combination of oocyte/embryo cryopreservation with OTC. In patients who are undergoing embryo cryopreservation, I recommend preimplantation genetic testing for aneuploidies, although there are limitations to its application. Other novel pieces of information we are using in counseling are baseline AMH levels and BRCA mutation status for women with breast cancer. In an 8-year-long NIH-funded prospective longitudinal study we found that women with both baseline AMH &lt; 2 ng/mL and BRCA mutations are at significantly higher risk of losing their ovarian reserve and developing amenorrhea.<sup>21</sup> Because the oocytes of women with BRCA mutations are deficient in DNA repair as we have previously shown,<sup>19</sup> they are more liable to death upon exposure to DNA-damaging cancer drugs such as cyclophosphamide and doxorubicin.<sup>22</sup></p> <p><strong>Q7. What is the time limit for use of cryopreserved oocytes/tissue?</strong> </p> <p><strong>A:</strong> Under optimal storage conditions, cryopreserved oocytes/tissue can be utilized indefinitely without a negative effect on pregnancy outcomes. <br/><br/></p> <p><strong>Q8. What does the future hold for fertility preservation?</strong> <br/><br/><strong>A:</strong> The future holds promise for both the medical and nonmedical (planned) utility of fertility preservation. With the former, we will see that the utility of OTC and orthotopic and heterotopic tissue transplantation increase as success rates improve. Improved neovascularizing agents will make the transplants last longer and enhance pregnancy outcomes.<sup>23,24</sup> I see planned fertility preservation increasing, based on the experience gained from cancer patients and some preliminary experience with planned OTC, especially for healthy women who wish to consider delaying menopause.<sup>25,26</sup> </p> <p>Because of attrition from apoptosis, approximately 2,000 oocytes are wasted per ovulation. Through calculation models, we predict that if an equivalent of one-third of a woman’s ovarian cortex can be cryopreserved (which may not significantly affect the age at natural menopause) before age 40 years, transplantation at perimenopause may provide sufficient primordial follicles to delay menopause for 5 years or longer.<sup>26</sup> Because ovarian tissue can also be transplanted subcutaneously under local anesthesia, as we have shown,<sup>27,28</sup> repeated heterotopic transplants can be performed in an office setting at reduced cost, invasiveness, and with enhanced effectiveness. We can expect increasing reports and progress on this planned use of OTC and transplantation in the future. <br/><br/></p> <p> <em>Dr. Oktay is professor of obstetrics &amp; gynecology and reproductive sciences and director of the Laboratory of Molecular Reproduction and Fertility Preservation at Yale University, New Haven, Conn. Dr. Trolice is director of The IVF Center in Winter Park, Fla., and professor of obstetrics and gynecology at the University of Central Florida, Orlando. </em> </p> <h2>References</h2> <p>1. Lurie S. <span class="Hyperlink"><a href="https://www.ejog.org/article/0028-2243(92)90235-Q/pdf">Eur J Obstet Gynecol Reprod Biol. 1992 Jan 9;43(1):1-7</a></span>. <br/><br/>2. Oktay K and Karlikaya G. <span class="Hyperlink"><a href="https://www.nejm.org/doi/full/10.1056/NEJM200006223422516">N Engl J Med. 2000 Jun 22;342(25):1919</a></span>. </p> <p>3. Sonmezer and Oktay K. <span class="Hyperlink"><a href="https://academic.oup.com/humupd/article/10/3/251/682124?login=false">Hum Reprod Update. 2004;10(3):251-66</a></span>.</p> <p>4. Oktay K et al. <span class="Hyperlink"><a href="https://ascopubs.org/doi/10.1200/JCO.2018.78.1914">J Clin Oncol. 2018 Jul 1;36(19):1994-2001</a></span>. <br/><br/>5. Goldfarb SB et al. <span class="Hyperlink"><a href="https://link.springer.com/article/10.1007/s10549-020-05933-7">Breast Cancer Res Treat. 2021;185:165-73</a></span>.<br/><br/>6. Titus S et al. <span class="Hyperlink"><a href="https://www.nature.com/articles/s41598-020-79643-x">Sci Rep. 2021 Jan 11;11(1):407</a></span>. <br/><br/>7. Soleimani R et al. <span class="Hyperlink"><a href="https://www.aging-us.com/article/100363/text">Aging (Albany NY). 2011 Aug;3(8):782-93</a></span>. <br/><br/>8. Titus S et al. <span class="Hyperlink"><a href="https://www.science.org/journal/stm">Sci Transl Med. 2013 Feb 13;5(172):172ra21</a></span>. <br/><br/>9. Oktay KH et al. <span class="Hyperlink"><a href="https://www.fertstert.org/article/S0015-0282(21)02293-7/fulltext">Fertil Steril. 2022 Jan 5:S0015-0282(21)02293-7</a></span>.<br/><br/>10. Oktay K et al. <span class="Hyperlink"><a href="https://www.fertstert.org/article/S0015-0282(21)01975-0/fulltext">Fertil Steril. 2022;117(1):181-92</a></span>.<br/><br/>11. Practice Committee of the American Society for Reproductive Medicine. <span class="Hyperlink"><a href="https://www.fertstert.org/article/S0015-0282(19)32355-6/fulltext">Fertil Steril. 2019;112(6):1022–33</a></span>. <br/><br/>12. Cil A et al. <span class="Hyperlink"><a href="https://www.fertstert.org/article/S0015-0282(13)00519-0/fulltext">Fertil Steril. 2013 Aug;100(2):492-9.e3</a></span>. <br/><br/><span class="cf01">13. Goldman KN et al. </span><span class="Hyperlink"><a href="https://www.fertstert.org/article/S0015-0282(13)00554-2/fulltext">Fertil Steril. 2013 Sep;100(3):712-7</a>.<br/><br/></span>14. Marin L and Oktay K. Scientific history of ovarian tissue cryopreservation and transplantation. In: Oktay K (ed.), Principles and Practice of Ovarian Tissue Cryopreservation and Transplantation. <span class="Hyperlink"><a href="https://www.amazon.com/Principles-Practice-Ovarian-Cryopreservation-Transplantation/dp/0323957536">Elsevier;2022:1-10</a></span>.<br/><br/>15. Oktay K et al. <span class="Hyperlink"><a href="https://ascopubs.org/doi/10.1200/JCO.2005.05.037">J Clin Oncol. 2005 Jul 1;23(19):4347-53</a></span>. <br/><br/>16. Kim JY et al. <span class="Hyperlink"><a href="https://academic.oup.com/jcem/article/101/4/1364/2804499?login=false">J Clin Endocrinol Metab. 2016 Apr;101(4):1364-71</a></span>. <br/><br/>17. Turan V et al. <span class="Hyperlink"><a href="https://ascopubs.org/doi/10.1200/JCO.20.02880">J Clin Oncol. 2021;39:18</a></span>. <br/><br/>18. Oktay K et al. <span class="Hyperlink"><a href="https://ascopubs.org/doi/10.1200/JCO.2009.24.2057">J Clin Oncol. 2010 Jan 10;28(2):240-4</a></span>. <br/><br/>19. Lin W et al. <span class="Hyperlink"><a href="https://academic.oup.com/jcem/article/102/10/3839/4082865?login=false">J Clin Endocrinol Metab. 2017;102(10):3839-47</a>.<br/><br/></span>20. Turan V et al. <span class="Hyperlink"><a href="https://link.springer.com/article/10.1177/1933719117728800">Reprod Sci. 2018;(25):26-32</a></span>.</p> <p>21. Oktay K et al. Presence of BRCA mutations and a pre-chemotherapy AMH level of &lt; 2ng/mL strongly predict risk of amenorrhea in women with breast cancer <span class="Hyperlink">P-291. Presented at the American Society for Reproductive Medicine 78th annual meeting, Anaheim, Calif. Oct. 22-26, 2022</span>.</p> <p>22. Oktay KH et al. <span class="Hyperlink"><a href="https://www.fertstert.org/article/S0015-0282(20)30036-4/fulltext">Fertil Steril. 2020;113(6):1251‐60.e1</a></span>. <br/><br/>23. Soleimani R et al. <span class="Hyperlink"><a href="https://journals.plos.org/plosone/article?id=10.1371/journal.pone.0019475">PLoS One. 2011 Apr 29;6(4):e19475</a></span>. <br/><br/>24. Marin L et al. Future aspects of ovarian cryopreservation and transplantation. In: Oktay K (ed.). Principles and Practice of Ovarian Tissue Cryopreservation and Transplantation. <span class="Hyperlink"><a href="https://shop.elsevier.com/books/principles-and-practice-of-ovarian-tissue-cryopreservation-and-transplantation/oktay/978-0-12-823344-3">Elsevier; 2022;223-30</a></span>.<br/><br/>25. Oktay KH et al. <span class="Hyperlink"><a href="https://www.cell.com/trends/molecular-medicine/fulltext/S1471-4914(21)00036-8?_returnURL=https%3A%2F%2Flinkinghub.elsevier.com%2Fretrieve%2Fpii%2FS1471491421000368%3Fshowall%3Dtrue">Trends Mol Med. 2021;27(8):753-61</a></span>. <br/><br/>26. Oktay K and Marin L. Ovarian tissue cryopreservation for delaying childbearing and menopause. In: Oktay, K. (Ed.), Principles and Practice of Ovarian Tissue Cryopreservation and Transplantation. <span class="Hyperlink"><a href="https://www.amazon.com/Principles-Practice-Ovarian-Cryopreservation-Transplantation/dp/0323957536">Elsevier;2022:195-204</a></span>.<br/><br/>27. Oktay K et al. <span class="Hyperlink"><a href="https://jamanetwork.com/journals/jama/fullarticle/194227">JAMA. 2001 Sep 26;286(12):1490-3</a></span>.<br/><br/>28. Oktay K et al. <span class="Hyperlink"><a href="https://www.thelancet.com/journals/lancet/article/PIIS0140-6736(04)15728-0/fulltext">Lancet. 2004 Mar 13;363(9412):837-40</a></span>.</p> </itemContent> </newsItem> <newsItem> <itemMeta> <itemRole>teaser</itemRole> <itemClass>text</itemClass> <title/> <deck/> </itemMeta> <itemContent> </itemContent> </newsItem> </itemSet></root>
Disallow All Ads
Content Gating
No Gating (article Unlocked/Free)
Alternative CME
Disqus Comments
Default
Use ProPublica
Hide sidebar & use full width
render the right sidebar.
Conference Recap Checkbox
Not Conference Recap
Clinical Edge
Display the Slideshow in this Article
Medscape Article
Display survey writer
Reuters content
Disable Inline Native ads
WebMD Article

Top 10 unproven infertility tests and treatments

Article Type
Changed
Mon, 11/21/2022 - 13:42

In 2019, a New York Times opinion piece titled, “The Big IVF Add-On Racket – This is no way to treat patients desperate for a baby”1 alleged exploitation of infertility patients based on a Fertility and Sterility article, “Do à la carte menus serve infertility patients? The ethics and regulation of in vitro fertility add-ons.”2 The desperation of infertility patients combined with their financial burden, caused by inconsistent insurance coverage, has resulted in a perfect storm of frustration and overzealous recommendations for a successful outcome. Since the inception of in vitro fertilization (IVF) itself, infertility patients have been subjected to many unproven tests and procedures that enter the mainstream of care before unequivocal efficacy and safety have been shown.

From ovarian stimulation with intrauterine insemination (IUI) or IVF along with intracytoplasmic sperm injection (ICSI), assisted hatching, and preimplantation genetic testing for aneuploidy (PGT-A), a multitude of options with varying success can overwhelm fertility patients as they walk the tightrope of wanting “the kitchen sink” of treatment while experiencing sticker shock. This month’s article examines the top 10 infertility add-ons that have yet to be shown to improve pregnancy outcomes.

Trolice_Mark _P_FLA_2022_web.jpg
Dr. Mark P. Trolice

1. Blood testing: Prolactin and FSH

In a woman with ovulatory monthly menstrual cycles, a serum prolactin level provides no elucidation of the cause of infertility. If obtained following ovulation, prolactin can often be physiologically elevated, thereby compelling a repeat blood level, which is ideally performed during the early proliferative phase. False elevations of prolactin can be caused by an early morning blood sample, eating, and stress – which may result from worry caused by having to repeat the unnecessary initial blood test!

Follicle-stimulating hormone (FSH) was a first-line hormone test to assess for ovarian age. For nearly 15 years now, FSH has been replaced by anti-Müllerian hormone as a more reliable and earlier test for diminished ovarian reserve. However, FSH is still the hormone test of choice to diagnose primary ovarian insufficiency. Note that the use of ovarian age testing in a woman without infertility can result in both unnecessary patient anxiety and additional testing.
 

2. Endometrial scratch

The concept was understandable, that is, induce endometrial trauma by a biopsy or “scratch,” that results in an inflammatory and immunologic response to increase implantation. Endometrial sampling was recommended to be performed during the month prior to the embryo transfer cycle. While the procedure is brief, the pain response of women varies from minimal to severe. Unfortunately, a randomized controlled trial of over 1,300 patients did not show any improvement in the IVF live birth rate from the scratch procedure.3

3. Diagnostic laparoscopy

In years past, a diagnosis of unexplained infertility was not accepted until a laparoscopy was performed that revealed a normal pelvis. This approach subjected many women to an unindicated and a potentially risky surgery that has not shown benefit. The American Society for Reproductive Medicine’s ReproductiveFacts.org website states: “Routine diagnostic laparoscopy should not be performed unless there is a suspicion of pelvic pathology based on clinical history, an abnormal pelvic exam, or abnormalities identified with less invasive testing. In patients with a normal hysterosalpingogram or the presence of a unilaterally patent tube, diagnostic laparoscopy typically will not change the initial recommendation for treatment.”

 

 



4. Prescribing clomiphene citrate without IUI

Ovulation dysfunction is found in 40% of female factors for fertility. Provided testing reveals a reasonably normal sperm analysis and hysterosalpingogram, ovulation induction medication with ultrasound monitoring along with an hCG trigger is appropriate. In women who ovulate with unexplained infertility and/or mild male factor, the use of clomiphene citrate or letrozole with timed intercourse is often prescribed, particularly in clinics when IUI preparation is not available. Unfortunately, without including IUI, the use of oral ovarian stimulation has been shown by good evidence to be no more effective than natural cycle attempts at conception.4

5. Thrombophilia testing

Recurrent miscarriage, defined by the spontaneous loss of two or more pregnancies (often during the first trimester but may include up to 20 weeks estimated gestational age), has remained an ill-defined problem that lacks a consensus on the most optimal evaluation and treatment. In 2006, an international consensus statement provided guidance on laboratory testing for antiphospholipid syndrome limited to lupus anticoagulant, anticardiolipin IgG and IgM, and IgG and IgM anti–beta2-glycoprotein I assays.5 ASRM does not recommend additional thrombophilia tests as they are unproven causative factors of recurrent miscarriage.

6. Screening hysteroscopy

A standard infertility evaluation includes ovulation testing, assessment of fallopian tube patency, and a sperm analysis. In a subfertile women with a normal ultrasound or hysterosalpingogram in the basic fertility work‐up, a Cochrane data review concluded there is no definitive evidence for improved outcome with a screening hysteroscopy prior to IUI or IVF.6,7 Two large trials included in the Cochrane review, confirmed similar live birth rates whether or not hysteroscopy was performed before IVF. There may value in screening patients with recurrent implantation failure.

7. PGT-A for all

As the efficacy of the first generation of embryo preimplantation genetic testing, i.e., FISH (fluorescence in situ hybridization) was disproven, so has the same result been determined for PGT-A, specifically in women younger than 35.8 In an elegant randomized prospective trial, Munne and colleagues showed no improvement in the ongoing pregnancy rate (OPR) of study patients of all ages who were enrolled with the intention to treat. However, a subanalysis of patients aged 35-40 who completed the protocol did show an improved OPR and lower miscarriage rate per embryo transfer. While there is no evidence to support improved outcomes with the universal application of PGT-A, there may be some benefit in women older than 35 as well as in certain individual patient circumstances.

8. ICSI for nonmale factor infertility; assisted hatching

In an effort to reduce the risk of fertilization failure, programs have broadened the use of ICSI to nonmale factor infertility. While it has been used in PGT to reduce the risk of DNA contamination, particularly in PGT-M (monogenic disorder) and PGT-SR (structural rearrangement) cases, ICSI has not been shown to improve outcomes when there is a normal sperm analysis.9 During IVF embryo development, assisted hatching involves the thinning and/or opening of the zona pellucida either by chemical, mechanical, or laser means around the embryo before transfer with the intention of facilitating implantation. The routine use of assisted hatching is not recommended based on the lack of increase in live birth rates and because it may increase multiple pregnancy and monozygotic twinning rates.10

 

 

9. Acupuncture

Four meta-analyses showed no evidence of the overall benefit of acupuncture for improving live birth rates regardless of whether acupuncture was performed around the time of oocyte retrieval or around the day of embryo transfer. Consequently, acupuncture cannot be recommended routinely to improve IVF outcomes.11

10. Immunologic tests/treatments

Given the “foreign” genetic nature of a fetus, attempts to suppress the maternal immunologic response to sustain the pregnancy have been made for decades, especially for recurrent miscarriage and recurrent implantation failure with IVF. Testing has included natural killer (NK) cells, human leukocyte antigen (HLA) genotypes, and cytokines. While NK cells can be examined by endometrial biopsy, levels fluctuate based on the cycle phase, and no correlation between peripheral blood testing and uterine NK cell levels has been shown. Further, no consensus has been reached on reliable normal reference ranges in uterine NK cells.12

Several treatments have been proposed to somehow modulate the immune system during the implantation process thereby improving implantation and live birth, including lipid emulsion (intralipid) infusion, intravenous immunoglobulin, leukocyte immunization therapy, tacrolimus, anti–tumor necrosis factor agents, and granulocyte colony-stimulating factor. A recent systematic review and meta-analysis cited low-quality studies and did not recommend the use of any of these immune treatments.13 Further, immunomodulation has many known side effects, some of which are serious (including hepatosplenomegaly, thrombocytopenia, leukopenia, renal failure, thromboembolism, and anaphylactic reactions). Excluding women with autoimmune disease, taking glucocorticoids or other immune treatments to improve fertility has not been proven.13

Conclusion

To quote the New York Times opinion piece, “IVF remains an under-regulated arena, and entrepreneurial doctors and pharmaceutical and life science companies are eager to find new ways to cash in on a growing global market that is projected to be as large as $40 billion by 2024.” While this bold statement compels a huge “Ouch!”, it reminds us of our obligation to provide evidence-based medicine and to include emotional and financial harm to our oath of Primum non nocere.

References

1. The News York Times. 2019 Dec 12. Opinion.

2. Wilkinson J et al. Fertil Steril. 2019;112(6):973-7.

3. Lensen S et al. N Engl J Med. 2019 Jan 24;380(4):325-34.

4. Practice Committee of the American Society for Reproductive Medicine. Fertil Steril. 2020;113(2):305-22.

5. Miyakis S et al. J Thromb Haemost. 2006;4(2):295-306.

6. Kamath MS et al. Cochrane Database Syst Rev. 2019 Apr 16;4(4):CD012856.

7. Bosteels J et al. Cochrane Database Syst Rev. 2013 Jan 31;(1):CD009461.

8. Munne S et al. Fertil Steril. 2019;112(6):1071-9.

9. Practice Committees of the American Society for Reproductive Medicine and the Society for Assisted Reproductive Technology. Fertil Steril. 2020;114(2):239-45.

10. Lacey L et al. Cochrane Database Syst Rev. March 7 2021;3:2199.

11. Coyle ME et al. Acupunct Med. 2021;39(1):20-9.

12. Von Woon E et al. Hum Reprod Update. 2022;30;28(4):548-82.

13. Achilli C et al. Fertil Steril. 2018;110(6):1089-100.

Publications
Topics
Sections

In 2019, a New York Times opinion piece titled, “The Big IVF Add-On Racket – This is no way to treat patients desperate for a baby”1 alleged exploitation of infertility patients based on a Fertility and Sterility article, “Do à la carte menus serve infertility patients? The ethics and regulation of in vitro fertility add-ons.”2 The desperation of infertility patients combined with their financial burden, caused by inconsistent insurance coverage, has resulted in a perfect storm of frustration and overzealous recommendations for a successful outcome. Since the inception of in vitro fertilization (IVF) itself, infertility patients have been subjected to many unproven tests and procedures that enter the mainstream of care before unequivocal efficacy and safety have been shown.

From ovarian stimulation with intrauterine insemination (IUI) or IVF along with intracytoplasmic sperm injection (ICSI), assisted hatching, and preimplantation genetic testing for aneuploidy (PGT-A), a multitude of options with varying success can overwhelm fertility patients as they walk the tightrope of wanting “the kitchen sink” of treatment while experiencing sticker shock. This month’s article examines the top 10 infertility add-ons that have yet to be shown to improve pregnancy outcomes.

Trolice_Mark _P_FLA_2022_web.jpg
Dr. Mark P. Trolice

1. Blood testing: Prolactin and FSH

In a woman with ovulatory monthly menstrual cycles, a serum prolactin level provides no elucidation of the cause of infertility. If obtained following ovulation, prolactin can often be physiologically elevated, thereby compelling a repeat blood level, which is ideally performed during the early proliferative phase. False elevations of prolactin can be caused by an early morning blood sample, eating, and stress – which may result from worry caused by having to repeat the unnecessary initial blood test!

Follicle-stimulating hormone (FSH) was a first-line hormone test to assess for ovarian age. For nearly 15 years now, FSH has been replaced by anti-Müllerian hormone as a more reliable and earlier test for diminished ovarian reserve. However, FSH is still the hormone test of choice to diagnose primary ovarian insufficiency. Note that the use of ovarian age testing in a woman without infertility can result in both unnecessary patient anxiety and additional testing.
 

2. Endometrial scratch

The concept was understandable, that is, induce endometrial trauma by a biopsy or “scratch,” that results in an inflammatory and immunologic response to increase implantation. Endometrial sampling was recommended to be performed during the month prior to the embryo transfer cycle. While the procedure is brief, the pain response of women varies from minimal to severe. Unfortunately, a randomized controlled trial of over 1,300 patients did not show any improvement in the IVF live birth rate from the scratch procedure.3

3. Diagnostic laparoscopy

In years past, a diagnosis of unexplained infertility was not accepted until a laparoscopy was performed that revealed a normal pelvis. This approach subjected many women to an unindicated and a potentially risky surgery that has not shown benefit. The American Society for Reproductive Medicine’s ReproductiveFacts.org website states: “Routine diagnostic laparoscopy should not be performed unless there is a suspicion of pelvic pathology based on clinical history, an abnormal pelvic exam, or abnormalities identified with less invasive testing. In patients with a normal hysterosalpingogram or the presence of a unilaterally patent tube, diagnostic laparoscopy typically will not change the initial recommendation for treatment.”

 

 



4. Prescribing clomiphene citrate without IUI

Ovulation dysfunction is found in 40% of female factors for fertility. Provided testing reveals a reasonably normal sperm analysis and hysterosalpingogram, ovulation induction medication with ultrasound monitoring along with an hCG trigger is appropriate. In women who ovulate with unexplained infertility and/or mild male factor, the use of clomiphene citrate or letrozole with timed intercourse is often prescribed, particularly in clinics when IUI preparation is not available. Unfortunately, without including IUI, the use of oral ovarian stimulation has been shown by good evidence to be no more effective than natural cycle attempts at conception.4

5. Thrombophilia testing

Recurrent miscarriage, defined by the spontaneous loss of two or more pregnancies (often during the first trimester but may include up to 20 weeks estimated gestational age), has remained an ill-defined problem that lacks a consensus on the most optimal evaluation and treatment. In 2006, an international consensus statement provided guidance on laboratory testing for antiphospholipid syndrome limited to lupus anticoagulant, anticardiolipin IgG and IgM, and IgG and IgM anti–beta2-glycoprotein I assays.5 ASRM does not recommend additional thrombophilia tests as they are unproven causative factors of recurrent miscarriage.

6. Screening hysteroscopy

A standard infertility evaluation includes ovulation testing, assessment of fallopian tube patency, and a sperm analysis. In a subfertile women with a normal ultrasound or hysterosalpingogram in the basic fertility work‐up, a Cochrane data review concluded there is no definitive evidence for improved outcome with a screening hysteroscopy prior to IUI or IVF.6,7 Two large trials included in the Cochrane review, confirmed similar live birth rates whether or not hysteroscopy was performed before IVF. There may value in screening patients with recurrent implantation failure.

7. PGT-A for all

As the efficacy of the first generation of embryo preimplantation genetic testing, i.e., FISH (fluorescence in situ hybridization) was disproven, so has the same result been determined for PGT-A, specifically in women younger than 35.8 In an elegant randomized prospective trial, Munne and colleagues showed no improvement in the ongoing pregnancy rate (OPR) of study patients of all ages who were enrolled with the intention to treat. However, a subanalysis of patients aged 35-40 who completed the protocol did show an improved OPR and lower miscarriage rate per embryo transfer. While there is no evidence to support improved outcomes with the universal application of PGT-A, there may be some benefit in women older than 35 as well as in certain individual patient circumstances.

8. ICSI for nonmale factor infertility; assisted hatching

In an effort to reduce the risk of fertilization failure, programs have broadened the use of ICSI to nonmale factor infertility. While it has been used in PGT to reduce the risk of DNA contamination, particularly in PGT-M (monogenic disorder) and PGT-SR (structural rearrangement) cases, ICSI has not been shown to improve outcomes when there is a normal sperm analysis.9 During IVF embryo development, assisted hatching involves the thinning and/or opening of the zona pellucida either by chemical, mechanical, or laser means around the embryo before transfer with the intention of facilitating implantation. The routine use of assisted hatching is not recommended based on the lack of increase in live birth rates and because it may increase multiple pregnancy and monozygotic twinning rates.10

 

 

9. Acupuncture

Four meta-analyses showed no evidence of the overall benefit of acupuncture for improving live birth rates regardless of whether acupuncture was performed around the time of oocyte retrieval or around the day of embryo transfer. Consequently, acupuncture cannot be recommended routinely to improve IVF outcomes.11

10. Immunologic tests/treatments

Given the “foreign” genetic nature of a fetus, attempts to suppress the maternal immunologic response to sustain the pregnancy have been made for decades, especially for recurrent miscarriage and recurrent implantation failure with IVF. Testing has included natural killer (NK) cells, human leukocyte antigen (HLA) genotypes, and cytokines. While NK cells can be examined by endometrial biopsy, levels fluctuate based on the cycle phase, and no correlation between peripheral blood testing and uterine NK cell levels has been shown. Further, no consensus has been reached on reliable normal reference ranges in uterine NK cells.12

Several treatments have been proposed to somehow modulate the immune system during the implantation process thereby improving implantation and live birth, including lipid emulsion (intralipid) infusion, intravenous immunoglobulin, leukocyte immunization therapy, tacrolimus, anti–tumor necrosis factor agents, and granulocyte colony-stimulating factor. A recent systematic review and meta-analysis cited low-quality studies and did not recommend the use of any of these immune treatments.13 Further, immunomodulation has many known side effects, some of which are serious (including hepatosplenomegaly, thrombocytopenia, leukopenia, renal failure, thromboembolism, and anaphylactic reactions). Excluding women with autoimmune disease, taking glucocorticoids or other immune treatments to improve fertility has not been proven.13

Conclusion

To quote the New York Times opinion piece, “IVF remains an under-regulated arena, and entrepreneurial doctors and pharmaceutical and life science companies are eager to find new ways to cash in on a growing global market that is projected to be as large as $40 billion by 2024.” While this bold statement compels a huge “Ouch!”, it reminds us of our obligation to provide evidence-based medicine and to include emotional and financial harm to our oath of Primum non nocere.

References

1. The News York Times. 2019 Dec 12. Opinion.

2. Wilkinson J et al. Fertil Steril. 2019;112(6):973-7.

3. Lensen S et al. N Engl J Med. 2019 Jan 24;380(4):325-34.

4. Practice Committee of the American Society for Reproductive Medicine. Fertil Steril. 2020;113(2):305-22.

5. Miyakis S et al. J Thromb Haemost. 2006;4(2):295-306.

6. Kamath MS et al. Cochrane Database Syst Rev. 2019 Apr 16;4(4):CD012856.

7. Bosteels J et al. Cochrane Database Syst Rev. 2013 Jan 31;(1):CD009461.

8. Munne S et al. Fertil Steril. 2019;112(6):1071-9.

9. Practice Committees of the American Society for Reproductive Medicine and the Society for Assisted Reproductive Technology. Fertil Steril. 2020;114(2):239-45.

10. Lacey L et al. Cochrane Database Syst Rev. March 7 2021;3:2199.

11. Coyle ME et al. Acupunct Med. 2021;39(1):20-9.

12. Von Woon E et al. Hum Reprod Update. 2022;30;28(4):548-82.

13. Achilli C et al. Fertil Steril. 2018;110(6):1089-100.

In 2019, a New York Times opinion piece titled, “The Big IVF Add-On Racket – This is no way to treat patients desperate for a baby”1 alleged exploitation of infertility patients based on a Fertility and Sterility article, “Do à la carte menus serve infertility patients? The ethics and regulation of in vitro fertility add-ons.”2 The desperation of infertility patients combined with their financial burden, caused by inconsistent insurance coverage, has resulted in a perfect storm of frustration and overzealous recommendations for a successful outcome. Since the inception of in vitro fertilization (IVF) itself, infertility patients have been subjected to many unproven tests and procedures that enter the mainstream of care before unequivocal efficacy and safety have been shown.

From ovarian stimulation with intrauterine insemination (IUI) or IVF along with intracytoplasmic sperm injection (ICSI), assisted hatching, and preimplantation genetic testing for aneuploidy (PGT-A), a multitude of options with varying success can overwhelm fertility patients as they walk the tightrope of wanting “the kitchen sink” of treatment while experiencing sticker shock. This month’s article examines the top 10 infertility add-ons that have yet to be shown to improve pregnancy outcomes.

Dr. Mark P. Trolice

1. Blood testing: Prolactin and FSH

In a woman with ovulatory monthly menstrual cycles, a serum prolactin level provides no elucidation of the cause of infertility. If obtained following ovulation, prolactin can often be physiologically elevated, thereby compelling a repeat blood level, which is ideally performed during the early proliferative phase. False elevations of prolactin can be caused by an early morning blood sample, eating, and stress – which may result from worry caused by having to repeat the unnecessary initial blood test!

Follicle-stimulating hormone (FSH) was a first-line hormone test to assess for ovarian age. For nearly 15 years now, FSH has been replaced by anti-Müllerian hormone as a more reliable and earlier test for diminished ovarian reserve. However, FSH is still the hormone test of choice to diagnose primary ovarian insufficiency. Note that the use of ovarian age testing in a woman without infertility can result in both unnecessary patient anxiety and additional testing.
 

2. Endometrial scratch

The concept was understandable, that is, induce endometrial trauma by a biopsy or “scratch,” that results in an inflammatory and immunologic response to increase implantation. Endometrial sampling was recommended to be performed during the month prior to the embryo transfer cycle. While the procedure is brief, the pain response of women varies from minimal to severe. Unfortunately, a randomized controlled trial of over 1,300 patients did not show any improvement in the IVF live birth rate from the scratch procedure.3

3. Diagnostic laparoscopy

In years past, a diagnosis of unexplained infertility was not accepted until a laparoscopy was performed that revealed a normal pelvis. This approach subjected many women to an unindicated and a potentially risky surgery that has not shown benefit. The American Society for Reproductive Medicine’s ReproductiveFacts.org website states: “Routine diagnostic laparoscopy should not be performed unless there is a suspicion of pelvic pathology based on clinical history, an abnormal pelvic exam, or abnormalities identified with less invasive testing. In patients with a normal hysterosalpingogram or the presence of a unilaterally patent tube, diagnostic laparoscopy typically will not change the initial recommendation for treatment.”

 

 



4. Prescribing clomiphene citrate without IUI

Ovulation dysfunction is found in 40% of female factors for fertility. Provided testing reveals a reasonably normal sperm analysis and hysterosalpingogram, ovulation induction medication with ultrasound monitoring along with an hCG trigger is appropriate. In women who ovulate with unexplained infertility and/or mild male factor, the use of clomiphene citrate or letrozole with timed intercourse is often prescribed, particularly in clinics when IUI preparation is not available. Unfortunately, without including IUI, the use of oral ovarian stimulation has been shown by good evidence to be no more effective than natural cycle attempts at conception.4

5. Thrombophilia testing

Recurrent miscarriage, defined by the spontaneous loss of two or more pregnancies (often during the first trimester but may include up to 20 weeks estimated gestational age), has remained an ill-defined problem that lacks a consensus on the most optimal evaluation and treatment. In 2006, an international consensus statement provided guidance on laboratory testing for antiphospholipid syndrome limited to lupus anticoagulant, anticardiolipin IgG and IgM, and IgG and IgM anti–beta2-glycoprotein I assays.5 ASRM does not recommend additional thrombophilia tests as they are unproven causative factors of recurrent miscarriage.

6. Screening hysteroscopy

A standard infertility evaluation includes ovulation testing, assessment of fallopian tube patency, and a sperm analysis. In a subfertile women with a normal ultrasound or hysterosalpingogram in the basic fertility work‐up, a Cochrane data review concluded there is no definitive evidence for improved outcome with a screening hysteroscopy prior to IUI or IVF.6,7 Two large trials included in the Cochrane review, confirmed similar live birth rates whether or not hysteroscopy was performed before IVF. There may value in screening patients with recurrent implantation failure.

7. PGT-A for all

As the efficacy of the first generation of embryo preimplantation genetic testing, i.e., FISH (fluorescence in situ hybridization) was disproven, so has the same result been determined for PGT-A, specifically in women younger than 35.8 In an elegant randomized prospective trial, Munne and colleagues showed no improvement in the ongoing pregnancy rate (OPR) of study patients of all ages who were enrolled with the intention to treat. However, a subanalysis of patients aged 35-40 who completed the protocol did show an improved OPR and lower miscarriage rate per embryo transfer. While there is no evidence to support improved outcomes with the universal application of PGT-A, there may be some benefit in women older than 35 as well as in certain individual patient circumstances.

8. ICSI for nonmale factor infertility; assisted hatching

In an effort to reduce the risk of fertilization failure, programs have broadened the use of ICSI to nonmale factor infertility. While it has been used in PGT to reduce the risk of DNA contamination, particularly in PGT-M (monogenic disorder) and PGT-SR (structural rearrangement) cases, ICSI has not been shown to improve outcomes when there is a normal sperm analysis.9 During IVF embryo development, assisted hatching involves the thinning and/or opening of the zona pellucida either by chemical, mechanical, or laser means around the embryo before transfer with the intention of facilitating implantation. The routine use of assisted hatching is not recommended based on the lack of increase in live birth rates and because it may increase multiple pregnancy and monozygotic twinning rates.10

 

 

9. Acupuncture

Four meta-analyses showed no evidence of the overall benefit of acupuncture for improving live birth rates regardless of whether acupuncture was performed around the time of oocyte retrieval or around the day of embryo transfer. Consequently, acupuncture cannot be recommended routinely to improve IVF outcomes.11

10. Immunologic tests/treatments

Given the “foreign” genetic nature of a fetus, attempts to suppress the maternal immunologic response to sustain the pregnancy have been made for decades, especially for recurrent miscarriage and recurrent implantation failure with IVF. Testing has included natural killer (NK) cells, human leukocyte antigen (HLA) genotypes, and cytokines. While NK cells can be examined by endometrial biopsy, levels fluctuate based on the cycle phase, and no correlation between peripheral blood testing and uterine NK cell levels has been shown. Further, no consensus has been reached on reliable normal reference ranges in uterine NK cells.12

Several treatments have been proposed to somehow modulate the immune system during the implantation process thereby improving implantation and live birth, including lipid emulsion (intralipid) infusion, intravenous immunoglobulin, leukocyte immunization therapy, tacrolimus, anti–tumor necrosis factor agents, and granulocyte colony-stimulating factor. A recent systematic review and meta-analysis cited low-quality studies and did not recommend the use of any of these immune treatments.13 Further, immunomodulation has many known side effects, some of which are serious (including hepatosplenomegaly, thrombocytopenia, leukopenia, renal failure, thromboembolism, and anaphylactic reactions). Excluding women with autoimmune disease, taking glucocorticoids or other immune treatments to improve fertility has not been proven.13

Conclusion

To quote the New York Times opinion piece, “IVF remains an under-regulated arena, and entrepreneurial doctors and pharmaceutical and life science companies are eager to find new ways to cash in on a growing global market that is projected to be as large as $40 billion by 2024.” While this bold statement compels a huge “Ouch!”, it reminds us of our obligation to provide evidence-based medicine and to include emotional and financial harm to our oath of Primum non nocere.

References

1. The News York Times. 2019 Dec 12. Opinion.

2. Wilkinson J et al. Fertil Steril. 2019;112(6):973-7.

3. Lensen S et al. N Engl J Med. 2019 Jan 24;380(4):325-34.

4. Practice Committee of the American Society for Reproductive Medicine. Fertil Steril. 2020;113(2):305-22.

5. Miyakis S et al. J Thromb Haemost. 2006;4(2):295-306.

6. Kamath MS et al. Cochrane Database Syst Rev. 2019 Apr 16;4(4):CD012856.

7. Bosteels J et al. Cochrane Database Syst Rev. 2013 Jan 31;(1):CD009461.

8. Munne S et al. Fertil Steril. 2019;112(6):1071-9.

9. Practice Committees of the American Society for Reproductive Medicine and the Society for Assisted Reproductive Technology. Fertil Steril. 2020;114(2):239-45.

10. Lacey L et al. Cochrane Database Syst Rev. March 7 2021;3:2199.

11. Coyle ME et al. Acupunct Med. 2021;39(1):20-9.

12. Von Woon E et al. Hum Reprod Update. 2022;30;28(4):548-82.

13. Achilli C et al. Fertil Steril. 2018;110(6):1089-100.

Publications
Publications
Topics
Article Type
Sections
Teambase XML
<?xml version="1.0" encoding="UTF-8"?>
<!--$RCSfile: InCopy_agile.xsl,v $ $Revision: 1.35 $-->
<!--$RCSfile: drupal.xsl,v $ $Revision: 1.7 $-->
<root generator="drupal.xsl" gversion="1.7"> <header> <fileName>161113</fileName> <TBEID>0C046B78.SIG</TBEID> <TBUniqueIdentifier>MD_0C046B78</TBUniqueIdentifier> <newsOrJournal>News</newsOrJournal> <publisherName>Frontline Medical Communications</publisherName> <storyname/> <articleType>353</articleType> <TBLocation>QC Done-All Pubs</TBLocation> <QCDate>20221121T134807</QCDate> <firstPublished>20221121T133837</firstPublished> <LastPublished>20221121T133837</LastPublished> <pubStatus qcode="stat:"/> <embargoDate/> <killDate/> <CMSDate>20221121T133836</CMSDate> <articleSource/> <facebookInfo/> <meetingNumber/> <byline>Mark P Trolice</byline> <bylineText>MARK P. TROLICE, MD</bylineText> <bylineFull>MARK P. TROLICE, MD</bylineFull> <bylineTitleText/> <USOrGlobal/> <wireDocType/> <newsDocType/> <journalDocType/> <linkLabel/> <pageRange/> <citation/> <quizID/> <indexIssueDate/> <itemClass qcode="ninat:text"/> <provider qcode="provider:imng"> <name>IMNG Medical Media</name> <rightsInfo> <copyrightHolder> <name>Frontline Medical News</name> </copyrightHolder> <copyrightNotice>Copyright (c) 2015 Frontline Medical News, a Frontline Medical Communications Inc. company. All rights reserved. This material may not be published, broadcast, copied, or otherwise reproduced or distributed without the prior written permission of Frontline Medical Communications Inc.</copyrightNotice> </rightsInfo> </provider> <abstract/> <metaDescription>In 2019, a New York Times opinion piece titled, “The Big IVF Add-On Racket – This is no way to treat patients desperate for a baby”1 alleged exploitation of inf</metaDescription> <articlePDF/> <teaserImage>284599</teaserImage> <teaser>Since the inception of in vitro fertilization itself, infertility patients have been subjected to many unproven tests and procedures that enter the mainstream of care before unequivocal efficacy and safety have been shown.</teaser> <title>Top 10 unproven infertility tests and treatments</title> <deck/> <disclaimer/> <AuthorList/> <articleURL/> <doi/> <pubMedID/> <publishXMLStatus/> <publishXMLVersion>1</publishXMLVersion> <useEISSN>0</useEISSN> <urgency/> <pubPubdateYear/> <pubPubdateMonth/> <pubPubdateDay/> <pubVolume/> <pubNumber/> <wireChannels/> <primaryCMSID/> <CMSIDs/> <keywords/> <seeAlsos/> <publications_g> <publicationData> <publicationCode>ob</publicationCode> <pubIssueName/> <pubArticleType/> <pubTopics/> <pubCategories/> <pubSections/> </publicationData> </publications_g> <publications> <term canonical="true">23</term> </publications> <sections> <term canonical="true">68807</term> </sections> <topics> <term canonical="true">287</term> </topics> <links> <link> <itemClass qcode="ninat:picture"/> <altRep contenttype="image/jpeg">images/24010881.jpg</altRep> <description role="drol:caption">Dr. Mark P. Trolice</description> <description role="drol:credit"/> </link> </links> </header> <itemSet> <newsItem> <itemMeta> <itemRole>Main</itemRole> <itemClass>text</itemClass> <title>Top 10 unproven infertility tests and treatments</title> <deck/> </itemMeta> <itemContent> <p>In 2019, a New York Times opinion piece titled, “The Big IVF Add-On Racket – This is no way to treat patients desperate for a baby”<sup>1</sup> alleged exploitation of infertility patients based on a Fertility and Sterility article, “Do à la carte menus serve infertility patients? The ethics and regulation of in vitro fertility add-ons.”<sup>2 </sup>The desperation of infertility patients combined with their financial burden, caused by inconsistent insurance coverage, has resulted in a perfect storm of frustration and overzealous recommendations for a successful outcome. Since the inception of in vitro fertilization (IVF) itself, infertility patients have been subjected to many unproven tests and procedures that enter the mainstream of care before unequivocal efficacy and safety have been shown.</p> <p>From ovarian stimulation with intrauterine insemination (IUI) or IVF along with intracytoplasmic sperm injection (ICSI), assisted hatching, and preimplantation genetic testing for aneuploidy (PGT-A), a multitude of options with varying success can overwhelm fertility patients as they walk the tightrope of wanting “the kitchen sink” of treatment while experiencing sticker shock. This month’s article examines the top 10 infertility add-ons that have yet to be shown to improve pregnancy outcomes.<br/><br/>[[{"fid":"284599","view_mode":"medstat_image_flush_left","fields":{"format":"medstat_image_flush_left","field_file_image_alt_text[und][0][value]":"Dr. Trolice is director of The IVF Center in Winter Park, Fla., and professor of obstetrics and gynecology at the University of Central Florida, Orlando.","field_file_image_credit[und][0][value]":"","field_file_image_caption[und][0][value]":"Dr. Mark P. Trolice"},"type":"media","attributes":{"class":"media-element file-medstat_image_flush_left"}}]]</p> <h2>1. Blood testing: Prolactin and FSH</h2> <p>In a woman with ovulatory monthly menstrual cycles, a serum prolactin level provides no elucidation of the cause of infertility. If obtained following ovulation, prolactin can often be physiologically elevated, thereby compelling a repeat blood level, which is ideally performed during the early proliferative phase. False elevations of prolactin can be caused by an early morning blood sample, eating, and stress – which may result from worry caused by having to repeat the unnecessary initial blood test!</p> <p>Follicle-stimulating hormone (FSH) was a first-line hormone test to assess for ovarian age. For nearly 15 years now, FSH has been replaced by anti-Müllerian hormone as a more reliable and earlier test for diminished ovarian reserve. However, FSH is still the hormone test of choice to diagnose primary ovarian insufficiency. Note that the use of ovarian age testing in a woman without infertility can result in both unnecessary patient anxiety and additional testing.<br/><br/></p> <h2>2. Endometrial scratch</h2> <p>The concept was understandable, that is, induce endometrial trauma by a biopsy or “scratch,” that results in an inflammatory and immunologic response to increase implantation. Endometrial sampling was recommended to be performed during the month prior to the embryo transfer cycle. While the procedure is brief, the pain response of women varies from minimal to severe. Unfortunately, a randomized controlled trial of over 1,300 patients did not show any improvement in the IVF live birth rate from the scratch procedure.<sup>3</sup></p> <h2>3. Diagnostic laparoscopy</h2> <p>In years past, a diagnosis of unexplained infertility was not accepted until a laparoscopy was performed that revealed a normal pelvis. This approach subjected many women to an unindicated and a potentially risky surgery that has not shown benefit. The American Society for Reproductive Medicine’s ReproductiveFacts.org website states: “Routine diagnostic laparoscopy should not be performed unless there is a suspicion of pelvic pathology based on clinical history, an abnormal pelvic exam, or abnormalities identified with less invasive testing. In patients with a normal hysterosalpingogram or the presence of a unilaterally patent tube, diagnostic laparoscopy typically will not change the initial recommendation for treatment.”</p> <h2><br/><br/>4. Prescribing clomiphene citrate without IUI</h2> <p>Ovulation dysfunction is found in 40% of female factors for fertility. Provided testing reveals a reasonably normal sperm analysis and hysterosalpingogram, ovulation induction medication with ultrasound monitoring along with an hCG trigger is appropriate. In women who ovulate with unexplained infertility and/or mild male factor, the use of clomiphene citrate or letrozole with timed intercourse is often prescribed, particularly in clinics when IUI preparation is not available. Unfortunately, without including IUI, the use of oral ovarian stimulation has been shown by good evidence to be no more effective than natural cycle attempts at conception.<sup>4</sup></p> <h2>5. Thrombophilia testing</h2> <p>Recurrent miscarriage, defined by the spontaneous loss of two or more pregnancies (often during the first trimester but may include up to 20 weeks estimated gestational age), has remained an ill-defined problem that lacks a consensus on the most optimal evaluation and treatment. In 2006, an international consensus statement provided guidance on laboratory testing for antiphospholipid syndrome limited to lupus anticoagulant, anticardiolipin IgG and IgM, and IgG and IgM anti–beta<sub>2</sub>-glycoprotein I assays.<sup>5</sup> ASRM does not recommend additional thrombophilia tests as they are unproven causative factors of recurrent miscarriage.</p> <h2>6. Screening hysteroscopy</h2> <p>A standard infertility evaluation includes ovulation testing, assessment of fallopian tube patency, and a sperm analysis. In a subfertile women with a normal ultrasound or hysterosalpingogram in the basic fertility work‐up, a Cochrane data review concluded there is no definitive evidence for improved outcome with a screening hysteroscopy prior to IUI or IVF.<sup>6,7 </sup>Two large trials included in the Cochrane review, confirmed similar live birth rates whether or not hysteroscopy was performed before IVF. There may value in screening patients with recurrent implantation failure. </p> <h2>7. PGT-A for all</h2> <p>As the efficacy of the first generation of embryo preimplantation genetic testing, i.e., FISH (fluorescence in situ hybridization) was disproven, so has the same result been determined for PGT-A, specifically in women younger than 35.<sup>8</sup> In an elegant randomized prospective trial, Munne and colleagues showed no improvement in the ongoing pregnancy rate (OPR) of study patients of all ages who were enrolled with the intention to treat. However, a subanalysis of patients aged 35-40 who completed the protocol did show an improved OPR and lower miscarriage rate per embryo transfer. While there is no evidence to support improved outcomes with the universal application of PGT-A, there may be some benefit in women older than 35 as well as in certain individual patient circumstances.</p> <h2>8. ICSI for nonmale factor infertility; assisted hatching</h2> <p>In an effort to reduce the risk of fertilization failure, programs have broadened the use of ICSI to nonmale factor infertility. While it has been used in PGT to reduce the risk of DNA contamination, particularly in PGT-M (monogenic disorder) and PGT-SR (structural rearrangement) cases, ICSI has not been shown to improve outcomes when there is a normal sperm analysis.<sup>9</sup> During IVF embryo development, assisted hatching involves the thinning and/or opening of the zona pellucida either by chemical, mechanical, or laser means around the embryo before transfer with the intention of facilitating implantation. The routine use of assisted hatching is not recommended based on the lack of increase in live birth rates and because it may increase multiple pregnancy and monozygotic twinning rates.<sup>10</sup></p> <h2>9. Acupuncture</h2> <p>Four meta-analyses showed no evidence of the overall benefit of acupuncture for improving live birth rates regardless of whether acupuncture was performed around the time of oocyte retrieval or around the day of embryo transfer. Consequently, acupuncture cannot be recommended routinely to improve IVF outcomes.<sup>11</sup></p> <h2>10. Immunologic tests/treatments</h2> <p>Given the “foreign” genetic nature of a fetus, attempts to suppress the maternal immunologic response to sustain the pregnancy have been made for decades, especially for recurrent miscarriage and recurrent implantation failure with IVF. Testing has included natural killer (NK) cells, human leukocyte antigen (HLA) genotypes, and cytokines. While NK cells can be examined by endometrial biopsy, levels fluctuate based on the cycle phase, and no correlation between peripheral blood testing and uterine NK cell levels has been shown. Further, no consensus has been reached on reliable normal reference ranges in uterine NK cells.<sup>12</sup></p> <p>Several treatments have been proposed to somehow modulate the immune system during the implantation process thereby improving implantation and live birth, including lipid emulsion (intralipid) infusion, intravenous immunoglobulin, leukocyte immunization therapy, tacrolimus, anti–tumor necrosis factor agents, and granulocyte colony-stimulating factor. A recent systematic review and meta-analysis cited low-quality studies and did not recommend the use of any of these immune treatments.<sup>13</sup> Further, immunomodulation has many known side effects, some of which are serious (including hepatosplenomegaly, thrombocytopenia, leukopenia, renal failure, thromboembolism, and anaphylactic reactions). Excluding women with autoimmune disease, taking glucocorticoids or other immune treatments to improve fertility has not been proven.<sup>13</sup></p> <h2>Conclusion</h2> <p>To quote the New York Times opinion piece, “IVF remains an under-regulated arena, and entrepreneurial doctors and pharmaceutical and life science companies are eager to find new ways to cash in on a growing global market that is projected to be as large as $40 billion by 2024.” While this bold statement compels a huge “Ouch!”, it reminds us of our obligation to provide evidence-based medicine and to include emotional and financial harm to our oath of Primum non nocere.</p> <h2>References</h2> <p>1. <span class="Hyperlink"><a href="https://www.nytimes.com/2019/12/12/opinion/ivf-add-ons.html">The News York Times. 2019 Dec 12. Opinion</a></span>. <br/><br/>2. Wilkinson J et al. <span class="Hyperlink"><a href="https://www.fertstert.org/article/S0015-0282(19)32454-9/fulltext">Fertil Steril. 2019;112(6):973-7</a></span>.<br/><br/>3. Lensen S et al. <span class="Hyperlink"><a href="https://www.nejm.org/doi/10.1056/NEJMoa1808737">N Engl J Med. 2019 Jan 24;380(4):325-34</a></span>.<br/><br/>4. Practice Committee of the American Society for Reproductive Medicine. <span class="Hyperlink"><a href="https://www.fertstert.org/article/S0015-0282(19)32484-7/fulltext">Fertil Steril. 2020;113(2):305-22</a></span>.<br/><br/>5. Miyakis S et al. <span class="Hyperlink"><a href="https://onlinelibrary.wiley.com/doi/10.1111/j.1538-7836.2006.01753.x">J Thromb Haemost. 2006;4(2):295-306</a></span>.<br/><br/>6. Kamath MS et al. <span class="Hyperlink"><a href="https://www.cochranelibrary.com/cdsr/doi/10.1002/14651858.CD012856.pub2/full">Cochrane Database Syst Rev. 2019 Apr 16;4(4):CD012856</a></span>. <br/><br/>7. Bosteels J et al. <span class="Hyperlink"><a href="https://www.cochranelibrary.com/cdsr/doi/10.1002/14651858.CD009461.pub2/full">Cochrane Database Syst Rev. 2013 Jan 31;(1):CD009461</a></span>.<br/><br/>8. Munne S et al. <span class="Hyperlink"><a href="https://www.fertstert.org/article/S0015-0282(19)31979-X/fulltext">Fertil Steril. 2019;112(6):1071-9</a></span>.<br/><br/>9. Practice Committees of the American Society for Reproductive Medicine and the Society for Assisted Reproductive Technology. <span class="Hyperlink"><a href="https://www.fertstert.org/article/S0015-0282(20)30523-9/fulltext">Fertil Steril. 2020;114(2):239-45</a></span>.<br/><br/>10. Lacey L et al. <span class="Hyperlink"><a href="https://www.cochranelibrary.com/cdsr/doi/10.1002/14651858.CD001894.pub6/full">Cochrane Database Syst Rev. March 7 2021;3:2199</a></span>.<br/><br/>11. Coyle ME et al. <span class="Hyperlink"><a href="https://journals.sagepub.com/doi/10.1177/0964528420958711">Acupunct Med. 2021;39(1):20-9</a></span>.<br/><br/>12. Von Woon E et al. <span class="Hyperlink"><a href="https://academic.oup.com/humupd/article/28/4/548/6545823?login=false">Hum Reprod Update. 2022;30;28(4):548-82.</a></span><br/><br/>13. Achilli C et al. <span class="Hyperlink"><a href="https://www.fertstert.org/article/S0015-0282(18)30569-7/fulltext">Fertil Steril. 2018;110(6):1089-100</a></span>.</p> </itemContent> </newsItem> <newsItem> <itemMeta> <itemRole>teaser</itemRole> <itemClass>text</itemClass> <title/> <deck/> </itemMeta> <itemContent> </itemContent> </newsItem> </itemSet></root>
Disallow All Ads
Content Gating
No Gating (article Unlocked/Free)
Alternative CME
Disqus Comments
Default
Use ProPublica
Hide sidebar & use full width
render the right sidebar.
Conference Recap Checkbox
Not Conference Recap
Clinical Edge
Display the Slideshow in this Article
Medscape Article
Display survey writer
Reuters content
Disable Inline Native ads
WebMD Article

BMI and reproduction – weighing the evidence

Article Type
Changed
Thu, 10/27/2022 - 15:17

Arguably, no topic during an infertility consultation generates more of an emotional reaction than discussing body mass index (BMI), particularly when it is high. Patients have become increasingly sensitive to weight discussions with their physicians because of concerns about body shaming. Among patients with an elevated BMI, criticism on social media of health care professionals’ counseling and a preemptive presentation of “Don’t Weigh Me” cards have become popular responses. Despite the medical evidence on impaired reproduction with an abnormal BMI, patients are choosing to forgo the topic. Research has demonstrated “extensive evidence [of] strong weight bias” in a wide range of health staff.1 A “viral” TikTok study revealed that medical “gaslighting” founded in weight stigma and bias is harmful, as reported on KevinMD.com.2 This month, we review the effect of abnormal BMI, both high and low, on reproduction and pregnancy.

160453_pho1_web.jpg

A method to assess relative weight was first described in 1832 as its ratio in kilograms divided by the square of the height in meters, or the Quetelet Index. The search for a functional assessment of relative body weight began after World War II when reports by actuaries noted the increased mortality of overweight policyholders. The relationship between weight and cardiovascular disease was further revealed in epidemiologic studies. The Quetelet Index became the BMI in 1972.3

Weight measurement is a mainstay in the assessment of a patient’s vital signs along with blood pressure, pulse rate, respiration rate, and temperature. Weight is vital to the calculation of medication dosage – for instance, administration of conscious sedative drugs, methotrexate, and gonadotropins. Some state boards of medicine, such as Florida, have a limitation on patient BMI at office-based surgery centers (40 kg/m2).
 

Obesity is a disease

As reported by the World Health Organization in 2022, the disease of obesity is an epidemic afflicting more than 1 billion people worldwide, or 1 in 8 individuals globally.4 The health implications of an elevated BMI include increased mortality, diabetes, heart disease, and stroke, physical limitations to activities of daily living, and complications affecting reproduction.

Female obesity is related to poorer outcomes in natural and assisted conception, including an increased risk of miscarriage. Compared with normal-weight women, those with obesity are three times more likely to have ovulatory dysfunction,5 infertility,6 a lower chance for conception,7 higher rate of miscarriage, and low birth weight.8,9During pregnancy, women with obesity have three to four times higher rates of gestational diabetes and preeclampsia,10 as well as likelihood of delivering preterm,11 having a fetus with macrosomia and birth defects, and a 1.3- to 2.1-times higher risk of stillbirth.12

Obesity is present in 40%-80% of women with polycystic ovary syndrome,13 the most common cause of ovulatory dysfunction from dysregulation of the hypothalamic-pituitary-ovarian axis. While PCOS is associated with reproductive and metabolic consequences, even in regularly ovulating women, increasing obesity appears to be associated with decreasing spontaneous pregnancy rates and increased time to pregnancy.14

Obesity and IVF

Women with obesity have reduced success with assisted reproductive technology, an increased number of canceled cycles, and poorer quality oocytes retrieved. A prospective cohort study of nearly 2,000 women reported that every 5 kg of body weight increase (from the patient’s baseline weight at age 18) was associated with a 5% increase in the mean duration of time required for conception (95% confidence interval, 3%-7%).15 Given that approximately 90% of these women had regular menstrual cycles, ovulatory dysfunction was not the suspected pathophysiology.

A meta-analysis of 21 cohort studies reported a lower likelihood of live birth following in vitro fertilization for women with obesity, compared with normal-weight women (risk ratio, 0.85; 95% CI, 0.82-0.87).16 A further subgroup analysis that evaluated only women with PCOS showed a reduction in the live birth rate following IVF for individuals with obesity, compared with normal-weight individuals (RR, 0.78; 95% CI, 0.74-0.82).

In a retrospective study of almost 500,000 fresh autologous IVF cycles, women with obesity had a 6% reduction in pregnancy rates and a 13% reduction in live birth rates, compared with normal-weight women. Both high and low BMI were associated with an increased risk of low birth weight and preterm delivery.17 The live birth rates per transfer for normal-weight and higher-weight women were 38% and 33%, respectively.

Contrarily, a randomized controlled trial showed that an intensive weight-reduction program resulted in a large weight loss but did not substantially affect live birth rates in women with obesity scheduled for IVF.18

Low BMI

A noteworthy cause of low BMI is functional hypothalamic amenorrhea (FHA), a disorder with low energy availability either from decreased caloric intake and/or excessive energy expenditure associated with eating disorders, excessive exercise, and stress. Consequently, a reduced GnRH drive results in a decreased pulse frequency and amplitude leading to low levels of follicle-stimulating hormone and luteinizing hormone, resulting in anovulation. Correction of lifestyle behaviors related to FHA can restore menstrual cycles. After normal weight is achieved, it appears unlikely that fertility is affected.19 In 47% of adolescent patients with anorexia, menses spontaneously returned within the first 12 months after admission, with an improved prognosis in secondary over primary amenorrhea.20,21 Interestingly, mildly and significantly underweight infertile women have pregnancy and live birth rates similar to normal-weight patients after IVF treatment.22

Pregnancy is complicated in underweight women, resulting in an increased risk of anemia, fetal growth retardation, and low birth weight, as well as preterm birth.21

Take-home message

The extremes of BMI both impair natural reproduction. Elevated BMI reduces success with IVF but rapid weight loss prior to IVF does not improve outcomes. A normal BMI is the goal for optimal reproductive and pregnancy health.

Dr. Trolice is director of the IVF Center in Winter Park, Fla., and professor of obstetrics and gynecology at the University of Central Florida, Orlando.
 

References

1. Talumaa B et al. Obesity Rev. 2022;23:e13494.

2. https://bit.ly/3rHCivE.

3. Eknoyan G. Nephrol Dial Transplant. 2008;23:47-51.

4. Wells JCK. Dis Models Mech. 2012;5:595-607.

5. Brewer CJ and Balen AH. Reproduction. 2010;140:347-64.

6. Silvestris E et al. Reprod Biol Endocrinol. 2018;16:22.

7. Wise LA et al. Hum Reprod. 2010;25:253-64.

8. Bellver J. Curr Opin Obstet Gynecol. 2022;34:114-21.

9.
Dickey RP et al. Am J Obstet Gynecol. 2013;209:349.e1.

10. Alwash SM et al. Obes Res Clin Pract. 2021;15:425-30
.

11. Cnattingius S et al. JAMA. 2013;309:2362-70.

12. Aune D et al. JAMA. 2014;311:1536-46.

13. Sam S. Obes Manag. 2007;3:69-73.

14. van der Steeg JW et al. Hum Reprod. 2008;23:324-8.

15. Gaskins AJ et al. Obstet Gynecol. 2015;126:850-8.

16. Sermondade N et al. Hum Reprod Update. 2019;25:439-519.

17. Kawwass JF et al. Fertil Steril. 2016;106[7]:1742-50.

18. Einarsson S et al. Hum Reprod. 2017;32:1621-30.

19. Chaer R et al. Diseases. 2020;8:46.

20. Dempfle A et al. Psychiatry. 2013;13:308.

21. Verma A and Shrimali L. J Clin Diagn Res. 2012;6:1531-3.

22. Romanski PA et al. Reprod Biomed Online. 2020;42:366-74.

Publications
Topics
Sections

Arguably, no topic during an infertility consultation generates more of an emotional reaction than discussing body mass index (BMI), particularly when it is high. Patients have become increasingly sensitive to weight discussions with their physicians because of concerns about body shaming. Among patients with an elevated BMI, criticism on social media of health care professionals’ counseling and a preemptive presentation of “Don’t Weigh Me” cards have become popular responses. Despite the medical evidence on impaired reproduction with an abnormal BMI, patients are choosing to forgo the topic. Research has demonstrated “extensive evidence [of] strong weight bias” in a wide range of health staff.1 A “viral” TikTok study revealed that medical “gaslighting” founded in weight stigma and bias is harmful, as reported on KevinMD.com.2 This month, we review the effect of abnormal BMI, both high and low, on reproduction and pregnancy.

160453_pho1_web.jpg

A method to assess relative weight was first described in 1832 as its ratio in kilograms divided by the square of the height in meters, or the Quetelet Index. The search for a functional assessment of relative body weight began after World War II when reports by actuaries noted the increased mortality of overweight policyholders. The relationship between weight and cardiovascular disease was further revealed in epidemiologic studies. The Quetelet Index became the BMI in 1972.3

Weight measurement is a mainstay in the assessment of a patient’s vital signs along with blood pressure, pulse rate, respiration rate, and temperature. Weight is vital to the calculation of medication dosage – for instance, administration of conscious sedative drugs, methotrexate, and gonadotropins. Some state boards of medicine, such as Florida, have a limitation on patient BMI at office-based surgery centers (40 kg/m2).
 

Obesity is a disease

As reported by the World Health Organization in 2022, the disease of obesity is an epidemic afflicting more than 1 billion people worldwide, or 1 in 8 individuals globally.4 The health implications of an elevated BMI include increased mortality, diabetes, heart disease, and stroke, physical limitations to activities of daily living, and complications affecting reproduction.

Female obesity is related to poorer outcomes in natural and assisted conception, including an increased risk of miscarriage. Compared with normal-weight women, those with obesity are three times more likely to have ovulatory dysfunction,5 infertility,6 a lower chance for conception,7 higher rate of miscarriage, and low birth weight.8,9During pregnancy, women with obesity have three to four times higher rates of gestational diabetes and preeclampsia,10 as well as likelihood of delivering preterm,11 having a fetus with macrosomia and birth defects, and a 1.3- to 2.1-times higher risk of stillbirth.12

Obesity is present in 40%-80% of women with polycystic ovary syndrome,13 the most common cause of ovulatory dysfunction from dysregulation of the hypothalamic-pituitary-ovarian axis. While PCOS is associated with reproductive and metabolic consequences, even in regularly ovulating women, increasing obesity appears to be associated with decreasing spontaneous pregnancy rates and increased time to pregnancy.14

Obesity and IVF

Women with obesity have reduced success with assisted reproductive technology, an increased number of canceled cycles, and poorer quality oocytes retrieved. A prospective cohort study of nearly 2,000 women reported that every 5 kg of body weight increase (from the patient’s baseline weight at age 18) was associated with a 5% increase in the mean duration of time required for conception (95% confidence interval, 3%-7%).15 Given that approximately 90% of these women had regular menstrual cycles, ovulatory dysfunction was not the suspected pathophysiology.

A meta-analysis of 21 cohort studies reported a lower likelihood of live birth following in vitro fertilization for women with obesity, compared with normal-weight women (risk ratio, 0.85; 95% CI, 0.82-0.87).16 A further subgroup analysis that evaluated only women with PCOS showed a reduction in the live birth rate following IVF for individuals with obesity, compared with normal-weight individuals (RR, 0.78; 95% CI, 0.74-0.82).

In a retrospective study of almost 500,000 fresh autologous IVF cycles, women with obesity had a 6% reduction in pregnancy rates and a 13% reduction in live birth rates, compared with normal-weight women. Both high and low BMI were associated with an increased risk of low birth weight and preterm delivery.17 The live birth rates per transfer for normal-weight and higher-weight women were 38% and 33%, respectively.

Contrarily, a randomized controlled trial showed that an intensive weight-reduction program resulted in a large weight loss but did not substantially affect live birth rates in women with obesity scheduled for IVF.18

Low BMI

A noteworthy cause of low BMI is functional hypothalamic amenorrhea (FHA), a disorder with low energy availability either from decreased caloric intake and/or excessive energy expenditure associated with eating disorders, excessive exercise, and stress. Consequently, a reduced GnRH drive results in a decreased pulse frequency and amplitude leading to low levels of follicle-stimulating hormone and luteinizing hormone, resulting in anovulation. Correction of lifestyle behaviors related to FHA can restore menstrual cycles. After normal weight is achieved, it appears unlikely that fertility is affected.19 In 47% of adolescent patients with anorexia, menses spontaneously returned within the first 12 months after admission, with an improved prognosis in secondary over primary amenorrhea.20,21 Interestingly, mildly and significantly underweight infertile women have pregnancy and live birth rates similar to normal-weight patients after IVF treatment.22

Pregnancy is complicated in underweight women, resulting in an increased risk of anemia, fetal growth retardation, and low birth weight, as well as preterm birth.21

Take-home message

The extremes of BMI both impair natural reproduction. Elevated BMI reduces success with IVF but rapid weight loss prior to IVF does not improve outcomes. A normal BMI is the goal for optimal reproductive and pregnancy health.

Dr. Trolice is director of the IVF Center in Winter Park, Fla., and professor of obstetrics and gynecology at the University of Central Florida, Orlando.
 

References

1. Talumaa B et al. Obesity Rev. 2022;23:e13494.

2. https://bit.ly/3rHCivE.

3. Eknoyan G. Nephrol Dial Transplant. 2008;23:47-51.

4. Wells JCK. Dis Models Mech. 2012;5:595-607.

5. Brewer CJ and Balen AH. Reproduction. 2010;140:347-64.

6. Silvestris E et al. Reprod Biol Endocrinol. 2018;16:22.

7. Wise LA et al. Hum Reprod. 2010;25:253-64.

8. Bellver J. Curr Opin Obstet Gynecol. 2022;34:114-21.

9.
Dickey RP et al. Am J Obstet Gynecol. 2013;209:349.e1.

10. Alwash SM et al. Obes Res Clin Pract. 2021;15:425-30
.

11. Cnattingius S et al. JAMA. 2013;309:2362-70.

12. Aune D et al. JAMA. 2014;311:1536-46.

13. Sam S. Obes Manag. 2007;3:69-73.

14. van der Steeg JW et al. Hum Reprod. 2008;23:324-8.

15. Gaskins AJ et al. Obstet Gynecol. 2015;126:850-8.

16. Sermondade N et al. Hum Reprod Update. 2019;25:439-519.

17. Kawwass JF et al. Fertil Steril. 2016;106[7]:1742-50.

18. Einarsson S et al. Hum Reprod. 2017;32:1621-30.

19. Chaer R et al. Diseases. 2020;8:46.

20. Dempfle A et al. Psychiatry. 2013;13:308.

21. Verma A and Shrimali L. J Clin Diagn Res. 2012;6:1531-3.

22. Romanski PA et al. Reprod Biomed Online. 2020;42:366-74.

Arguably, no topic during an infertility consultation generates more of an emotional reaction than discussing body mass index (BMI), particularly when it is high. Patients have become increasingly sensitive to weight discussions with their physicians because of concerns about body shaming. Among patients with an elevated BMI, criticism on social media of health care professionals’ counseling and a preemptive presentation of “Don’t Weigh Me” cards have become popular responses. Despite the medical evidence on impaired reproduction with an abnormal BMI, patients are choosing to forgo the topic. Research has demonstrated “extensive evidence [of] strong weight bias” in a wide range of health staff.1 A “viral” TikTok study revealed that medical “gaslighting” founded in weight stigma and bias is harmful, as reported on KevinMD.com.2 This month, we review the effect of abnormal BMI, both high and low, on reproduction and pregnancy.

160453_pho1_web.jpg

A method to assess relative weight was first described in 1832 as its ratio in kilograms divided by the square of the height in meters, or the Quetelet Index. The search for a functional assessment of relative body weight began after World War II when reports by actuaries noted the increased mortality of overweight policyholders. The relationship between weight and cardiovascular disease was further revealed in epidemiologic studies. The Quetelet Index became the BMI in 1972.3

Weight measurement is a mainstay in the assessment of a patient’s vital signs along with blood pressure, pulse rate, respiration rate, and temperature. Weight is vital to the calculation of medication dosage – for instance, administration of conscious sedative drugs, methotrexate, and gonadotropins. Some state boards of medicine, such as Florida, have a limitation on patient BMI at office-based surgery centers (40 kg/m2).
 

Obesity is a disease

As reported by the World Health Organization in 2022, the disease of obesity is an epidemic afflicting more than 1 billion people worldwide, or 1 in 8 individuals globally.4 The health implications of an elevated BMI include increased mortality, diabetes, heart disease, and stroke, physical limitations to activities of daily living, and complications affecting reproduction.

Female obesity is related to poorer outcomes in natural and assisted conception, including an increased risk of miscarriage. Compared with normal-weight women, those with obesity are three times more likely to have ovulatory dysfunction,5 infertility,6 a lower chance for conception,7 higher rate of miscarriage, and low birth weight.8,9During pregnancy, women with obesity have three to four times higher rates of gestational diabetes and preeclampsia,10 as well as likelihood of delivering preterm,11 having a fetus with macrosomia and birth defects, and a 1.3- to 2.1-times higher risk of stillbirth.12

Obesity is present in 40%-80% of women with polycystic ovary syndrome,13 the most common cause of ovulatory dysfunction from dysregulation of the hypothalamic-pituitary-ovarian axis. While PCOS is associated with reproductive and metabolic consequences, even in regularly ovulating women, increasing obesity appears to be associated with decreasing spontaneous pregnancy rates and increased time to pregnancy.14

Obesity and IVF

Women with obesity have reduced success with assisted reproductive technology, an increased number of canceled cycles, and poorer quality oocytes retrieved. A prospective cohort study of nearly 2,000 women reported that every 5 kg of body weight increase (from the patient’s baseline weight at age 18) was associated with a 5% increase in the mean duration of time required for conception (95% confidence interval, 3%-7%).15 Given that approximately 90% of these women had regular menstrual cycles, ovulatory dysfunction was not the suspected pathophysiology.

A meta-analysis of 21 cohort studies reported a lower likelihood of live birth following in vitro fertilization for women with obesity, compared with normal-weight women (risk ratio, 0.85; 95% CI, 0.82-0.87).16 A further subgroup analysis that evaluated only women with PCOS showed a reduction in the live birth rate following IVF for individuals with obesity, compared with normal-weight individuals (RR, 0.78; 95% CI, 0.74-0.82).

In a retrospective study of almost 500,000 fresh autologous IVF cycles, women with obesity had a 6% reduction in pregnancy rates and a 13% reduction in live birth rates, compared with normal-weight women. Both high and low BMI were associated with an increased risk of low birth weight and preterm delivery.17 The live birth rates per transfer for normal-weight and higher-weight women were 38% and 33%, respectively.

Contrarily, a randomized controlled trial showed that an intensive weight-reduction program resulted in a large weight loss but did not substantially affect live birth rates in women with obesity scheduled for IVF.18

Low BMI

A noteworthy cause of low BMI is functional hypothalamic amenorrhea (FHA), a disorder with low energy availability either from decreased caloric intake and/or excessive energy expenditure associated with eating disorders, excessive exercise, and stress. Consequently, a reduced GnRH drive results in a decreased pulse frequency and amplitude leading to low levels of follicle-stimulating hormone and luteinizing hormone, resulting in anovulation. Correction of lifestyle behaviors related to FHA can restore menstrual cycles. After normal weight is achieved, it appears unlikely that fertility is affected.19 In 47% of adolescent patients with anorexia, menses spontaneously returned within the first 12 months after admission, with an improved prognosis in secondary over primary amenorrhea.20,21 Interestingly, mildly and significantly underweight infertile women have pregnancy and live birth rates similar to normal-weight patients after IVF treatment.22

Pregnancy is complicated in underweight women, resulting in an increased risk of anemia, fetal growth retardation, and low birth weight, as well as preterm birth.21

Take-home message

The extremes of BMI both impair natural reproduction. Elevated BMI reduces success with IVF but rapid weight loss prior to IVF does not improve outcomes. A normal BMI is the goal for optimal reproductive and pregnancy health.

Dr. Trolice is director of the IVF Center in Winter Park, Fla., and professor of obstetrics and gynecology at the University of Central Florida, Orlando.
 

References

1. Talumaa B et al. Obesity Rev. 2022;23:e13494.

2. https://bit.ly/3rHCivE.

3. Eknoyan G. Nephrol Dial Transplant. 2008;23:47-51.

4. Wells JCK. Dis Models Mech. 2012;5:595-607.

5. Brewer CJ and Balen AH. Reproduction. 2010;140:347-64.

6. Silvestris E et al. Reprod Biol Endocrinol. 2018;16:22.

7. Wise LA et al. Hum Reprod. 2010;25:253-64.

8. Bellver J. Curr Opin Obstet Gynecol. 2022;34:114-21.

9.
Dickey RP et al. Am J Obstet Gynecol. 2013;209:349.e1.

10. Alwash SM et al. Obes Res Clin Pract. 2021;15:425-30
.

11. Cnattingius S et al. JAMA. 2013;309:2362-70.

12. Aune D et al. JAMA. 2014;311:1536-46.

13. Sam S. Obes Manag. 2007;3:69-73.

14. van der Steeg JW et al. Hum Reprod. 2008;23:324-8.

15. Gaskins AJ et al. Obstet Gynecol. 2015;126:850-8.

16. Sermondade N et al. Hum Reprod Update. 2019;25:439-519.

17. Kawwass JF et al. Fertil Steril. 2016;106[7]:1742-50.

18. Einarsson S et al. Hum Reprod. 2017;32:1621-30.

19. Chaer R et al. Diseases. 2020;8:46.

20. Dempfle A et al. Psychiatry. 2013;13:308.

21. Verma A and Shrimali L. J Clin Diagn Res. 2012;6:1531-3.

22. Romanski PA et al. Reprod Biomed Online. 2020;42:366-74.

Publications
Publications
Topics
Article Type
Sections
Teambase XML
<?xml version="1.0" encoding="UTF-8"?>
<!--$RCSfile: InCopy_agile.xsl,v $ $Revision: 1.35 $-->
<!--$RCSfile: drupal.xsl,v $ $Revision: 1.7 $-->
<root generator="drupal.xsl" gversion="1.7"> <header> <fileName>160453</fileName> <TBEID>0C045CD9.SIG</TBEID> <TBUniqueIdentifier>MD_0C045CD9</TBUniqueIdentifier> <newsOrJournal>News</newsOrJournal> <publisherName>Frontline Medical Communications</publisherName> <storyname>RR - BMI and cnception</storyname> <articleType>353</articleType> <TBLocation>QC Done-All Pubs</TBLocation> <QCDate>20221027T151048</QCDate> <firstPublished>20221027T151401</firstPublished> <LastPublished>20221027T151401</LastPublished> <pubStatus qcode="stat:"/> <embargoDate/> <killDate/> <CMSDate>20221027T151401</CMSDate> <articleSource/> <facebookInfo/> <meetingNumber/> <byline>Mark Trolice</byline> <bylineText>MARK P. TROLICE, MD</bylineText> <bylineFull>MARK P. TROLICE, MD</bylineFull> <bylineTitleText/> <USOrGlobal/> <wireDocType/> <newsDocType>Column</newsDocType> <journalDocType/> <linkLabel/> <pageRange/> <citation/> <quizID/> <indexIssueDate/> <itemClass qcode="ninat:text"/> <provider qcode="provider:imng"> <name>IMNG Medical Media</name> <rightsInfo> <copyrightHolder> <name>Frontline Medical News</name> </copyrightHolder> <copyrightNotice>Copyright (c) 2015 Frontline Medical News, a Frontline Medical Communications Inc. company. All rights reserved. This material may not be published, broadcast, copied, or otherwise reproduced or distributed without the prior written permission of Frontline Medical Communications Inc.</copyrightNotice> </rightsInfo> </provider> <abstract/> <metaDescription>Arguably, no topic during an infertility consultation generates more of an emotional reaction than discussing body mass index (BMI), particularly when it is hig</metaDescription> <articlePDF/> <teaserImage>290069</teaserImage> <teaser>Female obesity is related to poorer outcomes in natural and assisted conception, including an increased risk of miscarriage. </teaser> <title>BMI and reproduction – weighing the evidence</title> <deck/> <disclaimer/> <AuthorList/> <articleURL/> <doi/> <pubMedID/> <publishXMLStatus/> <publishXMLVersion>1</publishXMLVersion> <useEISSN>0</useEISSN> <urgency/> <pubPubdateYear/> <pubPubdateMonth/> <pubPubdateDay/> <pubVolume/> <pubNumber/> <wireChannels/> <primaryCMSID/> <CMSIDs/> <keywords/> <seeAlsos/> <publications_g> <publicationData> <publicationCode>endo</publicationCode> <pubIssueName/> <pubArticleType/> <pubTopics/> <pubCategories/> <pubSections/> </publicationData> <publicationData> <publicationCode>fp</publicationCode> <pubIssueName/> <pubArticleType/> <pubTopics/> <pubCategories/> <pubSections/> </publicationData> <publicationData> <publicationCode>ob</publicationCode> <pubIssueName/> <pubArticleType/> <pubTopics/> <pubCategories/> <pubSections/> </publicationData> </publications_g> <publications> <term>34</term> <term>15</term> <term canonical="true">23</term> </publications> <sections> <term>52</term> <term>41022</term> <term canonical="true">68807</term> </sections> <topics> <term>261</term> <term canonical="true">287</term> <term>322</term> <term>262</term> </topics> <links> <link> <itemClass qcode="ninat:picture"/> <altRep contenttype="image/jpeg">images/240113ce.jpg</altRep> <description role="drol:caption"/> <description role="drol:credit"/> </link> </links> </header> <itemSet> <newsItem> <itemMeta> <itemRole>Main</itemRole> <itemClass>text</itemClass> <title>BMI and reproduction – weighing the evidence</title> <deck/> </itemMeta> <itemContent> <p>Arguably, no topic during an infertility consultation generates more of an emotional reaction than discussing body mass index (BMI), particularly when it is high. Patients have become increasingly sensitive to weight discussions with their physicians because of concerns about body shaming. Among patients with an elevated BMI, criticism on social media of health care professionals’ counseling and a preemptive presentation of “Don’t Weigh Me” cards have become popular responses. Despite the medical evidence on impaired reproduction with an abnormal BMI, patients are choosing to forgo the topic. Research has demonstrated “extensive evidence [of] strong weight bias” in a wide range of health staff.<sup>1</sup> A “viral” TikTok study revealed that medical “gaslighting” founded in weight stigma and bias is harmful, as reported on KevinMD.com.<sup>2</sup> This month, we review the effect of abnormal BMI, both high and low, on reproduction and pregnancy.</p> <p>[[{"fid":"290069","view_mode":"medstat_image_flush_left","fields":{"format":"medstat_image_flush_left","field_file_image_alt_text[und][0][value]":"A card requesting &amp;quot;Please don't weigh me unless it's (really) medically necessary.&amp;quot;","field_file_image_credit[und][0][value]":"","field_file_image_caption[und][0][value]":""},"type":"media","attributes":{"class":"media-element file-medstat_image_flush_left"}}]]A method to assess relative weight was first described in 1832 as its ratio in kilograms divided by the square of the height in meters, or the Quetelet Index. The search for a functional assessment of relative body weight began after World War II when reports by actuaries noted the increased mortality of overweight policyholders. The relationship between weight and cardiovascular disease was further revealed in epidemiologic studies. The Quetelet Index became the BMI in 1972.<sup>3</sup><br/><br/>Weight measurement is a mainstay in the assessment of a patient’s vital signs along with blood pressure, pulse rate, respiration rate, and temperature. Weight is vital to the calculation of medication dosage – for instance, administration of conscious sedative drugs, methotrexate, and gonadotropins. Some state boards of medicine, such as Florida, have a limitation on patient BMI at office-based surgery centers (40 kg/m<sup>2</sup>). <br/><br/></p> <h2>Obesity is a disease </h2> <p>As reported by the World Health Organization in 2022, the disease of obesity is an epidemic afflicting more than 1 billion people worldwide, or 1 in 8 individuals globally.<sup>4</sup> The health implications of an elevated BMI include increased mortality, diabetes, heart disease, and stroke, physical limitations to activities of daily living, and complications affecting reproduction.</p> <p>Female obesity is related to poorer outcomes in natural and assisted conception, including an increased risk of miscarriage. Compared with normal-weight women, those with obesity are three times more likely to have ovulatory dysfunction,<sup>5</sup> infertility,<sup>6</sup> a lower chance for conception,<sup>7</sup> higher rate of miscarriage, and low birth weight.<sup>8,9</sup> <span class="Hyperlink">During pregnancy, women with obesity have three to four times higher rates of gestational diabetes and preeclampsia</span>,<sup>10</sup> as well as likelihood of delivering preterm,<sup>11</sup> having a fetus with macrosomia and birth defects, and a 1.3- to 2.1-times higher risk of stillbirth.<sup>12</sup><br/><br/>Obesity is present in 40%-80% of women with polycystic ovary syndrome,<sup>13</sup> the most common cause of ovulatory dysfunction from dysregulation of the hypothalamic-pituitary-ovarian axis. While PCOS is associated with reproductive and metabolic consequences, even in regularly ovulating women, increasing obesity appears to be associated with decreasing spontaneous pregnancy rates and increased time to pregnancy.<sup>14</sup></p> <h2>Obesity and IVF</h2> <p>Women with obesity have reduced success with assisted reproductive technology, an increased number of canceled cycles, and poorer quality oocytes retrieved. A prospective cohort study of nearly 2,000 women reported that every 5 kg of body weight increase (from the patient’s baseline weight at age 18) was associated with a 5% increase in the mean duration of time required for conception (95% confidence interval, 3%-7%).<sup>15</sup> Given that approximately 90% of these women had regular menstrual cycles, ovulatory dysfunction was not the suspected pathophysiology. </p> <p>A meta-analysis of 21 cohort studies reported a lower likelihood of live birth following in vitro fertilization for women with obesity, compared with normal-weight women (risk ratio, 0.85; 95% CI, 0.82-0.87).<sup>16</sup> A further subgroup analysis that evaluated only women with PCOS showed a reduction in the live birth rate following IVF for individuals with obesity, compared with normal-weight individuals (RR, 0.78; 95% CI, 0.74-0.82).<br/><br/>In a retrospective study of almost 500,000 fresh autologous IVF cycles, women with obesity had a 6% reduction in pregnancy rates and a 13% reduction in live birth rates, compared with normal-weight women. Both high and low BMI were associated with an increased risk of low birth weight and preterm delivery.<sup>17</sup> The live birth rates per transfer for normal-weight and higher-weight women were 38% and 33%, respectively.<br/><br/>Contrarily, a randomized controlled trial showed that an intensive weight-reduction program resulted in a large weight loss but did not substantially affect live birth rates in women with obesity scheduled for IVF.<sup>18</sup></p> <h2>Low BMI</h2> <p>A noteworthy cause of low BMI is functional hypothalamic amenorrhea (FHA), a disorder with low energy availability either from decreased caloric intake and/or excessive energy expenditure associated with eating disorders, excessive exercise, and stress. Consequently, a reduced GnRH drive results in a decreased pulse frequency and amplitude leading to low levels of follicle-stimulating hormone and luteinizing hormone, resulting in anovulation. Correction of lifestyle behaviors related to FHA can restore menstrual cycles. After normal weight is achieved, it appears unlikely that fertility is affected.<sup>19</sup> In 47% of adolescent patients with anorexia, menses spontaneously returned within the first 12 months after admission, with an improved prognosis in secondary over primary amenorrhea.<sup>20,21</sup> Interestingly, mildly and significantly underweight infertile women have pregnancy and live birth rates similar to normal-weight patients after IVF treatment.<sup>22</sup></p> <p>Pregnancy is complicated in underweight women, resulting in an increased risk of anemia, fetal growth retardation, and low birth weight, as well as preterm birth.<sup>21</sup></p> <h2>Take-home message</h2> <p>The extremes of BMI both impair natural reproduction. Elevated BMI reduces success with IVF but rapid weight loss prior to IVF does not improve outcomes. A normal BMI is the goal for optimal reproductive and pregnancy health. <span class="end"/></p> <p> <em>Dr. Trolice is director of the IVF Center in Winter Park, Fla., and professor of obstetrics and gynecology at the University of Central Florida, Orlando.<br/><br/></em> </p> <h2>References</h2> <p>1. Talumaa B et al. <span class="Hyperlink"><a href="https://onlinelibrary.wiley.com/doi/10.1111/obr.13494">Obesity Rev. 2022;23:e13494</a>.</span></p> <p>2. <span class="Hyperlink"><a href="https://bit.ly/3rHCivE">https://bit.ly/3rHCivE</a></span>.<br/><br/>3. Eknoyan G. <span class="Hyperlink"><a href="https://pubmed.ncbi.nlm.nih.gov/17890752/">Nephrol Dial Transplant. 2008;23:47-51</a>.</span><br/><br/>4. Wells JCK. <span class="Hyperlink"><a href="https://journals.biologists.com/dmm/article/5/5/595/3258/The-evolution-of-human-adiposity-and-obesity-where">Dis Models Mech. 2012;5:595-607</a>.</span><br/><br/>5. Brewer CJ and Balen AH. <span class="Hyperlink"><a href="https://rep.bioscientifica.com/view/journals/rep/140/3/347.xml">Reproduction. 2010;140:347-64</a>.</span><br/><br/>6. Silvestris E et al.<b> </b><span class="Hyperlink"><a href="https://rbej.biomedcentral.com/articles/10.1186/s12958-018-0336-z">Reprod Biol Endocrinol. 2018;16:22</a>.</span><br/><br/>7. Wise LA et al. <span class="Hyperlink"><a href="https://academic.oup.com/humrep/article/25/1/253/696509?login=false">Hum Reprod. 2010;25:253-64</a>.<br/><br/></span>8. Bellver J. <span class="Hyperlink"><a href="https://journals.lww.com/co-obgyn/Abstract/2022/06000/BMI_and_miscarriage_after_IVF.4.aspx">Curr Opin Obstet Gynecol. 2022;34:114-21</a>.<br/><br/>9.</span> Dickey RP et al. <span class="Hyperlink"><a href="https://www.ajog.org/article/S0002-9378(13)00541-3/fulltext#%20">Am J Obstet Gynecol. 2013;209:349.e1</a>.<br/><br/>10. Alwash SM et al. <a href="https://www.sciencedirect.com/science/article/abs/pii/S1871403X21001137?via%3Dihub">Obes Res Clin Pract. 2021;15:425-30</a></span>.<br/><br/>11. Cnattingius S et al. <span class="Hyperlink"><a href="https://jamanetwork.com/journals/jama/fullarticle/1696099">JAMA. 2013;309:2362-70</a></span>.<br/><br/>12. Aune D et al. <span class="Hyperlink"><a href="https://jamanetwork.com/journals/jama/fullarticle/1860462">JAMA. 2014;311:1536-46</a></span>.<br/><br/>13.<span class="Hyperlink"> Sam S. <a href="https://www.liebertpub.com/doi/10.1089/obe.2007.0019">Obes Manag. 2007;3:69-73</a></span>. <br/><br/>14. van der Steeg JW et al. <span class="Hyperlink"><a href="https://academic.oup.com/humrep/article/23/2/324/627197?login=false">Hum Reprod. 2008;23:324-8</a>.</span><br/><br/>15.<span class="Hyperlink"> Gaskins AJ et al. <a href="https://journals.lww.com/greenjournal/Abstract/2015/10000/Association_of_Fecundity_With_Changes_in_Adult.26.aspx">Obstet Gynecol. 2015;126:850-8</a>.</span><br/><br/>16. Sermondade <span class="Hyperlink">N et al. <a href="https://academic.oup.com/humupd/article/25/4/439/5426458?login=false">Hum Reprod Update. 2019;25:439-519</a></span>.<br/><br/>17. Kawwass JF et al. <span class="Hyperlink"><a href="https://www.fertstert.org/article/S0015-0282(16)62725-5/fulltext">Fertil Steril. 2016;106[7]:1742-50</a>.</span><br/><br/>18. Einarsson S et al. <span class="Hyperlink"><a href="https://academic.oup.com/humrep/article/32/8/1621/3897609?login=false">Hum Reprod. 2017;32:1621-30</a></span>.<br/><br/>19. Chaer R et al. <span class="Hyperlink"><a href="https://www.mdpi.com/2079-9721/8/4/46">Diseases. 2020;8:46</a></span>.<br/><br/>20. Dempfle A et al. <span class="Hyperlink"><a href="https://bmcpsychiatry.biomedcentral.com/articles/10.1186/1471-244X-13-308">Psychiatry. 2013;13:308</a></span>.<br/><br/>21. Verma A and Shrimali L. <span class="Hyperlink"><a href="https://www.jcdr.net/article_fulltext.asp?issn=0973-709x&amp;year=2012&amp;month=November&amp;volume=6&amp;issue=9&amp;page=1531-1533&amp;id=2551">J Clin Diagn Res. 2012;6:1531-3</a></span>.<br/><br/>22. Romanski PA et al. <span class="Hyperlink"><a href="https://www.rbmojournal.com/article/S1472-6483(20)30576-9/fulltext">Reprod Biomed Online. 2020;42:366-74</a></span>.</p> </itemContent> </newsItem> <newsItem> <itemMeta> <itemRole>teaser</itemRole> <itemClass>text</itemClass> <title/> <deck/> </itemMeta> <itemContent> </itemContent> </newsItem> </itemSet></root>
Disallow All Ads
Content Gating
No Gating (article Unlocked/Free)
Alternative CME
Disqus Comments
Default
Use ProPublica
Hide sidebar & use full width
render the right sidebar.
Conference Recap Checkbox
Not Conference Recap
Clinical Edge
Display the Slideshow in this Article
Medscape Article
Display survey writer
Reuters content
Disable Inline Native ads
WebMD Article

Natural fertility: When less can be more

Article Type
Changed
Mon, 09/19/2022 - 09:20

As reproductive specialists, part of our obligation is to improve a woman’s or couple’s ability to conceive in the most cost-effective manner, ideally through natural attempts at conception. While assisted reproductive technologies (ART) have provided impressive pregnancy rates across many diagnoses, including unexplained infertility, this advanced procedure comes with a significant financial cost to those without insurance and an emotional burden from the lack of a guaranteed outcome. Infertility procedures have minimal associated but potentially significant risks, most importantly multiple gestations. Contrary to popular belief, ovulation induction with intrauterine insemination (IUI) treatment has a greater risk of high-order multiple gestation when compared with IVF, given the inability of the former to control the number of embryos that may enter and implant in the endometrial cavity and the increased use of single embryo transfers with the latter. The specialist should evaluate the woman or couple for the basic issues of ovulation, tubal, and sperm function, as well as for lifestyle and environmental factors that can impede reproduction. As a result, “one size fits all” should not apply to patients, specifically those with infertility. This month’s column will present the detrimental effect of environmental and lifestyle factors on the goal of enhancing fertility through natural cycles of urine luteinizing-hormone timed intercourse.

Dr. Mark P. Trolice

Nutrition

Often overlooked in the infertility evaluation, an optimal diet improves fertility for both partners. Processed meat has been associated with reduced sperm quality. In ART, red meat has been associated with decreased embryo blastocyst formation. Lower trans fatty acids and higher omega-3s may improve fecundity. Considered one of the best overall diets, the Mediterranean diet consists of plant-based foods, such as whole grains, vegetables, legumes, fruits, nuts, seeds, herbs, and spices. Olive oil is the main source of added fat whereas fish, seafood, dairy, and poultry should be eaten in moderation. Fatty fish, such as mackerel, herring, sardines, albacore tuna, and salmon, are rich in omega-3 fatty acids, which have been shown to improve fecundity and IVF success, and have a positive association with blastocyst embryo development.1-3

Stress

The emotional effect of an infertility diagnosis has been demonstrated to be equivalent to a diagnosis of cancer and other major medical morbidities.4 Whether stress causes or is a result of infertility has been a longstanding debate.5 Nevertheless, stress is the number-one reason patients discontinue fertility treatment.6 As fertility specialists, we must be cognizant of the devastation endured by infertility patients and maintain an open dialogue, as well as provide resources for coping strategies and counseling.

One popular method of improving mental health and fertility has been acupuncture. Initial enthusiasm originated from one of the first studies to explore the use of acupuncture during IVF. This was a prospective randomized study that showed treated patients had an approximately 100% improvement in clinical pregnancy rate. Unfortunately, there was no appropriate control group, just untreated controls.7 A subsequent study by the same investigator added a placebo acupuncture control group and did not show a statistically significant increase in pregnancy rates.8 Finally, a meta-analysis and reanalysis did not demonstrate any improvement in pregnancy outcome, whereas three of the studies analyzed suggested a possible reduction in pregnancies; placebo acupuncture was shown to have a higher success rate.9-11 While acupuncture is relatively safe, there appears to be only a placebo effect that may be helpful.

The effect of stress on reproduction has been addressed in one of my previous columns.
 

 

 

Alcohol and caffeine

The damaging effects of alcohol on the fetus during pregnancy are legion – abnormal facial features, microcephaly, low birth weight, hyperactive behavior, vision or hearing deficits, speech and language delays, and intellectual disability. Less known is the amount of alcohol that may have an effect during preconception. One of the first reports on the effect of alcohol on IVF concluded: a 13% decrease in the number of eggs aspirated; a 2.86 times increase in risk of not achieving pregnancy; and a 2.21 times increase in risk of miscarriage. For men, one additional drink per day increased the risk of not achieving a live birth from 2.28 to 8.32 times.12 Subsequent studies demonstrate a 16% reduction in IVF pregnancies in women who have at least four drinks per week; when the couple drank at least four drinks per week, the pregnancy rate decreased by 21%.13

However, a study from Denmark did not demonstrate a negative effect of low to moderate pretreatment amounts of alcohol and caffeine on IVF outcomes.14 Nevertheless, there is evidence that reducing or abstaining from alcohol intake may improve IVF outcomes.15 While there have been reports of higher miscarriage rates from caffeine,16,17 not all reports support a negative association.18

Smoking

The use of tobacco has been estimated to contribute to 13% of female infertility in a dose-response manner, including secondhand smoke. During ART, smoking reduces ovarian response to gonadotropins and decreases IVF success by up to 50%. Discontinuing smoking for 6 months beforehand appears to restore normal outcomes.19-20

The American Society for Reproductive Medicine Practice Committee on smoking provides the following invaluable information to share with patients on the harmful reproductive effects of smoking:21

  • Early menopause by accelerating the loss of eggs.
  • Higher rates of miscarriage and ectopic pregnancy.
  • A decrease in sperm function.
  • Possible genetic damage to eggs and sperm.
  • Reduced sperm in son from maternal smoking.

Weight and exercise

Compared with normal-weight women, those with obesity are three times more likely to have ovulatory dysfunction;22 a lower chance for conception;23 and infertility.24 Obese women have higher rates of miscarriage and recurrent miscarriage, reduced success with ART, an increased number of canceled cycles, and poorer quality oocytes retrieved. During pregnancy, obese women have three to four times higher rates of gestational diabetes and preeclampsia,25 as well as likelihood of having a fetus with macrosomia and birth defects, and a 1.3-2.1 times higher risk of stillbirth.26

Regarding physical activity, the rate of pregnancies (39.0% vs. 16.0%, P = .002) and live births (24.4% vs. 7.4% (P = .004) were higher with regular exercise vs. being sedentary. Obese women who exercised regularly had a live birth rate over threefold higher compared with those who were not active.27 Moderation should be employed given that women who exercise to exhaustion have 2.3 times the odds of fertility problems.28 In men, obesity has been shown to increase estrogens and reduce spermatogenesis. Exercise has improved semen parameters and testosterone. Paternal physical and sedentary activities were not related to clinical pregnancy or live birth rates following infertility treatment.29 As in women, men experience negative effects from high-intensity exercise, including bicycling, which can result in decreased semen parameters, follicle-stimulating hormone, LH, and testosterone levels.30

In couples desiring a more natural approach to infertility, fertility specialists can address environmental and lifestyle factors that may improve reproduction. When natural attempts at conception are not applicable or successful, IUI and ART are appropriate treatment options after considering estimated success rates as well as the physical, emotional, and financial investment of patients.

Dr. Trolice is director of The IVF Center in Winter Park, Fla., and professor of obstetrics and gynecology at the University of Central Florida, Orlando.
 

References

1. Wise LA et al. Am J Epidemiol. 2018;187:60-74.

2. Chui Y-H. Hum Reprod. 2018;33:156-65.

3. Ferreira Braga DPA et al. Reprod Biomed Online. 2015;31:30-8.

4. Domar AD et al. J Psychosom Obstet Gynaecol. 1993;14[suppl]:45-52.

5. Trolice MP. J Assist Reprod Genet. 2021 Apr;38[4]:873-5.

6. Gameiro S et al. Hum Reprod Update. 2012;18[6]:652-69.

7. Paulus WE et al. Fertil Steril. 2002;77:721-4.

8. Paulus WE et al. Hum Reprod. 2003;18:S18(abstr).

9. Wing SSE et al. Hum Reprod. 2009;24:341-8.

10. Hong Zheng C et al. Fertil Steril. 2012;97:599-611.

11. Meldrum DR et al. Fertil Steril. 2013;99:1821-4.

12. Klonoff-Cohen H et al. Fertil Steril. 2003;79:330-9.

13. Rossi BV et al. Obstet Gynecol. 2011;117:136-42.

14. Abadia L et al. Hum Reprod. 2017;32:1846-54.

15. Gormack AA et al. Hum Reprod. 2015;30:1617.

16. James JE. BMJ Evid Based Med. 2021;26:114-15.

17. Gaskins AJ et al. Eur J Nutr. 2018 Feb;57:107-17.

18. Machtinger R et al. Fertil Steril. 2017;108:1026-33.

19. Hughes EG et al. Fertil Steril. 1994;62:807.

20. de Ziegler D et al. Fertil Steril. 2013;100:927-8.

21. Practice Committee of the American Society for Reproductive Medicine. Fertil Steril. 2018;110:611-8.

22. Brewer CJ, Balen AH. Reproduction. 2010;140:347-64.

23. Wise LA et al. Hum Reprod. 2010;25:253-64.

24. Silvestris S et al. Reprod Biol Endocrinol. 2018;16[1]:22.

25. Alwash SM et al. Obes Res Clin Pract. 2021;15:425-30.

26. Aune D et al. JAMA. 2014;311:1536-46.

27. Palomba S et al. Reprod Biomed Online. 2014;29:72-9.

28. Gudmundsdottir SL et al. Hum Reprod. 2009;24[12]:3196-204.

29. Gaskins AJ et al. Hum Reprod. 2014;29:2575-82.

30. Wise LA et al. Fertil Steril. 2011;95:1025-30.

Publications
Topics
Sections

As reproductive specialists, part of our obligation is to improve a woman’s or couple’s ability to conceive in the most cost-effective manner, ideally through natural attempts at conception. While assisted reproductive technologies (ART) have provided impressive pregnancy rates across many diagnoses, including unexplained infertility, this advanced procedure comes with a significant financial cost to those without insurance and an emotional burden from the lack of a guaranteed outcome. Infertility procedures have minimal associated but potentially significant risks, most importantly multiple gestations. Contrary to popular belief, ovulation induction with intrauterine insemination (IUI) treatment has a greater risk of high-order multiple gestation when compared with IVF, given the inability of the former to control the number of embryos that may enter and implant in the endometrial cavity and the increased use of single embryo transfers with the latter. The specialist should evaluate the woman or couple for the basic issues of ovulation, tubal, and sperm function, as well as for lifestyle and environmental factors that can impede reproduction. As a result, “one size fits all” should not apply to patients, specifically those with infertility. This month’s column will present the detrimental effect of environmental and lifestyle factors on the goal of enhancing fertility through natural cycles of urine luteinizing-hormone timed intercourse.

Dr. Mark P. Trolice

Nutrition

Often overlooked in the infertility evaluation, an optimal diet improves fertility for both partners. Processed meat has been associated with reduced sperm quality. In ART, red meat has been associated with decreased embryo blastocyst formation. Lower trans fatty acids and higher omega-3s may improve fecundity. Considered one of the best overall diets, the Mediterranean diet consists of plant-based foods, such as whole grains, vegetables, legumes, fruits, nuts, seeds, herbs, and spices. Olive oil is the main source of added fat whereas fish, seafood, dairy, and poultry should be eaten in moderation. Fatty fish, such as mackerel, herring, sardines, albacore tuna, and salmon, are rich in omega-3 fatty acids, which have been shown to improve fecundity and IVF success, and have a positive association with blastocyst embryo development.1-3

Stress

The emotional effect of an infertility diagnosis has been demonstrated to be equivalent to a diagnosis of cancer and other major medical morbidities.4 Whether stress causes or is a result of infertility has been a longstanding debate.5 Nevertheless, stress is the number-one reason patients discontinue fertility treatment.6 As fertility specialists, we must be cognizant of the devastation endured by infertility patients and maintain an open dialogue, as well as provide resources for coping strategies and counseling.

One popular method of improving mental health and fertility has been acupuncture. Initial enthusiasm originated from one of the first studies to explore the use of acupuncture during IVF. This was a prospective randomized study that showed treated patients had an approximately 100% improvement in clinical pregnancy rate. Unfortunately, there was no appropriate control group, just untreated controls.7 A subsequent study by the same investigator added a placebo acupuncture control group and did not show a statistically significant increase in pregnancy rates.8 Finally, a meta-analysis and reanalysis did not demonstrate any improvement in pregnancy outcome, whereas three of the studies analyzed suggested a possible reduction in pregnancies; placebo acupuncture was shown to have a higher success rate.9-11 While acupuncture is relatively safe, there appears to be only a placebo effect that may be helpful.

The effect of stress on reproduction has been addressed in one of my previous columns.
 

 

 

Alcohol and caffeine

The damaging effects of alcohol on the fetus during pregnancy are legion – abnormal facial features, microcephaly, low birth weight, hyperactive behavior, vision or hearing deficits, speech and language delays, and intellectual disability. Less known is the amount of alcohol that may have an effect during preconception. One of the first reports on the effect of alcohol on IVF concluded: a 13% decrease in the number of eggs aspirated; a 2.86 times increase in risk of not achieving pregnancy; and a 2.21 times increase in risk of miscarriage. For men, one additional drink per day increased the risk of not achieving a live birth from 2.28 to 8.32 times.12 Subsequent studies demonstrate a 16% reduction in IVF pregnancies in women who have at least four drinks per week; when the couple drank at least four drinks per week, the pregnancy rate decreased by 21%.13

However, a study from Denmark did not demonstrate a negative effect of low to moderate pretreatment amounts of alcohol and caffeine on IVF outcomes.14 Nevertheless, there is evidence that reducing or abstaining from alcohol intake may improve IVF outcomes.15 While there have been reports of higher miscarriage rates from caffeine,16,17 not all reports support a negative association.18

Smoking

The use of tobacco has been estimated to contribute to 13% of female infertility in a dose-response manner, including secondhand smoke. During ART, smoking reduces ovarian response to gonadotropins and decreases IVF success by up to 50%. Discontinuing smoking for 6 months beforehand appears to restore normal outcomes.19-20

The American Society for Reproductive Medicine Practice Committee on smoking provides the following invaluable information to share with patients on the harmful reproductive effects of smoking:21

  • Early menopause by accelerating the loss of eggs.
  • Higher rates of miscarriage and ectopic pregnancy.
  • A decrease in sperm function.
  • Possible genetic damage to eggs and sperm.
  • Reduced sperm in son from maternal smoking.

Weight and exercise

Compared with normal-weight women, those with obesity are three times more likely to have ovulatory dysfunction;22 a lower chance for conception;23 and infertility.24 Obese women have higher rates of miscarriage and recurrent miscarriage, reduced success with ART, an increased number of canceled cycles, and poorer quality oocytes retrieved. During pregnancy, obese women have three to four times higher rates of gestational diabetes and preeclampsia,25 as well as likelihood of having a fetus with macrosomia and birth defects, and a 1.3-2.1 times higher risk of stillbirth.26

Regarding physical activity, the rate of pregnancies (39.0% vs. 16.0%, P = .002) and live births (24.4% vs. 7.4% (P = .004) were higher with regular exercise vs. being sedentary. Obese women who exercised regularly had a live birth rate over threefold higher compared with those who were not active.27 Moderation should be employed given that women who exercise to exhaustion have 2.3 times the odds of fertility problems.28 In men, obesity has been shown to increase estrogens and reduce spermatogenesis. Exercise has improved semen parameters and testosterone. Paternal physical and sedentary activities were not related to clinical pregnancy or live birth rates following infertility treatment.29 As in women, men experience negative effects from high-intensity exercise, including bicycling, which can result in decreased semen parameters, follicle-stimulating hormone, LH, and testosterone levels.30

In couples desiring a more natural approach to infertility, fertility specialists can address environmental and lifestyle factors that may improve reproduction. When natural attempts at conception are not applicable or successful, IUI and ART are appropriate treatment options after considering estimated success rates as well as the physical, emotional, and financial investment of patients.

Dr. Trolice is director of The IVF Center in Winter Park, Fla., and professor of obstetrics and gynecology at the University of Central Florida, Orlando.
 

References

1. Wise LA et al. Am J Epidemiol. 2018;187:60-74.

2. Chui Y-H. Hum Reprod. 2018;33:156-65.

3. Ferreira Braga DPA et al. Reprod Biomed Online. 2015;31:30-8.

4. Domar AD et al. J Psychosom Obstet Gynaecol. 1993;14[suppl]:45-52.

5. Trolice MP. J Assist Reprod Genet. 2021 Apr;38[4]:873-5.

6. Gameiro S et al. Hum Reprod Update. 2012;18[6]:652-69.

7. Paulus WE et al. Fertil Steril. 2002;77:721-4.

8. Paulus WE et al. Hum Reprod. 2003;18:S18(abstr).

9. Wing SSE et al. Hum Reprod. 2009;24:341-8.

10. Hong Zheng C et al. Fertil Steril. 2012;97:599-611.

11. Meldrum DR et al. Fertil Steril. 2013;99:1821-4.

12. Klonoff-Cohen H et al. Fertil Steril. 2003;79:330-9.

13. Rossi BV et al. Obstet Gynecol. 2011;117:136-42.

14. Abadia L et al. Hum Reprod. 2017;32:1846-54.

15. Gormack AA et al. Hum Reprod. 2015;30:1617.

16. James JE. BMJ Evid Based Med. 2021;26:114-15.

17. Gaskins AJ et al. Eur J Nutr. 2018 Feb;57:107-17.

18. Machtinger R et al. Fertil Steril. 2017;108:1026-33.

19. Hughes EG et al. Fertil Steril. 1994;62:807.

20. de Ziegler D et al. Fertil Steril. 2013;100:927-8.

21. Practice Committee of the American Society for Reproductive Medicine. Fertil Steril. 2018;110:611-8.

22. Brewer CJ, Balen AH. Reproduction. 2010;140:347-64.

23. Wise LA et al. Hum Reprod. 2010;25:253-64.

24. Silvestris S et al. Reprod Biol Endocrinol. 2018;16[1]:22.

25. Alwash SM et al. Obes Res Clin Pract. 2021;15:425-30.

26. Aune D et al. JAMA. 2014;311:1536-46.

27. Palomba S et al. Reprod Biomed Online. 2014;29:72-9.

28. Gudmundsdottir SL et al. Hum Reprod. 2009;24[12]:3196-204.

29. Gaskins AJ et al. Hum Reprod. 2014;29:2575-82.

30. Wise LA et al. Fertil Steril. 2011;95:1025-30.

As reproductive specialists, part of our obligation is to improve a woman’s or couple’s ability to conceive in the most cost-effective manner, ideally through natural attempts at conception. While assisted reproductive technologies (ART) have provided impressive pregnancy rates across many diagnoses, including unexplained infertility, this advanced procedure comes with a significant financial cost to those without insurance and an emotional burden from the lack of a guaranteed outcome. Infertility procedures have minimal associated but potentially significant risks, most importantly multiple gestations. Contrary to popular belief, ovulation induction with intrauterine insemination (IUI) treatment has a greater risk of high-order multiple gestation when compared with IVF, given the inability of the former to control the number of embryos that may enter and implant in the endometrial cavity and the increased use of single embryo transfers with the latter. The specialist should evaluate the woman or couple for the basic issues of ovulation, tubal, and sperm function, as well as for lifestyle and environmental factors that can impede reproduction. As a result, “one size fits all” should not apply to patients, specifically those with infertility. This month’s column will present the detrimental effect of environmental and lifestyle factors on the goal of enhancing fertility through natural cycles of urine luteinizing-hormone timed intercourse.

Dr. Mark P. Trolice

Nutrition

Often overlooked in the infertility evaluation, an optimal diet improves fertility for both partners. Processed meat has been associated with reduced sperm quality. In ART, red meat has been associated with decreased embryo blastocyst formation. Lower trans fatty acids and higher omega-3s may improve fecundity. Considered one of the best overall diets, the Mediterranean diet consists of plant-based foods, such as whole grains, vegetables, legumes, fruits, nuts, seeds, herbs, and spices. Olive oil is the main source of added fat whereas fish, seafood, dairy, and poultry should be eaten in moderation. Fatty fish, such as mackerel, herring, sardines, albacore tuna, and salmon, are rich in omega-3 fatty acids, which have been shown to improve fecundity and IVF success, and have a positive association with blastocyst embryo development.1-3

Stress

The emotional effect of an infertility diagnosis has been demonstrated to be equivalent to a diagnosis of cancer and other major medical morbidities.4 Whether stress causes or is a result of infertility has been a longstanding debate.5 Nevertheless, stress is the number-one reason patients discontinue fertility treatment.6 As fertility specialists, we must be cognizant of the devastation endured by infertility patients and maintain an open dialogue, as well as provide resources for coping strategies and counseling.

One popular method of improving mental health and fertility has been acupuncture. Initial enthusiasm originated from one of the first studies to explore the use of acupuncture during IVF. This was a prospective randomized study that showed treated patients had an approximately 100% improvement in clinical pregnancy rate. Unfortunately, there was no appropriate control group, just untreated controls.7 A subsequent study by the same investigator added a placebo acupuncture control group and did not show a statistically significant increase in pregnancy rates.8 Finally, a meta-analysis and reanalysis did not demonstrate any improvement in pregnancy outcome, whereas three of the studies analyzed suggested a possible reduction in pregnancies; placebo acupuncture was shown to have a higher success rate.9-11 While acupuncture is relatively safe, there appears to be only a placebo effect that may be helpful.

The effect of stress on reproduction has been addressed in one of my previous columns.
 

 

 

Alcohol and caffeine

The damaging effects of alcohol on the fetus during pregnancy are legion – abnormal facial features, microcephaly, low birth weight, hyperactive behavior, vision or hearing deficits, speech and language delays, and intellectual disability. Less known is the amount of alcohol that may have an effect during preconception. One of the first reports on the effect of alcohol on IVF concluded: a 13% decrease in the number of eggs aspirated; a 2.86 times increase in risk of not achieving pregnancy; and a 2.21 times increase in risk of miscarriage. For men, one additional drink per day increased the risk of not achieving a live birth from 2.28 to 8.32 times.12 Subsequent studies demonstrate a 16% reduction in IVF pregnancies in women who have at least four drinks per week; when the couple drank at least four drinks per week, the pregnancy rate decreased by 21%.13

However, a study from Denmark did not demonstrate a negative effect of low to moderate pretreatment amounts of alcohol and caffeine on IVF outcomes.14 Nevertheless, there is evidence that reducing or abstaining from alcohol intake may improve IVF outcomes.15 While there have been reports of higher miscarriage rates from caffeine,16,17 not all reports support a negative association.18

Smoking

The use of tobacco has been estimated to contribute to 13% of female infertility in a dose-response manner, including secondhand smoke. During ART, smoking reduces ovarian response to gonadotropins and decreases IVF success by up to 50%. Discontinuing smoking for 6 months beforehand appears to restore normal outcomes.19-20

The American Society for Reproductive Medicine Practice Committee on smoking provides the following invaluable information to share with patients on the harmful reproductive effects of smoking:21

  • Early menopause by accelerating the loss of eggs.
  • Higher rates of miscarriage and ectopic pregnancy.
  • A decrease in sperm function.
  • Possible genetic damage to eggs and sperm.
  • Reduced sperm in son from maternal smoking.

Weight and exercise

Compared with normal-weight women, those with obesity are three times more likely to have ovulatory dysfunction;22 a lower chance for conception;23 and infertility.24 Obese women have higher rates of miscarriage and recurrent miscarriage, reduced success with ART, an increased number of canceled cycles, and poorer quality oocytes retrieved. During pregnancy, obese women have three to four times higher rates of gestational diabetes and preeclampsia,25 as well as likelihood of having a fetus with macrosomia and birth defects, and a 1.3-2.1 times higher risk of stillbirth.26

Regarding physical activity, the rate of pregnancies (39.0% vs. 16.0%, P = .002) and live births (24.4% vs. 7.4% (P = .004) were higher with regular exercise vs. being sedentary. Obese women who exercised regularly had a live birth rate over threefold higher compared with those who were not active.27 Moderation should be employed given that women who exercise to exhaustion have 2.3 times the odds of fertility problems.28 In men, obesity has been shown to increase estrogens and reduce spermatogenesis. Exercise has improved semen parameters and testosterone. Paternal physical and sedentary activities were not related to clinical pregnancy or live birth rates following infertility treatment.29 As in women, men experience negative effects from high-intensity exercise, including bicycling, which can result in decreased semen parameters, follicle-stimulating hormone, LH, and testosterone levels.30

In couples desiring a more natural approach to infertility, fertility specialists can address environmental and lifestyle factors that may improve reproduction. When natural attempts at conception are not applicable or successful, IUI and ART are appropriate treatment options after considering estimated success rates as well as the physical, emotional, and financial investment of patients.

Dr. Trolice is director of The IVF Center in Winter Park, Fla., and professor of obstetrics and gynecology at the University of Central Florida, Orlando.
 

References

1. Wise LA et al. Am J Epidemiol. 2018;187:60-74.

2. Chui Y-H. Hum Reprod. 2018;33:156-65.

3. Ferreira Braga DPA et al. Reprod Biomed Online. 2015;31:30-8.

4. Domar AD et al. J Psychosom Obstet Gynaecol. 1993;14[suppl]:45-52.

5. Trolice MP. J Assist Reprod Genet. 2021 Apr;38[4]:873-5.

6. Gameiro S et al. Hum Reprod Update. 2012;18[6]:652-69.

7. Paulus WE et al. Fertil Steril. 2002;77:721-4.

8. Paulus WE et al. Hum Reprod. 2003;18:S18(abstr).

9. Wing SSE et al. Hum Reprod. 2009;24:341-8.

10. Hong Zheng C et al. Fertil Steril. 2012;97:599-611.

11. Meldrum DR et al. Fertil Steril. 2013;99:1821-4.

12. Klonoff-Cohen H et al. Fertil Steril. 2003;79:330-9.

13. Rossi BV et al. Obstet Gynecol. 2011;117:136-42.

14. Abadia L et al. Hum Reprod. 2017;32:1846-54.

15. Gormack AA et al. Hum Reprod. 2015;30:1617.

16. James JE. BMJ Evid Based Med. 2021;26:114-15.

17. Gaskins AJ et al. Eur J Nutr. 2018 Feb;57:107-17.

18. Machtinger R et al. Fertil Steril. 2017;108:1026-33.

19. Hughes EG et al. Fertil Steril. 1994;62:807.

20. de Ziegler D et al. Fertil Steril. 2013;100:927-8.

21. Practice Committee of the American Society for Reproductive Medicine. Fertil Steril. 2018;110:611-8.

22. Brewer CJ, Balen AH. Reproduction. 2010;140:347-64.

23. Wise LA et al. Hum Reprod. 2010;25:253-64.

24. Silvestris S et al. Reprod Biol Endocrinol. 2018;16[1]:22.

25. Alwash SM et al. Obes Res Clin Pract. 2021;15:425-30.

26. Aune D et al. JAMA. 2014;311:1536-46.

27. Palomba S et al. Reprod Biomed Online. 2014;29:72-9.

28. Gudmundsdottir SL et al. Hum Reprod. 2009;24[12]:3196-204.

29. Gaskins AJ et al. Hum Reprod. 2014;29:2575-82.

30. Wise LA et al. Fertil Steril. 2011;95:1025-30.

Publications
Publications
Topics
Article Type
Sections
Teambase XML
<?xml version="1.0" encoding="UTF-8"?>
<!--$RCSfile: InCopy_agile.xsl,v $ $Revision: 1.35 $-->
<!--$RCSfile: drupal.xsl,v $ $Revision: 1.7 $-->
<root generator="drupal.xsl" gversion="1.7"> <header> <fileName>159765</fileName> <TBEID>0C044F1F.SIG</TBEID> <TBUniqueIdentifier>MD_0C044F1F</TBUniqueIdentifier> <newsOrJournal>News</newsOrJournal> <publisherName>Frontline Medical Communications</publisherName> <storyname/> <articleType>353</articleType> <TBLocation>QC Done-All Pubs</TBLocation> <QCDate>20220916T165232</QCDate> <firstPublished>20220919T091505</firstPublished> <LastPublished>20220919T091505</LastPublished> <pubStatus qcode="stat:"/> <embargoDate/> <killDate/> <CMSDate>20220919T091505</CMSDate> <articleSource/> <facebookInfo/> <meetingNumber/> <byline>Mark P Trolice</byline> <bylineText>MARK P. TROLICE, MD</bylineText> <bylineFull>MARK P. TROLICE, MD</bylineFull> <bylineTitleText/> <USOrGlobal/> <wireDocType/> <newsDocType/> <journalDocType/> <linkLabel/> <pageRange/> <citation/> <quizID/> <indexIssueDate/> <itemClass qcode="ninat:text"/> <provider qcode="provider:imng"> <name>IMNG Medical Media</name> <rightsInfo> <copyrightHolder> <name>Frontline Medical News</name> </copyrightHolder> <copyrightNotice>Copyright (c) 2015 Frontline Medical News, a Frontline Medical Communications Inc. company. All rights reserved. This material may not be published, broadcast, copied, or otherwise reproduced or distributed without the prior written permission of Frontline Medical Communications Inc.</copyrightNotice> </rightsInfo> </provider> <abstract/> <metaDescription>As reproductive specialists, part of our obligation is to improve a woman’s or couple’s ability to conceive in the most cost-effective manner, ideally through n</metaDescription> <articlePDF/> <teaserImage>284599</teaserImage> <teaser>As reproductive specialists, part of our obligation is to improve a woman’s or couple’s ability to conceive in the most cost-effective manner, ideally through natural attempts at conception.</teaser> <title>Natural fertility: When less can be more</title> <deck/> <disclaimer/> <AuthorList/> <articleURL/> <doi/> <pubMedID/> <publishXMLStatus/> <publishXMLVersion>1</publishXMLVersion> <useEISSN>0</useEISSN> <urgency/> <pubPubdateYear/> <pubPubdateMonth/> <pubPubdateDay/> <pubVolume/> <pubNumber/> <wireChannels/> <primaryCMSID/> <CMSIDs/> <keywords/> <seeAlsos/> <publications_g> <publicationData> <publicationCode>FP</publicationCode> <pubIssueName/> <pubArticleType/> <pubTopics/> <pubCategories/> <pubSections/> <journalTitle/> <journalFullTitle/> <copyrightStatement>Copyright 2017 Frontline Medical News</copyrightStatement> </publicationData> <publicationData> <publicationCode>OB</publicationCode> <pubIssueName/> <pubArticleType/> <pubTopics/> <pubCategories/> <pubSections/> <journalTitle/> <journalFullTitle/> <copyrightStatement/> </publicationData> </publications_g> <publications> <term>15</term> <term canonical="true">23</term> </publications> <sections> <term>52</term> <term>41022</term> <term canonical="true">68807</term> </sections> <topics> <term>322</term> <term canonical="true">287</term> </topics> <links> <link> <itemClass qcode="ninat:picture"/> <altRep contenttype="image/jpeg">images/24010881.jpg</altRep> <description role="drol:caption">Dr. Mark P. Trolice</description> <description role="drol:credit"/> </link> </links> </header> <itemSet> <newsItem> <itemMeta> <itemRole>Main</itemRole> <itemClass>text</itemClass> <title>Natural fertility: When less can be more</title> <deck/> </itemMeta> <itemContent> <p>As reproductive specialists, part of our obligation is to improve a woman’s or couple’s ability to conceive in the most cost-effective manner, ideally through natural attempts at conception. While assisted reproductive technologies (ART) have provided impressive pregnancy rates across many diagnoses, including unexplained infertility, this advanced procedure comes with a significant financial cost to those without insurance and an emotional burden from the lack of a guaranteed outcome. Infertility procedures have minimal associated but potentially significant risks, most importantly multiple gestations. Contrary to popular belief, ovulation induction with intrauterine insemination (IUI) treatment has a greater risk of high-order multiple gestation when compared with IVF, given the inability of the former to control the number of embryos that may enter and implant in the endometrial cavity and the increased use of single embryo transfers with the latter. The specialist should evaluate the woman or couple for the basic issues of ovulation, tubal, and sperm function, as well as for lifestyle and environmental factors that can impede reproduction. As a result, “one size fits all” should not apply to patients, specifically those with infertility. This month’s column will present the detrimental effect of environmental and lifestyle factors on the goal of enhancing fertility through natural cycles of urine luteinizing-hormone timed intercourse. </p> <p> [[{"fid":"284599","view_mode":"medstat_image_flush_left","fields":{"format":"medstat_image_flush_left","field_file_image_alt_text[und][0][value]":"Dr. Trolice is director of The IVF Center in Winter Park, Fla., and professor of obstetrics and gynecology at the University of Central Florida, Orlando.","field_file_image_credit[und][0][value]":"","field_file_image_caption[und][0][value]":"Dr. Mark P. Trolice"},"type":"media","attributes":{"class":"media-element file-medstat_image_flush_left"}}]] </p> <h2>Nutrition</h2> <p>Often overlooked in the infertility evaluation, an optimal diet improves fertility for both partners. Processed meat has been associated with reduced sperm quality. In ART, red meat has been associated with decreased embryo blastocyst formation. Lower trans fatty acids and higher omega-3s may improve fecundity. Considered one of the best overall diets, the Mediterranean diet consists of plant-based foods, such as whole grains, vegetables, legumes, fruits, nuts, seeds, herbs, and spices. Olive oil is the main source of added fat whereas fish, seafood, dairy, and poultry should be eaten in moderation. Fatty fish, such as mackerel, herring, sardines, albacore tuna, and salmon, are rich in omega-3 fatty acids, which have been shown to improve fecundity and IVF success, and have a positive association with blastocyst embryo development.<sup>1-3 </sup></p> <h2>Stress</h2> <p>The emotional effect of an infertility diagnosis has been demonstrated to be equivalent to a diagnosis of cancer and other major medical morbidities.<sup>4</sup> Whether stress causes or is a result of infertility has been a longstanding debate.<sup>5</sup> Nevertheless, stress is the number-one reason patients discontinue fertility treatment.<sup>6</sup> As fertility specialists, we must be cognizant of the devastation endured by infertility patients and maintain an open dialogue, as well as provide resources for coping strategies and counseling. </p> <p>One popular method of improving mental health and fertility has been acupuncture. Initial enthusiasm originated from one of the first studies to explore the use of acupuncture during IVF. This was a prospective randomized study that showed treated patients had an approximately 100% improvement in clinical pregnancy rate. Unfortunately, there was no appropriate control group, just untreated controls.<sup>7</sup> A subsequent study by the same investigator added a placebo acupuncture control group and did not show a statistically significant increase in pregnancy rates.<sup>8</sup> Finally, a meta-analysis and reanalysis did not demonstrate any improvement in pregnancy outcome, whereas three of the studies analyzed suggested a possible reduction in pregnancies; placebo acupuncture was shown to have a higher success rate.<sup>9-11</sup> While acupuncture is relatively safe, there appears to be only a placebo effect that may be helpful.<br/><br/>The effect of stress on reproduction has been addressed in one of my previous columns. <br/><br/></p> <h2>Alcohol and caffeine</h2> <p>The damaging effects of alcohol on the fetus during pregnancy are legion – abnormal facial features, microcephaly, low birth weight, hyperactive behavior, vision or hearing deficits, speech and language delays, and intellectual disability. Less known is the amount of alcohol that may have an effect during preconception. One of the first reports on the effect of alcohol on IVF concluded: a 13% decrease in the number of eggs aspirated; a 2.86 times increase in risk of not achieving pregnancy; and a 2.21 times increase in risk of miscarriage. For men, one additional drink per day increased the risk of not achieving a live birth from 2.28 to 8.32 times.<sup>12</sup> Subsequent studies demonstrate a 16% reduction in IVF pregnancies in women who have at least four drinks per week; when the couple drank at least four drinks per week, the pregnancy rate decreased by 21%.<sup>13</sup></p> <p>However, a study from Denmark did not demonstrate a negative effect of low to moderate pretreatment amounts of alcohol and caffeine on IVF outcomes.<sup>14</sup> Nevertheless, there is evidence that reducing or abstaining from alcohol intake may improve IVF outcomes.<sup>15</sup> While there have been reports of higher miscarriage rates from caffeine,<sup>16,17</sup> not all reports support a negative association.<sup>18</sup></p> <h2>Smoking</h2> <p>The use of tobacco has been estimated to contribute to 13% of female infertility in a dose-response manner, including secondhand smoke. During ART, smoking reduces ovarian response to gonadotropins and decreases IVF success by up to 50%. Discontinuing smoking for 6 months beforehand appears to restore normal outcomes.<sup>19-20</sup></p> <p>The American Society for Reproductive Medicine Practice Committee on smoking provides the following invaluable information to share with patients on the harmful reproductive effects of smoking:<sup>21</sup></p> <ul class="body"> <li>Early menopause by accelerating the loss of eggs.</li> <li>Higher rates of miscarriage and ectopic pregnancy.</li> <li>A decrease in sperm function.</li> <li>Possible genetic damage to eggs and sperm.</li> <li>Reduced sperm in son from maternal smoking.</li> </ul> <h2>Weight and exercise</h2> <p>Compared with normal-weight women, those with obesity are three times more likely to have ovulatory dysfunction;<sup>22</sup> a lower chance for conception;<sup>23</sup> and infertility.<sup>24</sup> Obese women have higher rates of miscarriage and recurrent miscarriage, reduced success with ART, an increased number of canceled cycles, and poorer quality oocytes retrieved. During pregnancy, obese women have three to four times higher rates of gestational diabetes and preeclampsia,<sup>25</sup> as well as likelihood of having a fetus with macrosomia and birth defects, and a 1.3-2.1 times higher risk of stillbirth.<sup>26</sup></p> <p>Regarding physical activity, the rate of pregnancies (39.0% vs. 16.0%, <em>P</em> = .002) and live births (24.4% vs. 7.4% (<em>P</em> = .004) were higher with regular exercise vs. being sedentary. Obese women who exercised regularly had a live birth rate over threefold higher compared with those who were not active.<sup>27</sup> Moderation should be employed given that women who exercise to exhaustion have 2.3 times the odds of fertility problems.<sup>28</sup> In men, obesity has been shown to increase estrogens and reduce spermatogenesis. Exercise has improved semen parameters and testosterone. Paternal physical and sedentary activities were not related to clinical pregnancy or live birth rates following infertility treatment.<sup>29</sup> As in women, men experience negative effects from high-intensity exercise, including bicycling, which can result in decreased semen parameters, follicle-stimulating hormone, LH, and testosterone levels.<sup>30</sup> <br/><br/>In couples desiring a more natural approach to infertility, fertility specialists can address environmental and lifestyle factors that may improve reproduction. When natural attempts at conception are not applicable or successful, IUI and ART are appropriate treatment options after considering estimated success rates as well as the physical, emotional, and financial investment of patients.</p> <p> <em>Dr. Trolice is director of The IVF Center in Winter Park, Fla., and professor of obstetrics and gynecology at the University of Central Florida, Orlando.<br/><br/></em> </p> <h2>References</h2> <p>1. Wise LA et al. <span class="Hyperlink"><a href="https://academic.oup.com/aje/article/187/1/60/3863377?login=false">Am J Epidemiol. 2018;187:60-74</a></span>. <br/><br/>2. Chui Y-H. <span class="Hyperlink"><a href="https://academic.oup.com/humrep/article/33/1/156/4616532?login=false">Hum Reprod. 2018;33:156-65</a></span>. <br/><br/>3. Ferreira Braga DPA et al. <span class="Hyperlink"><a href="https://www.rbmojournal.com/article/S1472-6483(15)00148-0/fulltext">Reprod Biomed Online. 2015;31:30-8</a></span>. <br/><br/>4. Domar AD et al. <span class="Hyperlink"><a href="https://pubmed.ncbi.nlm.nih.gov/8142988/">J Psychosom Obstet Gynaecol. 1993;14[suppl]:45-52</a></span>. <br/><br/>5. Trolice MP. <span class="Hyperlink"><a href="https://link.springer.com/article/10.1007/s10815-021-02145-w">J Assist Reprod Genet. 2021 Apr;38[4]:873-5</a></span>. <br/><br/>6. Gameiro S et al.<span class="Hyperlink"><a href="https://academic.oup.com/humupd/article/18/6/652/628767"> Hum Reprod Update. 2012;18[6]:652-69</a></span>.<br/><br/>7. Paulus WE et al. <span class="Hyperlink"><a href="https://www.fertstert.org/article/S0015-0282(01)03273-3/fulltext">Fertil Steril. 2002;77:721-4</a></span>. <br/><br/>8. Paulus WE et al. Hum Reprod. 2003;18:S18(abstr). <br/><br/>9. Wing SSE et al. <span class="Hyperlink"><a href="https://academic.oup.com/humrep/article/24/2/341/809764?login=false">Hum Reprod. 2009;24:341-8</a></span>. <br/><br/>10. Hong Zheng C et al. <span class="Hyperlink"><a href="https://www.fertstert.org/article/S0015-0282(11)02859-7/fulltext">Fertil Steril. 2012;97:599-611</a></span>. <br/><br/>11. Meldrum DR et al.<span class="Hyperlink"><a href="https://www.sciencedirect.com/science/article/abs/pii/S0015028213000083"> Fertil Steril. 2013;99:1821-4</a></span>. <br/><br/>12. Klonoff-Cohen H et al. <span class="Hyperlink"><a href="https://www.fertstert.org/article/S0015-0282(02)04582-X/fulltext">Fertil Steril. 2003;79:330-9</a></span>. <br/><br/>13. Rossi BV et al. <span class="Hyperlink"><a href="https://journals.lww.com/greenjournal/Abstract/2011/01000/Effect_of_Alcohol_Consumption_on_In_Vitro.20.aspx">Obstet Gynecol. 2011;117:136-42</a></span>. <br/><br/>14. Abadia L et al.<span class="Hyperlink"><a href="https://academic.oup.com/humrep/article/32/9/1846/4079651?login=false"> Hum Reprod. 2017;32:1846-54</a></span>. <br/><br/>15. Gormack AA et al. <span class="Hyperlink"><a href="https://academic.oup.com/humrep/article/30/7/1617/653135">Hum Reprod. 2015;30:1617</a></span>. <br/><br/>16. James JE. <span class="Hyperlink"><a href="https://ebm.bmj.com/content/26/3/114">BMJ Evid Based Med. 2021;26:114-15</a></span>. <br/><br/>17. Gaskins AJ et al. <span class="Hyperlink"><a href="https://link.springer.com/article/10.1007/s00394-016-1301-2">Eur J Nutr. 2018 Feb;57:107-17</a></span>. <br/><br/>18. Machtinger R et al. <span class="Hyperlink"><a href="https://www.fertstert.org/article/S0015-0282(17)31911-8/fulltext">Fertil Steril. 2017;108:1026-33</a></span>. <br/><br/>19. Hughes EG et al. <span class="Hyperlink"><a href="https://www.sciencedirect.com/science/article/abs/pii/S0015028216570095?via%3Dihub">Fertil Steril. 1994;62:807</a></span>. <br/><br/>20. de Ziegler D et al. <span class="Hyperlink"><a href="https://pubmed.ncbi.nlm.nih.gov/23796366/">Fertil Steril. 2013;100:927-8</a></span>. <br/><br/>21. Practice Committee of the American Society for Reproductive Medicine. <span class="Hyperlink"><a href="https://www.fertstert.org/article/S0015-0282(18)30492-8/fulltext">Fertil Steril. 2018;110:611-8</a></span>. <br/><br/>22. Brewer CJ, Balen AH. <span class="Hyperlink"><a href="https://rep.bioscientifica.com/view/journals/rep/140/3/347.xml">Reproduction. 2010;140:347-64</a></span>. <br/><br/>23. Wise LA et al. <span class="Hyperlink"><a href="https://academic.oup.com/humrep/article/25/1/253/696509">Hum Reprod. 2010;25:253-64</a></span>. <br/><br/>24. Silvestris S et al. <span class="Hyperlink"><a href="https://rbej.biomedcentral.com/articles/10.1186/s12958-018-0336-z">Reprod Biol Endocrinol. 2018;16[1]:22</a>. </span><br/><br/>25. Alwash SM et al. <span class="Hyperlink"><a href="https://www.sciencedirect.com/science/article/abs/pii/S1871403X21001137?via%3Dihub">Obes Res Clin Pract. 2021;15:425-30</a></span>. <br/><br/>26. Aune D et al. <span class="Hyperlink"><a href="https://jamanetwork.com/journals/jama/fullarticle/1860462">JAMA. 2014;311:1536-46</a></span>. <br/><br/>27. Palomba S et al. <span class="Hyperlink"><a href="https://www.rbmojournal.com/article/S1472-6483(14)00137-0/fulltext">Reprod Biomed Online. 2014;29:72-9</a></span>.<br/><br/>28. Gudmundsdottir SL et al. <span class="Hyperlink"><a href="https://academic.oup.com/humrep/article/24/12/3196/647657">Hum Reprod. 2009;24[12]:3196-204</a></span>. <br/><br/>29. Gaskins AJ et al. <span class="Hyperlink"><a href="https://academic.oup.com/humrep/article/29/11/2575/2426973">Hum Reprod. 2014;29:2575-82</a></span>. <br/><br/>30. Wise LA et al. <span class="Hyperlink"><a href="https://www.fertstert.org/article/S0015-0282(10)02776-7/fulltext">Fertil Steril. 2011;95:1025-30</a></span>. </p> </itemContent> </newsItem> <newsItem> <itemMeta> <itemRole>teaser</itemRole> <itemClass>text</itemClass> <title/> <deck/> </itemMeta> <itemContent> </itemContent> </newsItem> </itemSet></root>
Disallow All Ads
Content Gating
No Gating (article Unlocked/Free)
Alternative CME
Disqus Comments
Default
Use ProPublica
Hide sidebar & use full width
render the right sidebar.
Conference Recap Checkbox
Not Conference Recap
Clinical Edge
Display the Slideshow in this Article
Medscape Article
Display survey writer
Reuters content
Disable Inline Native ads
WebMD Article